Currently Empty: ₹0.00
Cardiology Module June 2022 MRCP Part-1
Time limit: 0
Quiz Summary
0 of 141 Questions completed
Questions:
Information
You have already completed the quiz before. Hence you can not start it again.
Quiz is loading…
You must sign in or sign up to start the quiz.
You must first complete the following:
Results
Quiz complete. Results are being recorded.
Results
0 of 141 Questions answered correctly
Your time:
Time has elapsed
You have reached 0 of 0 point(s), (0)
Earned Point(s): 0 of 0, (0)
0 Essay(s) Pending (Possible Point(s): 0)
Categories
- Cardiology Module June 2022 MRCP Part-1 0%
- Neurology Module July 2022 MRCP Part-1 0%
- 1
- 2
- 3
- 4
- 5
- 6
- 7
- 8
- 9
- 10
- 11
- 12
- 13
- 14
- 15
- 16
- 17
- 18
- 19
- 20
- 21
- 22
- 23
- 24
- 25
- 26
- 27
- 28
- 29
- 30
- 31
- 32
- 33
- 34
- 35
- 36
- 37
- 38
- 39
- 40
- 41
- 42
- 43
- 44
- 45
- 46
- 47
- 48
- 49
- 50
- 51
- 52
- 53
- 54
- 55
- 56
- 57
- 58
- 59
- 60
- 61
- 62
- 63
- 64
- 65
- 66
- 67
- 68
- 69
- 70
- 71
- 72
- 73
- 74
- 75
- 76
- 77
- 78
- 79
- 80
- 81
- 82
- 83
- 84
- 85
- 86
- 87
- 88
- 89
- 90
- 91
- 92
- 93
- 94
- 95
- 96
- 97
- 98
- 99
- 100
- 101
- 102
- 103
- 104
- 105
- 106
- 107
- 108
- 109
- 110
- 111
- 112
- 113
- 114
- 115
- 116
- 117
- 118
- 119
- 120
- 121
- 122
- 123
- 124
- 125
- 126
- 127
- 128
- 129
- 130
- 131
- 132
- 133
- 134
- 135
- 136
- 137
- 138
- 139
- 140
- 141
- Current
- Review
- Answered
- Correct
- Incorrect
-
Question 1 of 141
1. Question
Which of the following pathological condition may be responsible for a Hypokalaemic Hypertension?
CorrectIncorrectHint
Liddle’s Syndrome is characterised by hypokalaemia, hypertension and decrease plasma renin and aldosterone concentration state. It is also known as pseudo-hyperaldosteronism._x000D_
Bartter’s Syndrome is associated with hypokalaemia and normotensive state._x000D_
In case of type IV Renal Tubular Acidosis (RTA), there is a hyporeninaemic and hypoaldosteronism. In diabetic nephropathy there is similar plasma renin and aldosterone concentration is found. So, instead of hypokalaemia, hyperkalaemia is more prominent finding here. -
Question 2 of 141
2. Question
You have joined as an intern in sports medicine centre for daily screening of cardiovascular health of newly appointed athletes. The main concern is to screen hypertrophic obstructive cardiomyopathy (HOCM) and its probability of severity at the time of diagnosis. Among the following options which one is the most dreaded risk factor for sudden circulatory collapse in HOCM?
CorrectIncorrectHint
Hypertrophic cardiomyopathy (HCM) is a critical disorder of heart where interventricular septum has asymmetrical hypertrophy and progressively it can obstruct the left ventricular outflow tract and ultimately causing sudden circulatory collapse. It runs in hereditary. So, family history of sudden cardiac death (SCD) might be present. Very often it may be asymptomatic in many affected persons. Usually, it may worsen during exercise or physical activity where significant drop of systolic blood pressure (around 25 mmHg) has been documented. Other risk factors like early onset and associated with non-sustained VT are significant.
-
Question 3 of 141
3. Question
Which one of the following statements is correct regarding the Austin Flint murmur?
CorrectIncorrectHint
The correct statement regarding the Austin Flint murmur is B. It is a low frequency mid / late diastolic murmur. This is because the Austin Flint murmur is a rumbling sound that is best heard at the cardiac apex in patients with severe aortic regurgitation. The other statements are incorrect for the following reasons:
_x000D_
- _x000D_
- A. It is associated with a loud first heart sound: This is false because the Austin Flint murmur is not associated with a loud first heart sound, but rather with a soft or absent first heart sound due to the premature closure of the mitral valve.
- C. It can be distinguished from the murmur of mitral stenosis by absence of presystolic accentuation: This is false because the Austin Flint murmur can have presystolic accentuation, which is the increase in intensity of the murmur just before the first heart sound. This can make it difficult to differentiate from the murmur of mitral stenosis, which also has presystolic accentuation.
- D. It does not occur in aortic incompetence secondary to an aortitis: This is false because the Austin Flint murmur can occur in any cause of severe aortic regurgitation, including aortitis, which is the inflammation of the aortic wall.
- E. It is an early sign of aortic regurgitation: This is false because the Austin Flint murmur is a late sign of aortic regurgitation, indicating a significant degree of regurgitant volume and left ventricular overload.
_x000D_
_x000D_
_x000D_
_x000D_
-
Question 4 of 141
4. Question
Which one of the following options best describes troponin?
CorrectIncorrectHint
Troponin is a complex of three regulatory proteins (troponin C, troponin I, and troponin T) that are integral to muscle contraction in skeletal muscle and cardiac muscle, but not smooth muscle. Troponin is attached to the protein tropomyosin and lies within the groove between actin filaments in muscle tissue. Therefore, the best option that describes troponin is C. A component of thin filaments.
_x000D_
Troponin, along with actin and tropomyosin, forms the thin filaments of muscle tissue. It regulates muscle contraction by binding calcium ions. Troponin consists of three subunits: TnC, which binds calcium; TnI, which inhibits myosin binding; and TnT, which binds tropomyosin. Calcium binding to TnC causes a conformational change in the thin filament that allows myosin, a motor protein in the thick filament, to interact with actin and generate force and/or movement. Without calcium, tropomyosin blocks this interaction and keeps the muscles relaxed.
-
Question 5 of 141
5. Question
Which one of the following mechanisms best explains the action of simvastatin?
CorrectIncorrectHint
The correct answer is C. Decreases hepatic cholesterol synthesis.
_x000D_
Simvastatin is a prodrug that is hydrolysed in the body to produce an active metabolite that inhibits HMG-CoA reductase, the enzyme that catalyses the rate-limiting step in cholesterol biosynthesis. By blocking this enzyme, simvastatin reduces the amount of cholesterol produced by the liver. This leads to a decrease in plasma low-density lipoprotein (LDL) cholesterol and very low-density lipoproteins (VLDL), and an increase in high-density lipoprotein (HDL) cholesterol.
_x000D_
The other incorrect answer options can be ruled out as follows:
_x000D_
- _x000D_
- A. Increases peroxisomal beta-oxidation of fatty acids: This is incorrect because simvastatin does not affect peroxisomal beta-oxidation, which is a process that breaks down long-chain fatty acids into acetyl-CoA and hydrogen peroxide. Simvastatin acts on the cholesterol biosynthesis pathway, not the fatty acid oxidation pathway.
- B. Inhibits cholesterol absorption: This is incorrect because simvastatin does not inhibit cholesterol absorption in the intestine, which is the mechanism of action of another class of lipid-lowering drugs called ezetimibe. Simvastatin acts on the liver, where it reduces the production of cholesterol by inhibiting HMG-CoA reductase.
- D. Activates peroxisome proliferator-activated receptor (PPAR)-alpha agonist: This is incorrect because simvastatin does not activate PPAR-alpha, which is a nuclear receptor that regulates the expression of genes involved in fatty acid oxidation, lipoprotein metabolism, and inflammation. Simvastatin does not have any direct effect on gene transcription or inflammation.
- E. Bile acid sequestration: This is incorrect because simvastatin does not sequester bile acids in the intestine, which is the mechanism of action of another class of lipid-lowering drugs called bile acid sequestrants. Simvastatin does not interfere with the enterohepatic circulation of bile acids, which are derived from cholesterol and aid in fat digestion.
_x000D_
_x000D_
_x000D_
_x000D_
-
Question 6 of 141
6. Question
Which one of the following is the first to get elevated following a myocardial infarction (MI)?
CorrectIncorrectHint
The first biomarker to get elevated following a myocardial infarction is D. Myoglobin.
_x000D_
Myoglobin is a protein that stores oxygen in the muscle cells, including the heart muscle. When the heart muscle is damaged by a lack of blood supply, myoglobin is released into the bloodstream. Myoglobin levels can rise within 1 to 3 hours after the onset of a myocardial infarction and peak within 6 to 9 hours.
_x000D_
The other biomarkers are less sensitive or specific for myocardial infarction because:
_x000D_
- _x000D_
- A. Creatine kinase-MB (CK-MB) is an enzyme that is found mainly in the heart muscle. CK-MB levels can rise within 4 to 6 hours after a myocardial infarction and peak within 12 to 24 hours. However, CK-MB can also be elevated in other conditions that affect the skeletal muscle, such as trauma, inflammation, or exercise.
- B. Serum creatine phosphokinase (CPK) is a general term for the enzymes that catalyse the transfer of phosphate groups from creatine to adenosine triphosphate (ATP), the main energy source for the cells. CPK has three isoforms: CPK-MM (found in skeletal muscle), CPK-MB (found in heart muscle), and CPK-BB (found in brain tissue). Creatine phosphokinase (CPK) levels can rise within 4 to 8 hours after a myocardial infarction and peak within 12 to 24 hours. However, creatine phosphokinase (CPK) is not specific for myocardial infarction, as it can be elevated in many other conditions that affect the muscle or brain, such as stroke, seizure, or rhabdomyolysis.
- C. Lactate dehydrogenase (LDH) is an enzyme that is involved in the conversion of lactate to pyruvate, a key step in the glycolysis pathway that produces energy for the cells. Lactate dehydrogenase (LDH) has five isoforms: LDH-1 and LDH-2 (found in heart and red blood cells), LDH-3 (found in lung and lymphocytes), LDH-4 (found in liver and skeletal muscle), and LDH-5 (found in liver and skeletal muscle). Lactate dehydrogenase (LDH) levels can rise within 12 to 24 hours after a myocardial infarction and peak within 48 to 72 hours. However, lactate dehydrogenase (LDH) is not specific for myocardial infarction, as it can be elevated in many other conditions that cause tissue damage or haemolysis, such as liver disease, kidney disease, or anaemia.
- E. Troponin I is a protein that is part of the troponin complex, which regulates the contraction and relaxation of the heart muscle. Troponin I levels can rise within 3 to 6 hours after a myocardial infarction and peak within 12 to 16 hours. Troponin I is highly specific for myocardial infarction, as it is not found in other tissues. However, troponin I is not the first biomarker to get elevated following a myocardial infarction, as it is slower than myoglobin.
_x000D_
_x000D_
_x000D_
_x000D_
-
Question 7 of 141
7. Question
Which one of the following is most commonly associated with prolonged QT interval?
CorrectIncorrectHint
The correct answer is E. Hypocalcaemia. This is a condition where the blood calcium level is lower than normal. Calcium is one of the minerals that control the electrical activity of the heart, and low calcium can cause the heart muscle to take longer to contract and relax, resulting in a prolonged QT interval. Other causes of prolonged QT interval include hypokalaemia, hypomagnesaemia, hypothermia, myocardial ischemia, some genetic conditions, and some medications.
_x000D_
A prolonged QT interval increases the risk of developing abnormal heart rhythms, such as Torsades de pointes (TdP), which can lead to sudden cardiac arrest and death. Therefore, it is important to identify and treat the underlying cause of prolonged QT interval and avoid any triggers that may worsen the condition.
-
Question 8 of 141
8. Question
Which one of the following is a manifestation of Marfan’s syndrome?
CorrectIncorrectHint
Marfan syndrome is a rare genetic disorder that affects the connective tissue, which supports and anchors your organs and other structures in your body. It is caused by a mutation in the FBN1 gene, which makes a protein called fibrillin-1. Marfan syndrome can affect many different parts of the body, such as the skeleton, heart, eyes, blood vessels, and lungs.
_x000D_
The correct answer to this question is B. Increased upper : lower body ratio. People with Marfan syndrome often have a tall and thin build, with long arms, legs, fingers, and toes. They may also have a high, arched palate, a breastbone that protrudes outward or dips inward, and an abnormally curved spine. These are some of the signs and symptoms of Marfan syndrome that affect the skeletal system.
_x000D_
The other options are incorrect:
_x000D_
- _x000D_
- Option A: Pulmonary stenosis is a condition where the valve that controls the blood flow from the heart to the lungs is narrowed. This is not a common complication of Marfan syndrome, although some people with the disorder may have other heart problems, such as mitral valve prolapse or aortic aneurysm.
- Option C: Retinal detachment is a serious eye condition that occurs when the retina, the light-sensitive layer of tissue at the back of the eye, peels away from its underlying support. It can cause vision loss or blindness if not treated promptly. People with Marfan syndrome may have eye problems, such as extreme near-sightedness, cataracts, glaucoma, or dislocation of the lens, but retinal detachment is not a typical feature of the disorder. The incidence of retinal detachment in Marfan syndrome ranges from 5% to 25%, which is similar to the general population.
- Option D: Autosomal recessive inheritance is a pattern of genetic transmission in which both copies of a gene must be mutated for a person to be affected by a disorder. Marfan syndrome is an autosomal dominant disorder, which means that only one copy of the FBN1 gene needs to be mutated for a person to have the condition.
- Option E: Cognitive impairment is a term that describes problems with thinking, learning, memory, or concentration. It can have many different causes, such as brain injury, dementia, mental illness, or developmental disorders. Marfan syndrome does not affect the brain or cause cognitive impairment.
_x000D_
_x000D_
_x000D_
_x000D_
-
Question 9 of 141
9. Question
Which one of the following infections is least likely to cause myocarditis?
CorrectIncorrectHint
Syphilis (option C) is a bacterial infection that can affect various organs, including the heart. However, myocarditis due to syphilis is very rare and usually occurs in the late stage of the infection, known as quaternary syphilis. Quaternary syphilis can cause damage to the aorta and the aortic valve, leading to aneurysm and regurgitation. However, quaternary syphilis is uncommon in the modern era due to the availability of antibiotics. Therefore, syphilis is the least likely cause of myocarditis among the options.
_x000D_
The other options can be ruled out as follows:
_x000D_
- _x000D_
- Coxsackie virus: Coxsackie virus is a common cause of viral myocarditis. It can infect the heart muscle and cause inflammation, necrosis, and fibrosis. Coxsackie virus can also trigger an autoimmune response that damages the heart tissue. Coxsackie virus can cause various symptoms, such as chest pain, shortness of breath, fatigue, fever, and arrhythmias.
- Diphtheria: Diphtheria is a bacterial infection that can cause a thick coating in the throat, difficulty breathing, and heart problems. Diphtheria can cause myocarditis by producing toxins that damage the heart muscle. Diphtheria can also affect the conduction system of the heart and cause arrhythmias, heart block, or cardiac arrest.
- Chagas disease: Chagas disease is a parasitic infection that can cause fever, swelling, and digestive problems. Chagas disease can cause myocarditis by destroying the heart muscle cells or triggering an immune response. Chagas disease can also cause chronic heart damage, such as dilated cardiomyopathy, heart failure, or sudden cardiac death.
- Toxoplasmosis: Toxoplasmosis is an infection caused by a parasite called Toxoplasma gondii. It can affect various organs, such as the brain, eyes, lungs, and heart. However, myocarditis due to toxoplasmosis is very rare, even in people with weakened immune systems. Most cases of toxoplasmosis are asymptomatic or cause mild flu-like symptoms.
_x000D_
_x000D_
_x000D_
_x000D_
-
Question 10 of 141
10. Question
Which one of the following features of the jugular venous waveform marks the closure of the tricuspid valve?
CorrectIncorrectHint
The correct answer is C. “c” wave.
_x000D_
The “c” wave of the jugular venous waveform marks the closure of the tricuspid valve during right ventricular contraction. The “c” wave is the second upward deflection in the waveform and is caused by the bowing of the tricuspid valve into the right atrium. The “c” wave is not ordinarily visible to the naked eye, but can be detected by a pressure transducer.
_x000D_
The other features of the jugular venous waveform are:
_x000D_
- _x000D_
- “a” wave: the first upward deflection in the waveform and is caused by the contraction of the right atrium during atrial systole.
- “v” wave: the third upward deflection in the waveform and is caused by the venous filling of the right atrium while the tricuspid valve is closed in late systole/early diastole.
- “x” descent: the first downward deflection in the waveform and is caused by the atrial relaxation and the downward movement of the tricuspid valve during ventricular systole.
- “y” descent: the second downward deflection in the waveform and is caused by the rapid emptying of the right atrium after the tricuspid valve opens in early diastole.
_x000D_
_x000D_
_x000D_
_x000D_
-
Question 11 of 141
11. Question
Which one of the following drugs shortens the QT interval?
CorrectIncorrectHint
The drug that shortens the QT interval among the options given is B. Digoxin. Digoxin is a cardiac glycoside that can lower the heart rate and increase the contractility of the heart. Digoxin can also shorten the QT interval by enhancing the vagal tone and reducing the sympathetic activity. However, digoxin can also cause arrhythmias, such as atrial fibrillation, atrioventricular block, and ventricular tachycardia, especially in patients with hypokalaemia, hypomagnesaemia, or renal impairment.
_x000D_
The other options are drugs that can prolong the QT interval and increase the risk of Torsade de pointes (TdP), a type of ventricular tachycardia that can lead to ventricular fibrillation and sudden cardiac death. These drugs are:
_x000D_
- _x000D_
- A. Moxonidine: This is a centrally acting antihypertensive agent that can inhibit the release of norepinephrine and lower the blood pressure. Moxonidine can prolong the QT interval by blocking the hERG potassium channel, which is responsible for the repolarization of the cardiac action potential.
- C. Sotalol: This is a beta blocker and a potassium channel blocker that can lower the heart rate and blood pressure, and prevent atrial and ventricular arrhythmias. Sotalol can prolong the QT interval by blocking the rapid component of the delayed rectifier potassium current (Ikr), which is mediated by the hERG channel.
- D. Sodium nitroprusside: This is a vasodilator that can lower the blood pressure and reduce the preload and afterload of the heart. Sodium nitroprusside can prolong the QT interval by increasing the nitric oxide levels, which can inhibit the hERG channel and the L-type calcium channel.
- E. Amiodarone: This is an antiarrhythmic agent that can block multiple ion channels, such as sodium, potassium, calcium, and beta-adrenergic receptors. Amiodarone can prolong the QT interval by blocking the Ikr and the slow component of the delayed rectifier potassium current (Iks), which are both involved in the repolarization of the cardiac action potential.
_x000D_
_x000D_
_x000D_
_x000D_
-
Question 12 of 141
12. Question
Which one of the following compounds has a vasodilation effect?
CorrectIncorrectHint
Vasodilation is the process of widening the blood vessels, which can lower blood pressure and increase blood flow to the tissues. Vasodilation can be caused by various factors, such as temperature, inflammation, exercise, or the release of certain chemicals within the body.
_x000D_
The correct answer to this question is A. Calcitonin gene related peptide (CGRP). Calcitonin gene related peptide (CGRP) is a neuropeptide that is released from sensory nerves and acts as a potent vasodilator. Calcitonin gene related peptide (CGRP) can bind to receptors on the vascular smooth muscle cells and cause relaxation, as well as stimulate the production of nitric oxide, which is another vasodilator. Calcitonin gene related peptide (CGRP) is involved in various physiological and pathological processes, such as migraine, inflammation, wound healing, and cardiovascular regulation.
_x000D_
The other options are not vasodilators, but rather have the opposite effect of vasoconstriction, which is the narrowing of the blood vessels, which can raise blood pressure and reduce blood flow to the tissues. Some of their functions are:
_x000D_
- _x000D_
- B. Somatostatin: Somatostatin is a hormone that inhibits the secretion of various other hormones, such as growth hormone, insulin, and glucagon. Somatostatin also causes vasoconstriction by reducing the release of nitric oxide and increasing the release of endothelin.
- C. Renin: Renin is an enzyme that catalyses the conversion of angiotensinogen to angiotensin I, which is then converted to angiotensin II by another enzyme called angiotensin-converting enzyme (ACE). Angiotensin II is a powerful vasoconstrictor that also stimulates the secretion of aldosterone, which increases sodium and water retention. Renin is part of the renin-angiotensin-aldosterone system (RAAS), which regulates blood pressure and fluid balance.
- D. Endothelin: Endothelin is a peptide that is produced by the endothelial cells that line the blood vessels. Endothelin is one of the most potent vasoconstrictors known, and it also promotes inflammation, cell proliferation, and fibrosis. Endothelin is involved in various cardiovascular diseases, such as hypertension, atherosclerosis, and heart failure.
- E. Antidiuretic hormone (ADH): Antidiuretic hormone (ADH) is a hormone that is secreted by the posterior pituitary gland in response to low blood volume or high blood osmolarity. Antidiuretic hormone (ADH) acts on the kidneys to increase water reabsorption, thus reducing urine output and increasing blood volume. Antidiuretic hormone (ADH) also causes vasoconstriction by stimulating the V1 receptors on the vascular smooth muscle cells.
_x000D_
_x000D_
_x000D_
_x000D_
-
Question 13 of 141
13. Question
Which one of the following arteries is a branch of the axillary artery?
CorrectIncorrectHint
The correct answer is D. Subscapular artery.
_x000D_
The subscapular artery is one of the three branches of the third part of the axillary artery, along with the anterior and posterior circumflex humeral arteries. It is the largest branch of the axillary artery and supplies blood to the subscapular region, the scapula, and the posterior thoracic wall. It also gives rise to the circumflex scapular and thoracodorsal arteries.
_x000D_
The other options are not branches of the axillary artery, but of the brachial artery or its branches. The superior and inferior ulnar collateral arteries are branches of the brachial artery that anastomose with the ulnar artery. The profunda brachii artery is a branch of the brachial artery that runs with the radial nerve and supplies the posterior compartment of the arm. The internal thoracic artery is a branch of the subclavian artery that runs along the anterior thoracic wall and gives rise to the anterior intercostal and pericardiacophrenic arteries.
-
Question 14 of 141
14. Question
What is the mechanism of action of Omega-3-Acid Ethyl esters?
CorrectIncorrectHint
Omega-3-Acid Ethyl esters (popularly known multi-national brand, named OMACOR) helps to reduce the hepatic production of triglyceride-rich very low-density lipoproteins. Possible cellular mechanism includes inhibition of acyl CoA:1,2 diacylglycerol acyltransferase and increase in hepatic mitochondrial and peroxisomal Beta-Oxidation with ultimate effect of reduction in the hepatic triglyceride’s synthesis. It can decrease the cardiovascular events risk (reduced events of atherosclerosis)
-
Question 15 of 141
15. Question
Regarding coronary circulation, which one of the following statements is true?
CorrectIncorrectHint
The correct answer is E. Adenosine is an important mediator of metabolic vasodilatation because adenosine is a potent vasodilator that is released from the myocardium in response to hypoxia, ischemia, or increased metabolic activity. It acts on the coronary arterioles to increase blood flow and oxygen delivery to the heart muscle.
_x000D_
Metabolic vasodilatation is the process by which blood vessels relax and widen in response to increased tissue metabolism, such as during exercise or hypoxia. This allows more blood flow and oxygen delivery to the active tissues.
_x000D_
Adenosine is a nucleoside that is formed from the breakdown of ATP, the main energy source of cells. When tissue metabolism increases, more ATP is consumed and more adenosine is produced. Adenosine then acts on specific receptors on the vascular smooth muscle cells and endothelial cells, causing them to relax and dilate. Adenosine also inhibits the release of vasoconstrictor substances, such as norepinephrine and angiotensin II, that would otherwise counteract the vasodilatation.
_x000D_
Adenosine is especially important for regulating coronary blood flow, which supplies oxygen to the heart muscle. The heart has a high metabolic rate and a low oxygen extraction ratio, meaning that it depends on increased blood flow to meet its oxygen demand. Adenosine is released from the myocardium in proportion to the oxygen consumption and acts on the coronary arterioles to increase blood flow. This mechanism ensures that the heart receives adequate oxygen even during periods of stress or ischaemia.
_x000D_
Here is a brief explanation why the other options are not correct:
_x000D_
- _x000D_
- Option A is false because increased myocardial O2 demand is met primarily by increasing coronary blood flow, not O2 extraction.
- Option B is false because coronary blood flow is directly proportional to myocardial oxygen consumption due to autoregulation.
- Option C is false because coronary blood flow within a normal range of blood pressure is primarily determined by the degree of coronary vascular resistance, not perfusion pressure.
- Option D is false because the vasodilatory reserve of the endocardium is greater than that of the epicardium under normal physiologic conditions.
_x000D_
_x000D_
_x000D_
_x000D_
-
Question 16 of 141
16. Question
In which one of the following circumstances angina due to an imbalance between oxygen supply and demand without atherosclerosis would most likely be seen?
CorrectIncorrectHint
The most likely circumstance in which angina due to an imbalance between oxygen supply and demand without atherosclerosis would be seen is E. Aortic regurgitation. This is because aortic regurgitation is a condition where the aortic valve does not close properly, allowing blood to leak back into the left ventricle. This causes the left ventricle to work harder and increase its oxygen demand, while reducing the effective forward flow of blood and oxygen supply to the coronary arteries.
_x000D_
The other options are less likely because they do not affect the oxygen supply and demand of the heart as much as aortic regurgitation. Cardiac tamponade (option A) is a condition where fluid accumulates in the pericardial sac, compressing the heart and reducing its filling and output. Tricuspid regurgitation (option B) is a condition where the tricuspid valve does not close properly, allowing blood to leak back into the right atrium. This causes the right ventricle to work harder and increase its pressure, but it does not affect the oxygen supply and demand of the left ventricle. Pulmonary regurgitation (option C) is a condition where the pulmonary valve does not close properly, allowing blood to leak back into the right ventricle. This causes the right ventricle to work harder and increase its volume, but it does not affect the oxygen supply and demand of the left ventricle. Right heart failure (option D) is a condition where the right ventricle fails to pump blood effectively to the lungs. This causes congestion and oedema in the systemic circulation, but it does not affect the oxygen supply and demand of the left ventricle. Therefore, the answer is E. Aortic regurgitation.
-
Question 17 of 141
17. Question
Intravenous Bicarbonate used in Cardiorespiratory Arrest in specific situations. Among the following options which one is correct?Increases Cerebral Output.
CorrectIncorrectHint
Bicarbonate is an extra cellular anion mostly. It has a negative inotropic effect and it shifts the oxygen dissociation curve to the left. Ultimately causing inhibition of tissue oxygen delivery. As it is alkaline in nature it can increase extra-cellular PH. If there is no adequate ventilation to wash out Carbon Dioxide (CO2) from the blood, hypercapnia itself decreases the intra cellular PH by diffusion of CO2 from extra cellular compartment to intra cellular compartment.
-
Question 18 of 141
18. Question
Neonatal Cyanosis is the hallmark of which of the following congenital heart disease?
CorrectIncorrectHint
Total Anomalous Pulmonary Venous Connection (TAPVC) primarily associated with Neonatal Cyanosis. It manifests due to abnormal connection of pulmonary veins into the systemic veins or right atrium with or without pulmonary venous obstruction, instead of draining into left atrium. So, ultimately there is mixing of pulmonary venous and systemic venous blood in the right atrium and causing persistent cyanosis.
_x000D_
-
Question 19 of 141
19. Question
Primary prevention trials for the treatment of hypercholesterolaemia reveal a reduction in all-cause mortality.
_x000D_
With which one of the following treatments it is achieved?
CorrectIncorrectHint
The correct answer is C. Statins.
_x000D_
Statins are a class of drugs that lower the level of low-density lipoprotein (LDL) cholesterol in the blood by inhibiting an enzyme called HMG-CoA reductase, which is involved in cholesterol synthesis. Statins have been shown to reduce the risk of cardiovascular events and death in people with high cholesterol, especially those who have other risk factors or existing heart disease.
_x000D_
Several primary prevention trials have demonstrated the benefits of statins in reducing all-cause mortality in people with hypercholesterolaemia. For example, the West of Scotland Coronary Prevention Study (WOSCOPS) found that pravastatin, a type of statin, reduced the risk of death by 22% in men with high cholesterol and no history of heart disease. Similarly, the Anglo-Scandinavian Cardiac Outcomes Trial-Lipid Lowering Arm (ASCOT-LLA) showed that atorvastatin, another statin, reduced the risk of death by 16% in people with high blood pressure and high cholesterol, but no previous cardiovascular disease.
_x000D_
Other treatments for hypercholesterolaemia, such as resins, nicotinic acid, fibrates, and fish oils, may have some effects on lowering low-density lipoprotein (LDL) cholesterol or raising high-density lipoprotein (HDL) cholesterol, but they have not been proven to reduce all-cause mortality in primary prevention trials. Therefore, statins are the most effective and recommended treatment for hypercholesterolaemia in terms of preventing death from any cause.
-
Question 20 of 141
20. Question
During a theoretical session on cardiac physiology, a postgraduate student was asked by the consultant to calculate how much blood (in ml) is ejected every time when a human heart beats, considering the following referential values:
_x000D_
- _x000D_
- Blood Pressure: 140/90 mmHg units.
- Cardiac Output: 6000 ml/minute.
- Heart Rate: 80/minute.
- Urine Output: 5 ml/Kg/hr.
_x000D_
_x000D_
_x000D_
_x000D_
CorrectIncorrectHint
Cardiac output is a term that describes the function of the human heart and the amount of blood that it circulates throughout the body in one minute. The normal cardiac output for a healthy adult is about 5 to 6 litres per minute. Cardiac output is important for maintaining the normal function of the body, because it determines how much oxygen and nutrients are delivered to the cells and tissues, and how much waste products are removed. Cardiac output is influenced by many factors, but the two main ones are the heart rate and the stroke volume. The heart rate is the number of times the heart beats per minute, and the stroke volume is the amount of blood that is ejected from the left ventricle of the heart with each contraction. The formula for calculating cardiac output is:
_x000D_
Cardiac Output (CO) = Heart Rate (HR) x Stroke Volume (SV)
_x000D_
For example, if the heart rate is 80 beats per minute and the stroke volume is 75 millilitres, the cardiac output is:
_x000D_
Cardiac Output (CO) = 80 beats/minute x 75 ml/beat
_x000D_
Cardiac Output (CO) = 6000 ml/minute
_x000D_
Hence, Stroke Volume (SV) = Cardiac Output (CO) ÷ Heart Rate (HR); (6000 ÷ 80 = 75)
_x000D_
This means that the heart pumps 6 litres of blood every minute. The cardiac output is not constant, but it changes according to the needs and demands of the body. For instance, when a person exercises, the heart rate and the stroke volume increase to provide more blood flow and oxygen to the muscles. When a person rests, the heart rate and the stroke volume decrease to conserve energy. The body has mechanisms to regulate the cardiac output and keep it within a normal range, by adjusting the heart rate and the stroke volume accordingly. In the given situation, the blood pressure and the urine output were irrelevant information, because they did not affect the calculation of the cardiac output. The only information that was needed was the heart rate and the stroke volume, or the cardiac output itself, which can vary from 5 to 8 litres per minute in a normal adult.
_x000D_
-
Question 21 of 141
21. Question
Congenital Heart Disease may happen about one in hundred babies in UK. Which of the following statement regarding this disorder is true?
CorrectIncorrectHint
Ebstein’s anomaly mostly associated with maternal exposure of lithium carbonate during first trimester. It is characterised primarily by abnormality of tricuspid valve and right ventricle. Usually right sided heart failure, increased right atrial pressure and severe tricuspid regurgitation. Often cyanotic spells are also associated into right to left intra-atrial shunting._x000D_
Ventricular Septal Defect (VSD) is far more common than Atrial Septal Defect (ASD). _x000D_
The most common cause of Congenital Complete Heart Block usually associated with autoimmune disorders like Anti-RO Antibody positive mothers._x000D_
Left sided cardiac chamber did not develop in Hypoplastic Left Heart Syndrome. Here systemic blood circulation takes place through pulmonary artery and patent ductus arteriosus. -
Question 22 of 141
22. Question
Concerning the treatment of acute myocardial infarction (MI), which one of the following statements is correct?
CorrectIncorrectHint
The correct answer to this question is D. Treatment with a dihydropyridine short acting calcium antagonist nifedipine is associated with increased cardiovascular mortality. This statement is based on the results of a randomized trial that compared nifedipine with placebo in patients with acute myocardial infarction (MI). The trial found that nifedipine increased the risk of death, recurrent myocardial infarction (MI), and heart failure in the first month after myocardial infarction (MI). The mechanism of this adverse effect is not fully understood, but it may be related to the reflex increase in heart rate and sympathetic activity caused by nifedipine, which can worsen ischaemia and arrhythmias. Therefore, nifedipine is not recommended for the treatment of acute myocardial infarction (MI), especially in patients with low blood pressure or heart failure.
_x000D_
The other options are incorrect for the following reasons:
_x000D_
Option A. Heparin is beneficial if given with streptokinase: This statement is false. Heparin is an anticoagulant that prevents blood clots from forming or growing. Streptokinase is a thrombolytic agent that dissolves existing blood clots. Both drugs are used to restore blood flow to the heart in patients with acute MI. However, heparin does not enhance the efficacy of streptokinase, and may increase the risk of bleeding complications. Therefore, heparin is not routinely given with streptokinase, unless there is a high risk of clotting or reocclusion.
_x000D_
Option B. Dipyridamole therapy reduces reinfarction within the first year: This statement is false. Dipyridamole is an antiplatelet drug that inhibits the aggregation of platelets, which are involved in blood clotting. It is used to prevent stroke in patients with atrial fibrillation or prosthetic heart valves. However, there is no evidence that dipyridamole reduces the risk of reinfarction or death in patients with acute myocardial infarction (MI). In fact, some studies have suggested that dipyridamole may increase the risk of bleeding and angina in this population. Therefore, dipyridamole is not indicated for the treatment or prevention of acute myocardial infarction (MI).
_x000D_
Option C. A pansystolic murmur developing within the first 24 hours does not require further investigation: This statement is false. A pansystolic murmur is a heart sound that lasts throughout the systole, or the contraction phase of the heart. It can indicate a leaky valve or a hole in the heart wall. A pansystolic murmur developing within the first 24 hours of acute myocardial infarction (MI) may be a sign of a serious complication, such as ventricular septal defect (VSD) or mitral regurgitation. Both conditions can cause heart failure and shock, and require urgent diagnosis and treatment. Therefore, a pansystolic murmur developing within the first 24 hours of acute myocardial infarction (MI) does require further investigation, such as echocardiography or cardiac catheterization.
_x000D_
Option E. Prophylactic lidocaine given in the first 48 hours is effective in preventing ventricular fibrillation: This statement is false. Lidocaine is an antiarrhythmic drug that suppresses abnormal electrical impulses in the heart. Ventricular fibrillation is a life-threatening arrhythmia that causes the heart to quiver instead of pumping blood. It is a common cause of sudden cardiac death in patients with acute myocardial infarction (MI). However, prophylactic lidocaine given in the first 48 hours of acute myocardial infarction (MI) does not prevent ventricular fibrillation, and may increase the risk of other arrhythmias and mortality. Therefore, lidocaine is not recommended for the routine prevention of ventricular fibrillation in patients with acute myocardial infarction (MI).
-
Question 23 of 141
23. Question
Concerning complete atrioventricular septal defects, which one of the following statements is correct?
CorrectIncorrectHint
The correct answer is E. Atrioventricular septal defects are seen frequently in patients with trisomy 21. Trisomy 21, also known as Down syndrome, is a genetic condition that involves an extra chromosome 21.
_x000D_
Atrioventricular septal defects (AVSD) are heart defects that affect the valves and walls between the chambers of the heart. They are caused by abnormal development of the endocardial cushions, which are structures that form the septum and valves during embryonic life.
_x000D_
Trisomy 21, also known as Down syndrome, is a genetic condition that involves an extra copy of chromosome 21. This extra chromosome affects many aspects of development, including the formation of the heart. Studies have shown that genes on chromosome 21, such as CRELD1 and GATA6, are involved in the regulation of endocardial cushion development. Mutations or variations in these genes can increase the risk of atrioventricular septal defects (AVSD) in people with or without Down syndrome.
_x000D_
Therefore, atrioventricular septal defects (AVSD) are seen frequently in patients with trisomy 21 because the extra chromosome 21 disrupts the normal development of the endocardial cushions, leading to defects in the septum and valves of the heart.
_x000D_
The other statements are incorrect because:
_x000D_
- _x000D_
- A. Patients with atrioventricular septal defects do not have a normal mitral valve structure. They have a single large valve that crosses the defect in the wall between the two sides of the heart, instead of two separate valves (tricuspid and mitral).
- B. Atrioventricular septal defects do not include a perimembranous ventricular septal defect. A perimembranous ventricular septal defect is a hole in the lower part of the wall between the ventricles, near the valves. Atrioventricular septal defects involve the whole area of the junction of the upper and lower chambers of the heart.
- C. Subjects with atrioventricular septal defects do not frequently have aortic valve insufficiency. Aortic valve insufficiency is a condition where the aortic valve does not close tightly, allowing blood to leak back into the left ventricle. Atrioventricular septal defects usually cause blood to flow from the left side of the heart to the right side, not the other way around.
- D. Atrioventricular septal defects do not include a coronary sinus atrial septal defect. A coronary sinus atrial septal defect is a hole in the wall between the atria, near the opening of the coronary sinus, which is a large vein that returns blood from the heart muscle to the right atrium. Atrioventricular septal defects affect the center of the heart, where the atria join the ventricles.
_x000D_
_x000D_
_x000D_
_x000D_
-
Question 24 of 141
24. Question
Chronic hypoxia can cause severe pulmonary circulatory vasoconstriction. It is different from systemic circulation in various way. How does pulmonary circulation vary from systemic circulation?
CorrectIncorrectHint
Pulmonary circulation is high compliant, low resistant vascular circuit. So, it will exert low pressures, high flow rates and high compliance.
-
Question 25 of 141
25. Question
Aspirin acts by which of the following mechanism?
CorrectIncorrectHint
Aspirin is anti-platelet agent. It blocks the cyclooxygenase (COX, both l and ll) and helps to stop formation of inflammatory mediators, which attract the platelet aggregation. So, it inhibits platelet aggregation.
-
Question 26 of 141
26. Question
An 88-year-old lady with obesity presents to the Emergency Department with a three-hour history of squeezing central chest pain; associated with sweating and breathlessness.
_x000D_
She has a history of hypertension and type 2 diabetes mellitus (T2DM). Her resting 12-lead electrocardiogram (ECG) demonstrates significant ST depression in the lateral leads. She has been given aspirin 300 mg by the paramedical staffs. The cardiology registrar on-call was called in to discuss the case. There is an on-site cardiac intervention laboratory equipped to manage unstable patients.
_x000D_
The cardiology registrar seeks for an opinion of the senior house officer on the lady’s bleeding risk.
_x000D_
Which one of the following features is associated with a higher bleeding risk?
CorrectIncorrectHint
The lady’s clinical presentation suggests that she has an acute coronary syndrome (ACS), which is a condition where the blood supply to the heart muscle is reduced or blocked due to a clot in a coronary artery. Acute coronary syndrome (ACS) is a serious and potentially life-threatening condition that requires urgent treatment with antithrombotic drugs and revascularization procedures. However, these treatments also increase the risk of bleeding, which can have adverse effects on the patient’s outcome.
_x000D_
To assess the bleeding risk of patients with acute coronary syndrome (ACS), several risk scores have been developed based on clinical and laboratory variables. Some of the common factors that are associated with a higher bleeding risk in patients with acute coronary syndrome (ACS) are:
_x000D_
- _x000D_
- Older age
- Female sex
- Anaemia
- Renal insufficiency
- Elevated white blood cell count
- Prior history of bleeding
- Use of multiple antithrombotic agents
- Invasive procedures
_x000D_
_x000D_
_x000D_
_x000D_
_x000D_
_x000D_
_x000D_
_x000D_
_x000D_
Based on these factors, the most correct answer to the question is E. Low body weight. Low body weight is a proxy for frailty and may increase the risk of bleeding due to lower drug clearance, higher drug exposure, and lower tissue tolerance. Low body weight is also associated with higher mortality in patients with acute coronary syndrome (ACS).
_x000D_
Type 2 diabetes mellitus (T2DM) is not a direct risk factor for bleeding, but it may increase the risk indirectly by causing renal impairment, vascular complications, and platelet dysfunction. Younger age is generally associated with a lower bleeding risk, unless there are other comorbidities or risk factors. Obesity is also not a direct risk factor for bleeding, but it may increase the risk indirectly by causing hypertension, dyslipidaemia, and inflammation. Use of proton-pump inhibitors may reduce the risk of gastrointestinal bleeding in patients taking aspirin or other antiplatelet drugs.
-
Question 27 of 141
27. Question
An 80-year-old lady has had worsening shortness of breath for past few years.
_x000D_
She now has to sleep sitting up on two pillows. She also has difficulty swallowing. She is afebrile and has no history of chest pain.
_x000D_
Recently, she suffered from a stroke with right hemiparesis. A chest x-ray reveals a near normal left ventricular size with a prominent left atrial border.
_x000D_
Which one of the following conditions is most likely to account for these findings?
CorrectIncorrectHint
The correct answer is D. Mitral valve stenosis.
_x000D_
Mitral valve stenosis is a narrowing of the valve between the left atrium and the left ventricle, which reduces the blood flow to the heart. It can cause symptoms such as shortness of breath, difficulty swallowing, fatigue, swollen legs, irregular heartbeat, and stroke.
_x000D_
The other answer options are incorrect for the following reasons:
_x000D_
- _x000D_
- A. Cardiomyopathy is a disease of the heart muscle that affects its function. It can cause symptoms such as shortness of breath, chest pain, fatigue, and swelling. However, it usually also causes enlargement of the left ventricle, which is not seen in the chest x-ray of the patient.
- B. Left renal artery stenosis is a narrowing of the artery that supplies blood to the left kidney. It can cause high blood pressure, reduced kidney function, and abdominal pain. However, it does not directly affect the heart valves or cause stroke.
- C. Essential hypertension is high blood pressure that has no identifiable cause. It can increase the risk of heart disease, stroke, and kidney damage. However, it does not cause mitral valve stenosis or prominent left atrial border.
- E. Aortic coarctation is a narrowing of the aorta, the main artery that carries blood from the heart to the rest of the body. It can cause high blood pressure, chest pain, headache, and leg pain. However, it does not affect the mitral valve or the left atrium.
_x000D_
_x000D_
_x000D_
_x000D_
-
Question 28 of 141
28. Question
An 80-year-old gentleman came to cardiology clinic with complaints of less compliance during treadmill workout session for last few months. Even after initiation of mild to moderate walking, he feels breathlessness and fatigue very quickly. As per him, his compliance was much better even few years back. His echocardiography showed concentric LVH and ECG shows sinus rhythm. He had been in regular exercise for long duration and without any significant past medical history.
_x000D_
What physiological change can cause his decreased compliance to exercise for last the few years?
CorrectIncorrectHint
As we grow older, there is decreased response of parasympathetic withdrawal and as well as sympathetic stimulation on heart. So, ultimately there is reduced cardiovascular response to stress with advancing of age. During exercise, physical stress the heart needs to increase its stroke volume for metabolic demand of the tissues. But with advancement of age, the heart cannot compensate to increase stroke volume due to its reduced sympathetic and parasympathetic withdrawal response. Thus, the failure of the heart’s response during exercise can lead to breathlessness.
-
Question 29 of 141
29. Question
Among the following which is not a component of the Cardiac Electrical Conduction Pathway?
CorrectIncorrectHint
To maintain normal cardiac cycle heart has to depend on its extensive cardiac electrical conduction circuit. The anatomy and the physiology of the conducting system is crucial for understanding Electrocardiogram.
_x000D_
The electrical impulse starts from the sinoatrial node (SAN) (pacemaker of heart). It then descends to the atrioventricular node (AVN) depolarising the left and right atria. During descending it causes the contraction. At the atrioventricular node, a short delay occurs for allowing the atrium to relax. Then the impulse descends down the bundle of His and divides along the left and right branches. Here it causes depolarisation of the right and left ventricles and thus ventricular systole takes place. Once the atrium and ventricle have repolarised, the cycle starts anew.
_x000D_
_x000D_
- _x000D_
- Sinoatrial node
- Atrioventricular node
- Bundle of His
- Left bundle branch
- Left posterior fascicle
- Left anterior fascicle
- Left ventricle.
- Ventricular Septum
- Right Ventricle
- Right Bundle Branch
_x000D_
_x000D_
_x000D_
_x000D_
_x000D_
_x000D_
_x000D_
_x000D_
_x000D_
_x000D_
-
Question 30 of 141
30. Question
An 18-year-old student, being brought by his parents to the Emergency Department for worsening shortness of breath and wheeze for last few days. He has a past medical history of bronchial Asthma and is on high doses of oral corticosteroid. On detailed examination he also complaints of tingling and numbness affecting his toes for the last few months. He was given a full course of antibiotics and oral corticosteroids by the Primary Care Physician few weeks ago but his symptoms did not improve. Other systemic and general findings were unremarkable. There is no family history of any significant medical disorder. He had no addictions in the form of smoking or consuming of alcohol. For the last few months, he was noticing of exercise intolerance. There was no history of travelling in recent past.
_x000D_
Examination revealed:
_x000D_
Pulse Rate: 110/Minute
_x000D_
Blood Pressure: 130/70 mmHg
_x000D_
Respiratory Rate: 24/Minute
_x000D_
Temperature: 97.60F
_x000D_
Oxygen Saturations: 96% On Room Air.
_x000D_
On auscultation bilateral wheeze in the chest and vesicular breath sounds were noted.
_x000D_
Neurological examination revealed decreased sensation in a “glove and stocking” distribution. The rest of the neurological and systemic examination was within normal limits.
_x000D_
Blood parameters:
_x000D_
_x000D_ _x000D_
_x000D_ _x000D_ Haemoglobin
_x000D_
_x000D_
_x000D_ 108 g/L
_x000D_
_x000D_
_x000D_ (130-180)
_x000D_
_x000D_
_x000D_
_x000D_ _x000D_ White Blood Cell Count
_x000D_
_x000D_
_x000D_ 9.6 ×109/L
_x000D_
_x000D_
_x000D_ (4-11)
_x000D_
_x000D_
_x000D_
_x000D_ _x000D_ Neutrophils
_x000D_
_x000D_
_x000D_ 2.20 ×109/L
_x000D_
_x000D_
_x000D_ (1.5-7.0)
_x000D_
_x000D_
_x000D_
_x000D_ _x000D_ Lymphocytes
_x000D_
_x000D_
_x000D_ 1.80 ×109/L
_x000D_
_x000D_
_x000D_ (1.5-4.0)
_x000D_
_x000D_
_x000D_
_x000D_ _x000D_ Eosinophils
_x000D_
_x000D_
_x000D_ 4.5 ×109/L
_x000D_
_x000D_
_x000D_ (0.04-0.4)
_x000D_
_x000D_
_x000D_
_x000D_ _x000D_ Serum Urea
_x000D_
_x000D_
_x000D_ 4.7 mmol/L
_x000D_
_x000D_
_x000D_ (2.5-7.5)
_x000D_
_x000D_
_x000D_
_x000D_ _x000D_ Serum Creatinine
_x000D_
_x000D_
_x000D_ 124 μmol/L
_x000D_
_x000D_
_x000D_ (60-110)
_x000D_
_x000D_
_x000D_
_x000D_
_x000D_
Urine Dipstick: Red Blood Cells 2+. Chest X Ray: No abnormality detected.
_x000D_
Which one of the following is the most likely diagnosis?
CorrectIncorrectHint
Churg-Strauss syndrome is a rare type of vasculitis affecting small to medium sized vessels (Capillaries, Venules, Arterioles). It is usually associated with bronchial asthma and eosinophilia. The inflammation may progress to vasa nervosum causing peripheral neuropathy like situations. Renal vessel vasculitis may cause microscopic haematuria and nephropathy. More than one system involvement predicts the involvement of ongoing vasculitis. The American College of Rheumatology (ACR) recommends six criteria for the diagnosis of Churg-Strauss syndrome. The presence of four or more criteria predicts of high sensitivity and specificity._x000D_
The criteria include:_x000D_
1. Eosinophilia of more than 10 percent in peripheral blood_x000D_
2. Bronchial Asthma (recurrent episodes)_x000D_
3. Mononeuritis multiplex or polyneuropathy_x000D_
4. Histological proof of vasculitis with extra vascular eosinophils._x000D_
5. Paranasal sinusitis_x000D_
6. Pulmonary infiltrates_x000D_
It went treatment is based on glucocorticoids, immunosuppressants (mycophenolate, azathioprine, cyclophosphamide)_x000D_
Atopic asthma does not explain the other symptoms such as peripheral neuropathy, eosinophilia, nephropathy along with microscopic haematuria._x000D_
Atypical pneumonia does not explain the condition as the patient has a normal chest X-ray._x000D_
Normal chest X-ray and absence of haemoptysis (pulmonary renal syndrome) are suggestive of unlikely Goodpasture’s syndrome. Here positive anti-GBM antibodies are found._x000D_
Though there is presence of microscopic haematuria but that doesn’t explain glomerulonephritis alone. To establish it, more data is required. -
Question 31 of 141
31. Question
Among the following which antimicrobial agent is associated with prolongation of the QT Interval?
CorrectIncorrectHint
Erythromycin is a macrolide group of antimicrobials. It is associated with prolongation of the QT interval._x000D_
Other antimicrobials associated with prolonged QT interval include quinine and levofloxacin. -
Question 32 of 141
32. Question
Among the following options, which one is most suitable to treat ventricular fibrillation or pulseless ventricular tachycardia during resuscitation?
CorrectIncorrectHint
As per the guideline, 300 mg Amiodarone is recommended among the following options as first dose during resuscitation. In refractory cases, repeat of 150 mg Amiodarone can be tried. IV Lidocaine can be given as an second option.
-
Question 33 of 141
33. Question
Among the following option find out the appropriate state for contraindication of Thrombolysis?
CorrectIncorrectHint
The common absolute contraindications to thrombolysis include:_x000D_
• Recent history of stroke or surgery (within 3 months)_x000D_
• Previous history of haemorrhagic stroke_x000D_
• Prolonged cardiopulmonary resuscitation (CPR) (more than half an hour)._x000D_
• Heavy ongoing haemorrhagic manifestations including torrential GI bleeding_x000D_
• Uncontrolled severe hypertension_x000D_
• Space occupying lesion of brain, cerebral neoplasm._x000D_
People over aged 75 years may be benefited from thrombolysis in AMI._x000D_
Progressive diabetic retinopathy may be a relative contraindication of thrombolysis. -
Question 34 of 141
34. Question
A study reveals an immediate elevation in the blood pressure following infusion of a hormone in some normal volunteers who have no pathology.
_x000D_
Which one of the following is the most likely hormone used in this study?
CorrectIncorrectHint
The most likely hormone used in this study is C. Angiotensin II. This is because angiotensin II is a potent vasoconstrictor that increases blood pressure by narrowing the blood vessels and stimulating the release of aldosterone, a hormone that causes salt and water retention.
_x000D_
The other options are less likely because they have different effects on blood pressure. Angiotensin I (option A) is a precursor of angiotensin II, and has little or no vasoconstrictor activity. Prolactin (option B) is a hormone that stimulates milk production and has no direct effect on blood pressure. Atrial natriuretic peptide (ANP) (option D) and brain natriuretic peptide (BNP) (option E) are hormones that lower blood pressure by causing vasodilation and increasing urine output. Therefore, the answer is C. Angiotensin II.
-
Question 35 of 141
35. Question
A research paper describes a new diagnostic test for myocardial infarction (MI).
_x000D_
The Board for Assessment of Clinical Research and Studies is keen to know what proportion of patients who are classified as not having had a myocardial infarction (MI) by the test will actually not have had a myocardial infarction (MI).
_x000D_
Which one of the following measurements would indicate this proportion of patients?
CorrectIncorrectHint
The proportion of patients who are classified as not having had a myocardial infarction (MI) by the test and who actually do not have myocardial infarction (MI) is called the negative predictive value (NPV) of the test. Therefore, the correct answer is B.
_x000D_
The negative predictive value (NPV) is calculated by dividing the number of true negatives (TN) by the sum of true negatives and false negatives (FN):
_x000D_
NPV=TN ÷ (FN+TN)
_x000D_
The negative predictive value (NPV) depends on the prevalence of the disease in the population. A higher prevalence means a lower negative predictive value (NPV), and vice versa.
_x000D_
The other measurements have different meanings:
_x000D_
- _x000D_
- Positive predictive value (PPV) is the proportion of patients who are classified as having myocardial infarction (MI) by the test and who actually have myocardial infarction (MI).
- Accuracy is the proportion of patients who are correctly classified by the test, either as having or not having myocardial infarction (MI).
- Specificity is the proportion of patients who do not have myocardial infarction (MI) and who are correctly classified by the test as not having myocardial infarction (MI).
- Sensitivity is the proportion of patients who have myocardial infarction (MI) and who are correctly classified by the test as having myocardial infarction (MI).
_x000D_
_x000D_
_x000D_
_x000D_
-
Question 36 of 141
36. Question
A randomised, double-blind, placebo controlled trial of a cholesterol-lowering drug in the primary prevention of coronary heart disease is held where 1000 subjects are treated with the active drug, and 1000 are given placebo. They are followed up over a five year period and 100 individuals in the placebo group and 80 individuals in the treatment group suffer from a myocardial infarction (MI).
_x000D_
Which one of the following is the annual percentage risk of myocardial infarction (MI) in the group treated with placebo?
CorrectIncorrectHint
The annual percentage risk of myocardial infarction (MI) in the group treated with placebo is the number of new myocardial infarction (MI) cases per year divided by the number of people at risk in the placebo group, expressed as a percentage. To calculate this, we need to use the following formula:
_x000D_
Annual percentage risk of myocardial infarction (MI) = (Number of new MI cases per year ÷ Number of people at risk in the placebo group) × 100%
_x000D_
Since the trial lasted for five years, and the number of myocardial infarction (MI) cases in the placebo group was evenly distributed across the years, we can simply divide the total number of myocardial infarction (MI) cases by five to get the number of new myocardial infarction (MI) cases per year. This is 100 / 5 = 20. The number of people at risk in the placebo group is the initial number of subjects, which is 1000.
_x000D_
Therefore, the annual percentage risk of myocardial infarction (MI) in the placebo group is:
_x000D_
Annual percentage risk of myocardial infarction (MI) = (20 ÷ 1000) × 100% = 2%.
-
Question 37 of 141
37. Question
A diagnosis of myocardial infarction (MI) is mostly specified by which one of the following findings?
CorrectIncorrectHint
The correct answer to the above question is option B. Evolution of Q waves on electrocardiogram (ECG). This is because Q waves are a sign of irreversible myocardial necrosis, which is the hallmark of myocardial infarction (MI). Q waves reflect the loss of electrical activity in a part of the heart that has been damaged by ischaemia. Q waves usually develop within hours to days after an myocardial infarction (MI), and persist for months or years. Q waves are considered pathological if they meet any of the following criteria:
_x000D_
- _x000D_
- More than 1 mm wide
- More than 2 mm deep
- More than 25% of the depth of the QRS complex
- Seen in leads V1-V3
_x000D_
_x000D_
_x000D_
_x000D_
_x000D_
The other options are not as specific or sensitive as Q waves for the diagnosis of myocardial infarction (MI):
_x000D_
Option A: Elevated cardiac enzymes, such as troponin and CK-MB, are markers of myocardial injury, but they can also be elevated in other conditions that cause cardiac stress, such as pulmonary embolism, renal failure, sepsis, or myocarditis. Moreover, cardiac enzymes may not rise immediately after a myocardial infarction (MI), and may return to normal within a few days, making them less useful for detecting old or healed myocardial infarctions (MIs).
_x000D_
Option C: ST elevation on electrocardiogram (ECG) is a sign of acute or ongoing myocardial infarction (MI), but it is not specific for myocardial infarction (MI). ST elevation indicates the injury or ischaemia of a part of the heart, but it can also be caused by other conditions that affect the myocardium or the pericardium, such as pericarditis, myocarditis, ventricular aneurysm, or aortic dissection. ST elevation can also vary depending on the ethnicity, age, or gender of the patient, or the presence of other cardiac abnormalities, such as left ventricular hypertrophy or bundle branch block.
_x000D_
Option D: An akinetic area of LV wall motion on 2D echocardiography is a sign of a severe or extensive myocardial infarction (MI), but it is not sensitive for myocardial infarction (MI). Akinetic means that a part of the left ventricle (LV) does not contract normally, resulting in reduced pumping function. Akinetic areas can also be caused by other conditions that affect the regional function of the myocardium, such as cardiomyopathy, valvular disease, or congenital defects. Echocardiography can also be limited by the image quality, the operator’s skill, or the patient’s anatomy.
_x000D_
Option E: History of severe chest pain is a common symptom of MI, but it is not specific for myocardial infarction (MI). Chest pain can also be caused by other conditions that affect the heart or other organs, such as angina, pulmonary embolism, aortic dissection, or gastroesophageal reflux disease. Chest pain can also vary in intensity, duration, location, and radiation depending on the individual characteristics of the patient, such as age, gender, or comorbidities. Some patients may not even experience chest pain during a myocardial infarction (MI), especially those who are elderly, diabetic, or female.
-
Question 38 of 141
38. Question
A 78-year-old gentleman presented to the Emergency Department with palpitations for the past six hours.
_x000D_
Two months previously he suffered from weakness of the left arm and problems with his speech which resolved within three hours. He was taking no medication.
_x000D_
On examination, he was stable with a pulse rate of 130 beats per minute, which was confirmed to be atrial fibrillation on electrocardiogram (ECG). He had a blood pressure of 110/80 mmHg and appeared clinically euthyroid.
_x000D_
Within one hour he reverted to sinus rhythm spontaneously.
_x000D_
2D echocardiogram was normal but a 24-hour electrocardiogram (ECG) revealed three episodes of atrial fibrillation each lasting around ten minutes.
_x000D_
Which one of the following is the most appropriate initial treatment for this gentleman?
CorrectIncorrectHint
The most appropriate initial treatment for this gentleman is E. Warfarin. This is because he has a high risk of stroke due to his age, history of transient ischaemic attack (TIA), and recurrent episodes of atrial fibrillation. Warfarin is an anticoagulant that prevents blood clots from forming in the heart and reduces the risk of stroke.
_x000D_
The other options can be ruled out because of the following reasons:
_x000D_
- _x000D_
- A. Aspirin: This option can be ruled out because aspirin is a weak antiplatelet agent that has limited efficacy in preventing stroke in atrial fibrillation. According to the 2019 European Society of Cardiology guidelines, aspirin is not recommended for stroke prevention in atrial fibrillation.
- B. Amiodarone: This option can be ruled out because amiodarone is an antiarrhythmic drug that can control the heart rate and rhythm, but it does not prevent thromboembolism. Amiodarone also has many side effects, such as thyroid dysfunction, pulmonary toxicity, and liver damage. Therefore, it is not a suitable choice for initial treatment.
- C. Digoxin: This option can be ruled out because digoxin is also an antiarrhythmic drug that can control the heart rate, but it does not prevent thromboembolism. Digoxin has a narrow therapeutic window and can cause toxicity, especially in elderly patients. Therefore, it is not a suitable choice for initial treatment.
- D. Atenolol: This option can be ruled out because atenolol is a beta-blocker that can control the heart rate, but it does not prevent thromboembolism. Beta-blockers can also cause adverse effects, such as bradycardia, hypotension, and bronchospasm. Therefore, it is not a suitable choice for initial treatment.
_x000D_
_x000D_
_x000D_
_x000D_
-
Question 39 of 141
39. Question
A 78-year-old gentleman was travelling by car. He developed sudden cardiac arrest during driving. Emergency paramedical rescued him after proper resuscitation. Victim was already incubated by them but due to severe circulatory collapse no peripheral access could be established. You were along with the resuscitation team and the paramedics asked you for drug delivery route establishment. There was no availability of central venous catheter at your rescue van. What will be the ideal route for resuscitation from the following options?
CorrectIncorrectHint
Although the endotracheal route is recognised for drug administration but the efficacy and doses of administered drugs are very uncertain and difficult to predict. During circulatory collapse state the referred route is Intraosseous Route (provided IV access could not be established due to any reason). In adult, 2 cm below the tibial tuberosity on the antero-medial side or 2 cm proximal to the medial malleolus.
-
Question 40 of 141
40. Question
A 79-years-old lady came to the surgical clinic with history of progressively increasing an ulcerative lesion over the right ankle for last 8 months. On detailed discussion she revealed that she had type ll diabetes, hypertension for last 5 years. It was controlled with appropriate medications. She also informed about an episode of Deep Vein Thrombosis (DVT) two years back at her right leg. On examination, she is afebrile with stable vitals. A superficial slough-based ulcer with a diameter of 7 cm near the medial malleolus was noted. Local temperature was normal around the ulcer area with no evidence of active inflammation of surrounding tissues.
_x000D_
Among the following options which one is the best investigation, prior application of a Compression Bandage?
CorrectIncorrectHint
From the above scenario probably, she is suffering from chronic venous ulcer. The main aetiology is venous congestion secondary to chronic venous insufficiency. The ideal way to manage venous ulcer is correction of infection, control of venous stasis by applying compression bandages, control of oedema secondary to venous congestion. So before putting compression bandage, the arterial circulation of the affected limb should be evaluated first or else it (compression bandage) may cause ischemic limb injury. So, to evaluate any arterial impairment of the limb Ankle-Brachial Pressure Index is the simplest way. _x000D_
As there is no active signs of inflammation or infection so bacteriological swab is not recommended here. _x000D_
Though she had history of DVT but current history is not suggestive of its presence. So, Venous Duplex Ultrasound Scan, venogram is unlikely to be done here. -
Question 41 of 141
41. Question
A 78-year-old gentleman presented to the Emergency Department with sporadic chest pain at rest and the pain radiates to the jaw. He does not smoke or drinks alcohol.
_x000D_
Which one of the following would most strongly suggest that the pain is due to myocardial ischaemia?
CorrectIncorrectHint
The correct answer is: B. Radiation of pain to the jaw. This option is the most specific feature that suggests that the pain is due to myocardial ischaemia. Myocardial ischaemia is a condition where the heart muscle does not receive enough oxygen due to a blockage or narrowing of the coronary arteries. When this happens, the pain can radiate to other areas of the body, such as the jaw, back, arms, neck, or stomach. This is called referred pain, and it occurs because the nerves that supply the heart and the jaw share a common pathway in the spinal cord. Therefore, radiation of pain to the jaw is relatively specific for pain of myocardial ischaemia.
_x000D_
The other options can be ruled out individually with appropriate explanations as follows:
_x000D_
- _x000D_
- Relief of pain by sublingual nitrate: This option is not specific for pain of myocardial ischaemia, as sublingual nitrate can also relieve the pain of oesophageal spasm, which is a non-cardiac cause of chest pain. Sublingual nitrate is a medication that dilates the blood vessels and improves blood flow to the heart muscle, which can reduce the symptoms of myocardial ischaemia. However, it can also relax the smooth muscle of the oesophagus, which can relieve the spasm and the pain.
- Past history of cigarette smoking: This option is not specific for pain of myocardial ischaemia, as past history of cigarette smoking is a risk factor for the development of cardiovascular disease, but it does not necessarily mean that the current pain is due to myocardial ischaemia. Cigarette smoking can damage the lining of the blood vessels, increase the blood pressure and heart rate, and promote the formation of blood clots, which can lead to coronary artery disease and myocardial ischaemia. However, other factors, such as age, gender, family history, diabetes, high cholesterol, obesity, and physical inactivity, can also contribute to the risk of cardiovascular disease.
- Associated dyspnoea: This option is not specific for pain of myocardial ischaemia, as associated dyspnoea can be due to anxiety or other cardiac or pulmonary conditions, such as heart failure, pulmonary embolism, or chronic obstructive pulmonary disease. Dyspnoea is a term for shortness of breath or difficulty breathing. It can occur in myocardial ischaemia, as the heart is unable to pump blood effectively to meet the body’s oxygen demand. However, it can also occur in anxiety, as the person may hyperventilate or breathe faster and deeper than normal, which can cause dizziness, chest pain, and palpitations.
- Coexistent claudication: This option is not specific for pain of myocardial ischaemia, as coexistent claudication suggests the presence of peripheral vascular disease, which is a narrowing of the blood vessels in the limbs, but it does not confirm the diagnosis of myocardial ischaemia. Claudication is a term for pain, cramping, or fatigue in the muscles of the legs or arms that occurs with physical activity and improves with rest. It is caused by reduced blood flow to the muscles due to peripheral vascular disease. Peripheral vascular disease may indicate a generalized atherosclerosis that affects the coronary arteries as well, but it is not a definitive sign of myocardial ischaemia.
_x000D_
_x000D_
_x000D_
_x000D_
-
Question 42 of 141
42. Question
A 74-year-old lady presents with difficulty speaking.
_x000D_
She has a longstanding history of diabetes, hypertension, hypercholesterolaemia and chronic obstructive pulmonary disease. She is currently on aspirin, simvastatin, amlodipine and hydrochlorothiazide.
_x000D_
On examination, she is awake, her blood pressure is 155/75 mmHg. Her pulse is irregularly irregular. She has 4/5 strength in the left arm and leg and 5/5 strength on the right. When asked to point to the window she does this correctly.
_x000D_
When told to raise her arms and place her hands out she is seen to have a pronator drift on the left. She is shown a pencil and is asked to identify it. She is unable to identify it by name and appears frustrated: applying much effort to speak a sentence. She is asked to use it appropriately and begins to write on a piece of paper but she makes no legible words despite being a retired content writer. When asked to repeat ‘Today is a sunny day’, she is unable to do so.
_x000D_
With which one of the following is this type of dysphasia consistent?
CorrectIncorrectHint
The correct answer is E. Broca’s aphasia. Broca’s aphasia is a type of expressive dysphasia that affects the ability to produce fluent and coherent speech. It is caused by damage to the area of the brain responsible for speech production called Broca’s area, which is located in the inferior frontal gyrus of the dominant hemisphere. People with Broca’s aphasia have difficulty naming objects, forming sentences, writing, and repeating phrases. However, they can usually understand spoken and written language, and follow commands. This lady’s history of difficulty speaking, identifying objects, writing, and repeating phrases are consistent with Broca’s aphasia. Her ability to point to the window and follow instructions also supports this diagnosis.
_x000D_
The other options are incorrect for the following reasons:
_x000D_
- _x000D_
- Transcortical motor aphasia is a type of expressive dysphasia that affects the ability to initiate and maintain speech. It is caused by damage to the areas of the brain surrounding Broca’s area, such as the supplementary motor area or the anterior cingulate gyrus. People with transcortical motor aphasia have difficulty producing spontaneous speech, but they can usually repeat phrases and write. This lady’s history of difficulty writing and repeating phrases makes transcortical motor aphasia unlikely.
- Global aphasia is a type of combined dysphasia that affects both the production and comprehension of language. It is caused by extensive damage to the language centres of the brain, such as the perisylvian cortex or the middle cerebral artery territory. People with global aphasia have severe impairments in speaking, understanding, reading, writing, and repeating language. They can only produce a few words or sounds, and they cannot follow commands or identify objects. This lady’s history of being able to understand commands and point to the window makes global aphasia unlikely.
- Transcortical sensory aphasia is a type of receptive dysphasia that affects the ability to comprehend language. It is caused by damage to the areas of the brain surrounding Wernicke’s area, such as the posterior temporal lobe or the parieto-occipital junction. People with transcortical sensory aphasia have difficulty understanding spoken and written language, but they can usually produce fluent speech, repeat phrases, and write. This lady’s history of difficulty producing speech and writing makes transcortical sensory aphasia unlikely.
- Wernicke’s aphasia is a type of receptive dysphasia that affects the ability to comprehend and produce meaningful language. It is caused by damage to the area of the brain responsible for language comprehension called Wernicke’s area, which is located in the posterior superior temporal gyrus of the dominant hemisphere. People with Wernicke’s aphasia have difficulty understanding spoken and written language, and they produce fluent but nonsensical speech, often with paraphasias or neologisms. They are usually unaware of their speech errors and have difficulty repeating phrases. This lady’s history of being aware of her speech difficulties and being able to point to the window makes Wernicke’s aphasia unlikely.
_x000D_
_x000D_
_x000D_
_x000D_
_x000D_
Therefore, based on the clinical features and the types of dysphasia, the only type of dysphasia that is consistent with this lady’s presentation is Broca’s aphasia.
-
Question 43 of 141
43. Question
A 74-year-old lady presented to the Emergency Department with acute left-sided weakness.
_x000D_
Examination revealed minimal left facial weakness, impaired elevation of the left shoulder, with relatively preserved left hand strength. There was global weakness in the left leg which appeared to be maximal in the foot.
_x000D_
Which one of the following arteries is most likely have been affected?
CorrectIncorrectHint
The correct answer to the above question is E. Anterior cerebral artery. The anterior cerebral artery (ACA) branches off the internal carotid artery and supplies the medial and superior aspects of the frontal and parietal lobes. A stroke of the anterior cerebral artery (ACA) can cause contralateral hemiparesis, but the weakness is usually more severe in the leg than the arm, and spares the face. This is the classic presentation of an anterior cerebral artery (ACA) stroke, as described in the literature. A stroke of the anterior cerebral artery (ACA) can also cause behavioural changes, such as apathy, confusion, or abulia, which is a lack of will or initiative. A stroke of the anterior cerebral artery (ACA) can also cause urinary incontinence, grasp reflex, and gait apraxia, which is a difficulty in walking.
_x000D_
The remaining options are not correct because:
_x000D_
- _x000D_
- D. Lenticulostriate artery: The lenticulostriate arteries are small branches of the middle cerebral artery that supply the basal ganglia and the internal capsule. The internal capsule contains the corticospinal tract, which is responsible for voluntary movement of the limbs and face. A stroke of the lenticulostriate artery can cause a lacunar infarct, which is a small, deep lesion in the brain. A lacunar infarct can affect the motor function of the contralateral side of the body, causing weakness or paralysis. The weakness may be more pronounced in the leg than the arm, and may spare the face. This is known as the pure motor hemiparesis syndrome. However, this syndrome is not specific to the lenticulostriate artery, and can also occur with strokes of the anterior cerebral artery (ACA), the posterior cerebral artery, or the posterior limb of the internal capsule.
- A. Middle cerebral artery: The middle cerebral artery (MCA) is the largest branch of the internal carotid artery and supplies the lateral portions of the frontal, temporal, and parietal lobes. A stroke of the middle cerebral artery (MCA) can cause a large infarct that affects the motor, sensory, language, and visual functions of the contralateral side of the body. A stroke of the middle cerebral artery (MCA) can cause contralateral hemiplegia, but the weakness is usually more severe in the face and arm than the leg. A stroke of the middle cerebral artery (MCA) can also cause aphasia, which is a language disorder that affects speech and comprehension, and hemianopia, which is a loss of vision in half of the visual field.
- B. Posterior cerebral artery: The posterior cerebral artery (PCA) is a branch of the basilar artery and supplies the occipital lobe, the cerebellum, and the brain stem. A stroke of the posterior cerebral artery (PCA) can cause a posterior circulation infarct that affects the visual, sensory, and motor functions of the contralateral side of the body. A stroke of the posterior cerebral artery (PCA) can cause contralateral hemiparesis, but the weakness is usually mild and affects the face, arm, and leg equally. A stroke of the posterior cerebral artery (PCA) can also cause visual disturbances, such as homonymous hemianopia, cortical blindness, or visual agnosia, which is an inability to recognize objects or faces. A stroke of the posterior cerebral artery (PCA) can also cause vertigo, nausea, vomiting, ataxia, and cranial nerve palsies.
- C. Posterior communicating artery: The posterior communicating artery (PCoA) is a branch of the internal carotid artery that connects to the posterior cerebral artery, forming part of the circle of Willis. A stroke of the posterior communicating artery (PCoA) can cause an anterior circulation infarct that affects the optic nerve, the oculomotor nerve, and the internal carotid artery. A stroke of the posterior communicating artery (PCoA) can cause contralateral hemiparesis, but the weakness is usually mild and affects the face, arm, and leg equally. A stroke of the posterior communicating artery (PCoA) can also cause visual loss, such as monocular blindness, or anisocoria, which is a difference in pupil size. A stroke of the posterior communicating artery (PCoA) can also cause oculomotor nerve palsy, which can cause drooping of the eyelid, double vision, and deviation of the eye.
_x000D_
_x000D_
_x000D_
_x000D_
-
Question 44 of 141
44. Question
A 74-year-old gentleman presents with generalised bradykinesia.
_x000D_
Examination reveals a tremor of the hands.
_x000D_
Which one of the following frequencies of tremor could be suspected in Parkinson’s disease?
CorrectIncorrectHint
The correct answer is B. 5 Hz. The tremor in Parkinson’s disease occurs at rest and is characterized by a frequency of 4 to 6 Hz and a medium amplitude. This means that the affected body part shakes four to six times per second. The tremor usually starts asymmetrically, affecting only one side of the body, and tends to involve the hands, lower lip, jaw, or leg.
_x000D_
The other answer options are incorrect because:
_x000D_
- _x000D_
- A. 10 Hz: This is too high for a Parkinson’s tremor, which rarely exceeds 7 Hz. A tremor of 10 Hz is more likely to be seen in essential tremor, a condition that causes action tremor (tremor that worsens with movement) rather than resting tremor.
- C. 1 Hz: This is too low for a Parkinson’s tremor, which is usually faster than 3 Hz. A tremor of 1 Hz is more likely to be seen in cerebellar tremor, a condition that causes intention tremor (tremor that worsens with target-directed movement) due to damage to the cerebellum, the part of the brain that controls coordination.
- D. 8 Hz: This is also too high for a Parkinson’s tremor, which is typically slower than 7 Hz. A tremor of 8 Hz is more likely to be seen in physiological tremor, a normal and benign tremor that occurs in healthy people due to factors such as stress, anxiety, caffeine, or fatigue.
- E. 2 Hz: This is also too low for a Parkinson’s tremor, which is usually faster than 3 Hz. A tremor of 2 Hz is more likely to be seen in dystonic tremor, a condition that causes irregular and jerky tremor due to involuntary muscle contractions.
_x000D_
_x000D_
_x000D_
_x000D_
-
Question 45 of 141
45. Question
A 74-year-old gentleman has a four-month history of progressive numbness and unsteadiness of his gait. He has a history of hypertension which is controlled with indapamide and lisinopril and diet controlled diabetes but is otherwise well.
_x000D_
On examination his blood pressure is 130/80 mmHg, pulse rate is 74 beats per minute and regular, heart sounds are normal, abdomen is soft and non-tender, and his body mass index (BMI) is 24 kg/m2. There is a mild spastic paraparesis, with brisk knee reflexes, ankle reflexes are present with reinforcement, extensor plantars, sensory loss in the legs with a sensory level at T10, impaired joint position sense in the toes, and loss of vibration sense below the iliac crests.
_x000D_
Investigations Show:
_x000D_
_x000D_ _x000D_
_x000D_ _x000D_ Haemoglobin
_x000D_
_x000D_
_x000D_ 124 g/L
_x000D_
_x000D_
_x000D_ (120-160)
_x000D_
_x000D_
_x000D_
_x000D_ _x000D_ Mean Corpuscular Volume (MCV)
_x000D_
_x000D_
_x000D_ 93 fL
_x000D_
_x000D_
_x000D_ (80-96)
_x000D_
_x000D_
_x000D_
_x000D_
_x000D_
Which one of the following is the most plausible diagnosis?
CorrectIncorrectHint
The correct answer is C. Dorsal meningioma. This is a benign tumour that arises from the meninges covering the spinal cord, which can compress the spinal cord and cause neurological deficits. This gentleman has a sensory level at T10, which indicates a thoracic myelopathy, or a spinal cord lesion in the thoracic region. A dorsal meningioma can cause a thoracic myelopathy by compressing the posterior aspect of the spinal cord, which contains the sensory tracts. This gentleman also has a progressive and chronic history, which is consistent with the slow and gradual onset of symptoms caused by a meningioma.
_x000D_
The other options are less likely for the following reasons:
_x000D_
- _x000D_
- A. Subacute combined degeneration of the cord: This is a condition caused by vitamin B12 deficiency, which affects the posterior and lateral columns of the spinal cord, resulting in sensory and motor dysfunction. This gentleman’s haemoglobin concentration and mean corpuscular volume (MCV) are normal, which makes vitamin B12 deficiency unlikely. He also does not have any history suggestive of autoimmune disease, such as pernicious anaemia, which is a common cause of vitamin B12 deficiency.
- B. Anterior spinal artery occlusion: This is a vascular event that interrupts the blood supply to the anterior two-thirds of the spinal cord, resulting in motor paralysis, loss of pain and temperature sensation, and bladder dysfunction. This gentleman’s history is progressive and chronic, whereas anterior spinal artery occlusion usually occurs acutely and rapidly, over a period of approximately 6-12 hours. This gentleman also does not have any signs of bladder dysfunction, which is a common feature of anterior spinal artery occlusion.
- D. Tabes dorsalis: This is a late manifestation of neurosyphilis, which affects the dorsal roots and columns of the spinal cord, resulting in sensory ataxia, loss of reflexes, and lancinating pains. This gentleman does not have any symptoms of sensory ataxia, such as difficulty in walking or maintaining balance. He also does not have any signs of cognitive impairment or global degeneration, which are often associated with neurosyphilis. This gentleman also does not have any history of untreated syphilis, which is a prerequisite for developing tabes dorsalis.
- E. Multiple sclerosis: This is a demyelinating disease of the central nervous system, which can cause various neurological symptoms depending on the location and extent of the lesions. This gentleman is 74 years old, which is an uncommon age for presenting with multiple sclerosis, as it typically occurs before the age of 60. He also has a primary progressive form of the disease, which is less common than the relapsing and remitting form, which presents with acute neurological deficits that resolve (fully or partially) and recur months or years later. This gentleman’s pattern of disease and age are therefore not consistent with a diagnosis of multiple sclerosis.
_x000D_
_x000D_
_x000D_
_x000D_
-
Question 46 of 141
46. Question
A 73-year-old lady presents with complaints of weight loss, lower limb weakness and dry mouth. He has been a heavy smoker for many years.
_x000D_
On examination he looks cachectic; he has proximal lower limb weakness, areflexia (reflexes normalise with repetitive muscle contraction). There is no wasting or fasciculations. Sensory examination is normal.
_x000D_
Which one of the following blood tests is the most likely to confirm the diagnosis?
CorrectIncorrectHint
The most likely diagnosis is Lambert-Eaton syndrome. It results when IgG autoantibodies blockade the voltage-gated calcium channels of peripheral cholinergic nerve territory.
_x000D_
Fifty per cent of the cases are associated with small cell lung carcinoma.
_x000D_
Proximal lower limb weakness is the most consistent neurological feature.
_x000D_
Ptosis and ophthalmoplegia are rare. Autonomic dysfunction is common (for example, dry mouth).
_x000D_
The reflexes are depressed or absent but normalise with repetitive muscle contraction.
-
Question 47 of 141
47. Question
A 73-year-old gentleman presents with acute back pain followed by weakness of dorsiflexion of his right foot.
_x000D_
Where the associated sensory loss could be expected?
CorrectIncorrectHint
The most likely option for the location of the sensory loss is C. Dorsum of foot. This is because weakness of dorsiflexion of the foot suggests an injury to the common peroneal nerve, which branches from the sciatic nerve and innervates the muscles and skin of the anterior and lateral compartments of the leg. The common peroneal nerve can be compressed or damaged by various causes, such as trauma, fracture, dislocation, or prolonged pressure. The sensory loss would affect the dorsum of the foot and the anterolateral aspect of the lower leg, corresponding to the distribution of the superficial peroneal nerve, a branch of the common peroneal nerve.
_x000D_
The other options are less likely for the following reasons:
_x000D_
- _x000D_
- A. Sole of foot: The sole of the foot is innervated by the tibial nerve, another branch of the sciatic nerve that runs posteriorly in the leg and innervates the muscles and skin of the posterior compartment. An injury to the tibial nerve would cause weakness of plantarflexion and inversion of the foot, not dorsiflexion.
- B. Anterior thigh: The anterior thigh is innervated by the femoral nerve, which originates from the lumbar plexus and innervates the muscles and skin of the anterior compartment of the thigh. An injury to the femoral nerve would cause weakness of hip flexion and knee extension, not foot dorsiflexion.
- D. Perineum: The perineum is innervated by the pudendal nerve, which originates from the sacral plexus and innervates the muscles and skin of the external genitalia and anal region. An injury to the pudendal nerve would cause urinary and faecal incontinence, sexual dysfunction, and perineal pain, not foot weakness.
- E. Posterior calf: The posterior calf is innervated by the sural nerve, which is formed by the union of branches from the tibial and common peroneal nerves and innervates the skin of the posterolateral aspect of the lower leg and the lateral aspect of the foot. An isolated injury to the sural nerve would cause sensory loss in this area, but not weakness of foot dorsiflexion, as the sural nerve is purely sensory.
_x000D_
_x000D_
_x000D_
_x000D_
-
Question 48 of 141
48. Question
A 72-year-old lady attends the Emergency Department with complaint of recurrent falls. She has a history of a stroke affecting her right arm and leg a year earlier.
_x000D_
Computed Tomography (CT) Scan confirms that there is an evident infarct in the left parietal region. Carotid Scanning shows stenosis of 80% on the left and 90% on the right.
_x000D_
Which one of the following is the best course of action that can be taken?
CorrectIncorrectHint
This question is difficult. Her carotid atherosclerosis could be attributed to a stroke which the lady has obviously had though a significant period has passed since she had her symptoms.
_x000D_
Therefore the benefit of carotid endarterectomy is less clear.
_x000D_
Generally if the carotid artery has >70% stenosis on the relevant side it should be treated (unless the stroke has been significantly disabling). At the time of her symptoms, this lady should have been investigated; it is difficult to assess what should be done now. She should be referred to a specialist as an outpatient where the risks and benefits can be weighed up as nothing is indicated urgently.
_x000D_
Management of carotid stenosis outside of the acute stroke setting is difficult and needs to be managed by a specialist, and does not have the same urgency associated with it. Ultimately, this is the point that the question makes.
-
Question 49 of 141
49. Question
A 72-year-old lady is admitted with an acute stroke.
_x000D_
Examination revealed a right Horner’s syndrome, loss of corneal reflex on the right together with loss of pinprick sensation on the right side of the face.
_x000D_
Her right gag reflex was also decreased. She had right limb ataxia with left hemi-sensory loss of pain and temperature sensation.
_x000D_
Which one of the following arterial territories has been affected?
CorrectIncorrectHint
The arterial territory that has been affected in this lady is D. Right posterior inferior cerebellar. This is because the posterior inferior cerebellar artery (PICA) supplies the lateral medulla, the inferior cerebellum, and the posterior inferior temporal lobe. The symptoms of a right posterior inferior cerebellar artery (PICA) stroke include:
_x000D_
- _x000D_
- Right Horner’s syndrome: Due to damage to the descending sympathetic tract in the lateral medulla.
- Loss of corneal reflex and pinprick sensation on the right side of the face: Due to damage to the spinal trigeminal nucleus and tract in the lateral medulla.
- Decreased right gag reflex: Due to damage to the nucleus ambiguus in the lateral medulla.
- Right limb ataxia: Due to damage to the inferior cerebellar peduncle and the cerebellum.
- Left hemi-sensory loss of pain and temperature sensation: Due to damage to the spinothalamic tract in the lateral medulla.
_x000D_
_x000D_
_x000D_
_x000D_
_x000D_
_x000D_
The other options are incorrect for the following reasons:
_x000D_
- _x000D_
- A. Left posterior inferior cerebellar: This would cause similar symptoms as the right posterior inferior cerebellar artery (PICA) stroke, but on the opposite side of the body and face.
- B. Right superior cerebellar: This would cause cerebellar signs such as ataxia, dysmetria, and nystagmus, but not the cranial nerve or sensory deficits seen in this case.
- C. Left posterior communicating: This would cause symptoms of a posterior cerebral artery (PCA) stroke, such as visual field defects, memory impairment, and sensory loss, but not the cranial nerve or cerebellar signs seen in this case.
- E. Basilar: This would cause symptoms of a brainstem stroke, such as coma, quadriplegia, cranial nerve palsies, and locked-in syndrome, but not the lateralized signs seen in this case.
_x000D_
_x000D_
_x000D_
_x000D_
-
Question 50 of 141
50. Question
A 72-year-old lady presents to the Emergency Department with sudden loss of vision in her left eye, associated with a relative afferent pupillary defect.
_x000D_
She has poorly controlled systemic hypertension and raised cholesterol.
_x000D_
Which one of the following is the most likely aetiology of her presentation?
CorrectIncorrectHint
The most likely aetiology of this lady’s presentation is A. Retinal vascular occlusion. This is a condition where the blood flow to or from the retina is blocked, causing sudden vision loss in one eye. This lady’s risk factors of hypertension and hypercholesterolemia increase the likelihood of this diagnosis.
_x000D_
The other options are less likely because:
_x000D_
- _x000D_
- B. Retinitis pigmentosa is a genetic disorder that causes progressive degeneration of the retina, leading to night blindness and peripheral vision loss. It usually affects both eyes and starts in childhood or adolescence.
- C. Cataract is a clouding of the lens of the eye that causes blurry or dim vision. It usually develops gradually and affects both eyes. It is more common in older people and those with diabetes, smoking, or sun exposure.
- D. Chronic open angle glaucoma is a condition where the pressure inside the eye is elevated, damaging the optic nerve and causing vision loss. It usually affects both eyes and progresses slowly, often without symptoms until late stages. It is more common in older people, African-Americans, and those with a family history of glaucoma.
- E. Macular degeneration is a condition where the central part of the retina (the macula) deteriorates, causing loss of central vision. It usually affects both eyes and is more common in older people, smokers, and those with a family history of macular degeneration.
_x000D_
_x000D_
_x000D_
_x000D_
-
Question 51 of 141
51. Question
A 72-year-old lady presents with one-year history of cognitive impairment, Parkinsonism, intermittent confusion and generalised myoclonus.
_x000D_
She was started on sinemet 62.5 mg three times daily. In the following three months she has started experiencing visual hallucinations.
_x000D_
Which one of the following is the most proper diagnosis?
CorrectIncorrectHint
The lady in this question presents with a one-year history of cognitive impairment, Parkinsonism, intermittent confusion, and generalized myoclonus. She was initiated on Sinemet (levodopa-carbidopa) at a dose of 62.5 mg three times daily. Over the subsequent three months, she has developed visual hallucinations.
_x000D_
Let us evaluate the possible diagnoses:
_x000D_
- _x000D_
- Alzheimer’s Disease (A):
- Alzheimer’s disease typically manifests with gradual cognitive decline, memory impairment, and disorientation. Visual hallucinations are less common in Alzheimer’s.
- Rule Out: Unlikely due to the rapid onset and presence of visual hallucinations.
- Progressive Supranuclear Palsy (B):
- Progressive supranuclear palsy (PSP) is characterized by vertical gaze palsy, axial rigidity, and early falls. Cognitive impairment is present but not the primary feature.
- Rule Out: Less likely given the prominent cognitive symptoms and hallucinations.
- Diffuse Lewy Body Disease (C):
- Diffuse Lewy body disease (DLB) shares features with both Parkinson’s disease and dementia with Lewy bodies. It presents with cognitive fluctuations, visual hallucinations, and Parkinsonism.
- Explanation: Diffuse Lewy bodies (DLB) is the most appropriate diagnosis for this lady. The combination of cognitive impairment, Parkinsonism, and visual hallucinations aligns with diffuse Lewy bodies (DLB).
- Rule Out: Not applicable.
- Multiple System Atrophy (D):
- Multiple system atrophy (MSA) is characterized by autonomic dysfunction, cerebellar signs, and Parkinsonism. Cognitive impairment is less prominent.
- Rule Out: Less likely due to the significant cognitive symptoms.
- Idiopathic Parkinson’s Disease (E):
- Parkinson’s disease primarily presents with motor symptoms (tremor, rigidity, bradykinesia) and does not typically cause significant cognitive impairment or visual hallucinations.
- Rule Out: Unlikely given the cognitive and hallucinatory features.
_x000D_
- _x000D_
_x000D_
_x000D_
_x000D_
_x000D_
- _x000D_
_x000D_
_x000D_
_x000D_
_x000D_
- _x000D_
_x000D_
_x000D_
_x000D_
_x000D_
_x000D_
- _x000D_
_x000D_
_x000D_
_x000D_
_x000D_
- _x000D_
_x000D_
_x000D_
_x000D_
_x000D_
In summary, the most proper diagnosis for this lady is Diffuse Lewy Body Disease (C), considering the combination of cognitive decline, Parkinsonism, and visual hallucinations. Other conditions can be ruled out based on the specific clinical features.
-
Question 52 of 141
52. Question
A 71-year-old lady presents with a severe headache, the worst she has ever had, affecting the back of her head and her neck.
_x000D_
On admission to the Emergency Department she is very agitated and requires opiate based pain relief for her headache. She has a history of hypertension for which she takes ramipril, amlodipine and indapamide. Her blood pressure is elevated at 180/100 mmHg and she has a tachycardia of 92 beats per minute.
_x000D_
She is severely photophobic and finds it impossible to comply with ophthalmoscopy. Neurological examination, as far as can be told, is normal.
_x000D_
Investigations Show:
_x000D_
_x000D_ _x000D_
_x000D_ _x000D_ Haemoglobin
_x000D_
_x000D_
_x000D_ 115 g/L
_x000D_
_x000D_
_x000D_ (135-177)
_x000D_
_x000D_
_x000D_
_x000D_ _x000D_ Total Leucocyte Count
_x000D_
_x000D_
_x000D_ 4.3 ×109/L
_x000D_
_x000D_
_x000D_ (4-11)
_x000D_
_x000D_
_x000D_
_x000D_ _x000D_ Platelet Count
_x000D_
_x000D_
_x000D_ 240 ×109/L
_x000D_
_x000D_
_x000D_ (150-400)
_x000D_
_x000D_
_x000D_
_x000D_ _x000D_ Serum Sodium
_x000D_
_x000D_
_x000D_ 141 mmol/L
_x000D_
_x000D_
_x000D_ (135-146)
_x000D_
_x000D_
_x000D_
_x000D_ _x000D_ Serum Potassium
_x000D_
_x000D_
_x000D_ 4.1 mmol/L
_x000D_
_x000D_
_x000D_ (3.5-5)
_x000D_
_x000D_
_x000D_
_x000D_ _x000D_ Serum Creatinine
_x000D_
_x000D_
_x000D_ 86 μmol/L
_x000D_
_x000D_
_x000D_ (79-118)
_x000D_
_x000D_
_x000D_
_x000D_ _x000D_ Computed Tomography (CT) Scan Head
_x000D_
_x000D_
_x000D_ Left parietal lobe haemorrhage
_x000D_
_x000D_
_x000D_
_x000D_
_x000D_
Which one of the following neurological findings would be mostly expected?
CorrectIncorrectHint
The correct answer is E. Acalculia. This is the inability to perform simple arithmetic, such as adding, subtracting, multiplying, or dividing numbers. Acalculia can result from damage to the dominant inferior parietal lobule. This is the part of the brain that is located in the lower back of the left hemisphere (in most people) and is important for processing numerical and spatial information. The dominant inferior parietal lobule is also involved in other cognitive functions, such as reading, writing, language comprehension, and finger recognition. Damage to this area can cause various impairments, such as acalculia, agraphia (difficulty writing), alexia (difficulty reading), finger agnosia (inability to identify fingers), and Gerstmann’s syndrome (a combination of these symptoms).
_x000D_
This lady has a left parietal lobe haemorrhage, which means that there is bleeding in the brain tissue of the left parietal lobe. This can cause pressure on the surrounding structures and disrupt the blood supply to the affected area. Her symptoms of severe headache, agitation, photophobia, and elevated blood pressure suggest that she has increased intracranial pressure due to the haemorrhage. Her neurological examination is normal, as far as can be told, which means that she does not have any obvious signs of motor or sensory deficits, such as weakness, numbness, or visual loss. However, she may have subtle cognitive impairments that are not easily detected by a routine examination, such as acalculia. Acalculia is a specific and common sign of damage to the dominant inferior parietal lobule, and it can be tested by asking the patient to perform simple calculations or to count backwards.
_x000D_
The other options are incorrect as discussed below:
_x000D_
- _x000D_
- A. Contralateral homonymous superior quadrantanopia: This is a visual field defect that affects the upper half of the opposite side of the visual field. For example, if the right lower part of the optic radiation is damaged, the patient will lose the upper left part of the visual field in both eyes. The optic radiation is a bundle of nerve fibers that connects the thalamus (a relay station in the brain) to the primary visual cortex (the part of the brain that processes visual information) in the occipital lobe (the back of the brain). The lower part of the optic radiation runs through the temporal lobe (the side of the brain), which is not affected by the patient’s left parietal lobe haemorrhage. Therefore, the patient does not have contralateral homonymous superior quadrantanopia.
- B. Dysphagia: This is difficulty swallowing, which can be caused by various conditions affecting the muscles or nerves of the throat, such as stroke, Parkinson’s disease, or head and neck cancer. Dysphagia can result from damage to the brainstem (the lower part of the brain that connects to the spinal cord), the cranial nerves (the nerves that emerge from the brainstem and control the muscles of the face, mouth, and throat), or the cortex (the outer layer of the brain that controls voluntary movements). The patient’s left parietal lobe haemorrhage does not involve any of these structures and the patient does not have any signs of dysphagia, such as coughing, choking, or drooling.
- C. Left hemiplegia: This is paralysis of the left side of the body, which can result from damage to the right hemisphere of the brain, especially the motor cortex. The motor cortex is the part of the brain that controls voluntary movements of the muscles. The motor cortex is located in the frontal lobe (the front of the brain), which is separated from the parietal lobe by the central sulcus (a groove in the brain). The motor cortex is organized in a contralateral manner, which means that the right side of the brain controls the left side of the body and vice versa. Therefore, damage to the right motor cortex can cause left hemiplegia. The patient’s left parietal lobe haemorrhage does not affect the right hemisphere or the motor cortex and the patient does not have any signs of left hemiplegia, such as weakness, spasticity, or reflex changes.
- D. Expressive dysphasia: This is a type of aphasia that affects the ability to produce speech, while receptive dysphasia affects the ability to understand speech. Both types of dysphasia can result from damage to the left hemisphere of the brain, which is dominant for language functions in most people. However, expressive dysphasia is more commonly caused by damage to the posterior inferior frontal gyrus, also known as Broca’s area. This is the part of the brain that is responsible for generating the motor commands for speech production. Broca’s area is located in the frontal lobe, which is not affected by the patient’s left parietal lobe haemorrhage. The patient does not have any evidence of expressive dysphasia, such as slow, effortful, or non-fluent speech.
_x000D_
_x000D_
_x000D_
_x000D_
-
Question 53 of 141
53. Question
A 77-year-old lady is referred to the cardiology clinic with a history of palpitations.
_x000D_
She presented to her general physician after three days of fast, irregular palpitations. Her general physician noted an irregular pulse and a subsequent 12-lead electrocardiogram (ECG) confirmed atrial fibrillation (AF). She has been referred to the cardiologist for assessment. In the cardio clinic, she is found to be in sinus rhythm. Her usual state of health is good; she lives independently with her husband and has a well-controlled hypertension.
_x000D_
She has read in a Public Health Bulletin about stroke risk associated with atrial fibrillation (AF) and is keen to know if she needs any medication to reduce her risk.
_x000D_
From the list given below, which one should be selected as the most appropriate response?
CorrectIncorrectHint
The most appropriate response for this lady is D. Warfarin, dose adjusted to International Normalised Ratio (INR).
_x000D_
Warfarin is an anticoagulant medication that prevents the formation of blood clots in the heart and reduces the risk of stroke in patients with atrial fibrillation (AF). Warfarin requires regular monitoring of the INR, which is a measure of how quickly the blood clots. The target INR range for patients with atrial fibrillation (AF) is usually 2.0 to 3.0.
_x000D_
The other responses are less suitable for this lady because:
_x000D_
- _x000D_
- A. Aspirin 75 mg once daily is a low dose of antiplatelet medication that inhibits the aggregation of platelets and prevents the formation of small clots in the arteries. However, aspirin is not as effective as warfarin in preventing stroke in patients with atrial fibrillation (AF).
- B. Aspirin 300 mg once daily is a higher dose of antiplatelet medication that has a similar mechanism as aspirin 75 mg once daily. However, aspirin 300 mg once daily is not more effective than aspirin 75 mg once daily in preventing stroke in patients with atrial fibrillation (AF), and it may increase the risk of bleeding.
- C. Clopidogrel 75 mg once daily is another antiplatelet medication that inhibits the activation of platelets and prevents the formation of small clots in the arteries. However, clopidogrel is not as effective as warfarin in preventing stroke in patients with atrial fibrillation (AF).
- E. No anticoagulation necessary is not a valid option for this lady, as she has a high risk of stroke due to her age, history of hypertension, and diagnosis of atrial fibrillation (AF). Patients with atrial fibrillation (AF) have a five-fold increased risk of stroke compared with those without atrial fibrillation (AF). Anticoagulation drugs reduce the risk of stroke by nearly two thirds in patients with atrial fibrillation (AF).
_x000D_
_x000D_
_x000D_
_x000D_
-
Question 54 of 141
54. Question
A 75-year-old lady, who was previously seen in elderly clinic three months ago with a history of palpitations, has now come up for a review. She has a history of stable coronary artery disease (CAD) and hypertension controlled on bendroflumethiazide. She remains active and lives alone independently.
_x000D_
When she was seen previously, she was sent for a 2D echocardiogram. This demonstrates good LV function, mild concentric left ventricular hypertrophy (LVH) and a dilated left atrium (LA) (AP diameter: 5.7 cm). A 24-hour electrocardiogram (ECG) has shown atrial fibrillation throughout, and a maximal rate has been 135. On questioning during this consultation, she states that she has noted a few episodes of palpitations lasting for a few hours. On the day of her review, her electrocardiogram (ECG) confirms atrial fibrillation.
_x000D_
Which one of the following is the most appropriate initial management of her arrhythmia?
CorrectIncorrectHint
Atrial fibrillation (AF) is a common and serious heart rhythm disorder that causes irregular and rapid heartbeats. It can increase the risk of stroke, heart failure, and other complications. The goals of atrial fibrillation (AF) treatment are to control the heart rate, restore the normal heart rhythm, and prevent blood clots.
_x000D_
The most appropriate initial management of the lady’s arrhythmia depends on several factors, such as the duration and severity of her symptoms, the presence of underlying heart disease, and the risk of bleeding. Based on the information given, the best option is E. Start bisoprolol.
_x000D_
Bisoprolol is a beta-blocker drug that can lower the heart rate and reduce the symptoms of atrial fibrillation (AF). It can also protect the heart from further damage and improve the quality of life. Bisoprolol is recommended as the first-line drug for rate control in patients with atrial fibrillation (AF) and stable coronary artery disease (CAD). It has fewer side effects and interactions than other drugs.
_x000D_
The other options are not as effective or suitable for the lady’s arrhythmia. Sotalol, digoxin, and amiodarone are antiarrhythmic drugs that can restore the normal heart rhythm, but they can also cause serious side effects such as low blood pressure, bradycardia, and thyroid problems. They are not recommended for patients with atrial fibrillation (AF) and coronary artery disease (CAD), as they can increase the risk of ventricular arrhythmias and cardiac death. DC cardioversion is a procedure that uses an electric shock to reset the heart rhythm, but it is not indicated for patients with atrial fibrillation (AF) and mild symptoms, as it can cause complications such as stroke, skin burns, and heart damage. It is also not effective for long-term rhythm control, as atrial fibrillation (AF) often recurs after the procedure.
-
Question 55 of 141
55. Question
A 74-years-old gentleman is admitted to the Cardiology Ward with complaints of high-grade fever, shortness of breath and difficulty in normal range of activity for last few days. He had history of hypertension and diabetes mellitus which were controlled with Lisinopril and Metformin for last two years. No other significant medical history noted so far. Though he is aged enough, still his physical activity throughout the day was remarkably well prior to this episode. On clinical workup, positive blood culture of Streptococcus bovis and features of Infective Endocarditis (IE).
_x000D_
What is the most suitable investigations to be initiated next?
CorrectIncorrectHint
Streptococcus bovis induced Infective Endocarditis (IE) has higher prevalence among the aged population (more than 60 years of age) and it is strongly associated with neoplastic diseases of colon. There are few studies by which a general concept has been taken that Streptococcus bovis bacteraemia should be evaluated for neoplastic diseases of colon immediately. So, Colonoscopy is the basic minimum investigation to be done in case of Streptococcus bovis Infective Endocarditis (IE).
-
Question 56 of 141
56. Question
A 74-year-old lady, a known case of type 2 diabetes mellitus (T2DM) for past six years, presented to the clinic with a three day history of melaena, dizziness and fatigue. She states that she had taken an analgesic five days previously for her waist pain, but cannot recall the name of the analgesic.
_x000D_
On examination, she appears pale, her pulse rate is 85 beats per minute and blood pressure is 100/60 mmHg. She has a lower midline scar from an operation for intermittent claudication five months previously.
_x000D_
Investigations Revealed:
_x000D_
_x000D_ _x000D_
_x000D_ _x000D_ Haemoglobin
_x000D_
_x000D_
_x000D_ 78 g/L
_x000D_
_x000D_
_x000D_ (130-180)
_x000D_
_x000D_
_x000D_
_x000D_ _x000D_ Stool Occult Blood
_x000D_
_x000D_
_x000D_ Strongly Positive
_x000D_
_x000D_
_x000D_
_x000D_ _x000D_ Upper Gastrointestinal Tract Endoscopy
_x000D_
_x000D_
_x000D_ Normal
_x000D_
_x000D_
_x000D_
_x000D_
_x000D_
Which one of the following is the most likely cause of her upper gastrointestinal (GI) haemorrhage?
CorrectIncorrectHint
The most likely cause of the upper gastrointestinal (GI) haemorrhage in this lady is B. Aorto-enteric fistula because aorto-enteric fistula is a rare but serious complication of aortic surgery, which she had five months ago. It is defined as an abnormal connection between the aorta and the gastrointestinal tract, usually the duodenum. It can cause massive gastrointestinal (GI) bleeding, which may be preceded by a minor “herald” bleed. This lady’s history of melaena, dizziness, fatigue, and low haemoglobin are consistent with this diagnosis.
_x000D_
Here is a brief explanation why the other options are not correct:
_x000D_
- _x000D_
- Option A is unlikely because gastric ulcers usually cause epigastric pain, nausea, and vomiting, and are associated with Helicobacter pylori infection or non-steroidal anti-inflammatory drug (NSAID) use. This lady does not have these symptoms or risk factors, and her upper gastrointestinal (GI) endoscopy was normal.
- Option C is unlikely because gastric erosions are superficial lesions of the gastric mucosa that usually cause minimal bleeding. They are often asymptomatic or cause mild epigastric discomfort. This lady’s upper gastrointestinal (GI) endoscopy was normal, which excludes this diagnosis.
- Option D is unlikely because Mallory-Weiss syndrome is a tear in the lower oesophagus or upper stomach caused by forceful vomiting or retching. This lady does not have a history of vomiting or any other predisposing factors, such as alcohol abuse or hiatal hernia. Her upper gastrointestinal (GI) endoscopy was normal, which excludes this diagnosis.
- Option E is unlikely because oesophageal varices are dilated veins in the lower oesophagus that can rupture and bleed. They are usually caused by portal hypertension due to liver cirrhosis or other chronic liver diseases. This lady does not have any signs or symptoms of liver disease, and her upper gastrointestinal (GI) endoscopy was normal, which excludes this diagnosis
_x000D_
_x000D_
_x000D_
_x000D_
-
Question 57 of 141
57. Question
A 73-year-old lady with chronic hypertension is referred by her primary care physician to the cardiology clinic for further assessment as she is noted to have persistently elevated blood pressure around 185/88 mmHg over the last four months despite regular intake of her oral antihypertensive agents. She has a body mass index (BMI) of 26 Kg/m2. She is a non-smoker, does not drink alcohol and maintain a good balanced diet.
_x000D_
There are no features to suggest a secondary cause for her hypertension.
_x000D_
Which one of the following is the most appropriate treatment for her persistently elevated blood pressure?
CorrectIncorrectHint
The correct answer is B. Calcium channel blocker. This is because calcium channel blockers (CCBs) are effective and well-tolerated in lowering systolic blood pressure (SBP) in elderly patients with isolated systolic hypertension (ISH), which is defined as systolic blood pressure (SBP) ≥160 mmHg and diastolic blood pressure (DBP) <90 mmHg. Isolated systolic hypertension (ISH) is the most common form of hypertension in the elderly and is associated with increased risk of cardiovascular events, especially stroke.
_x000D_
Several clinical trials have shown that calcium channel blockers (CCBs) reduce the incidence of stroke and other cardiovascular outcomes in elderly patients with isolated systolic hypertension (ISH). For example, the Systolic Hypertension in Europe (Syst-Eur) trial randomized 4695 patients aged 60 years or older with isolated systolic hypertension (ISH) to receive either nitrendipine, a dihydropyridine calcium channel blocker, or placebo, with the possible addition of enalapril, hydrochlorothiazide, or both. The trial was stopped early after a median follow-up of 2 years, when it was found that active treatment reduced the risk of stroke by 42%, cardiovascular death by 31%, and all-cause mortality by 26%, compared with placebo. The benefit of calcium channel blockers (CCBs) was consistent across different subgroups of patients, including those with or without diabetes.
_x000D_
The Systolic Hypertension in the Elderly Program (SHEP) trial randomized 4736 patients aged 60 years or older with isolated systolic hypertension (ISH) to receive either chlorthalidone, a thiazide diuretic, or placebo, with the possible addition of atenolol, a beta-blocker. The trial was also stopped early after a mean follow-up of 4.5 years, when it was found that active treatment reduced the risk of stroke by 36%, major cardiovascular events by 32%, and heart failure by 54%, compared with placebo. However, the benefit of diuretics was not evident in patients with diabetes.
_x000D_
Therefore, based on these trials, guidelines recommend calcium channel blockers (CCBs) or diuretics as the first-line agents for the treatment of isolated systolic hypertension (ISH) in the elderly. Calcium channel blockers (CCBs) may have some advantages over diuretics, such as better tolerability, less metabolic effects, and more consistent benefit in patients with diabetes.
_x000D_
The other options are not the best choices for the treatment of isolated systolic hypertension (ISH) in the elderly. Angiotensin-converting enzyme (ACE) inhibitors (option A) and angiotensin receptor blockers (ARBs) (option D) are not as effective as calcium channel blockers (CCBs) or diuretics in lowering systolic blood pressure and preventing stroke in this population. They may also cause adverse effects such as cough, angioedema, hyperkalaemia, and renal impairment. Beta-blockers (option C) are not recommended as the first-line agents for isolated systolic hypertension (ISH) in the elderly, unless there is a specific indication such as heart failure, angina, or arrhythmia. They may also cause side effects such as fatigue, depression, and sexual dysfunction. Alpha-blockers (option E) are not commonly used for isolated systolic hypertension (ISH) in the elderly, unless there is a coexisting condition such as benign prostatic hyperplasia or pheochromocytoma. They may also cause side effects such as dizziness, headache, and orthostatic hypotension.
-
Question 58 of 141
58. Question
A 72-years-old gentleman was complaining of productive cough, low grade fever and mild exertional breathlessness for last few days. After clinical examination his GP had assessed him as a case of lower respiratory tract infection. So appropriate medication had been started. On detailed discussion he revealed that he was on stable dose of Warfarin for the last seven months as a treatment for Atrial Fibrillation, with INR recordings between 2 – 2.5. Currently he is complaining of raised INR 5 (<1.4) with some ecchymosis various part of his body.
_x000D_
Which one of the following medications that has recently been started is likely to be responsible for this episode?
CorrectIncorrectHint
Clarithromycin, a macrolide, has impact on cytochrome P450 enzyme inhibition. Warfarin is metabolised by cytochrome P450 enzyme system. So, macrolide induced enzyme inhibition may cause warfarin toxicity or overdose which has happened in the above scenario. Ciprofloxacin and Sulphonamides may also impart the same phenomena. _x000D_
Rifampicin is cytochrome P450 enzyme inducer, so it will decrease the effect of warfarin._x000D_
Digoxin, Temazepam and Co-dydramol have no significant effect on warfarin metabolism. -
Question 59 of 141
59. Question
A 72-year-old man admitted to cardiology ward with complaints of progressive shortness of breath, palpitations and difficulty in talking for last two days. He has past medical history of recurrent heart failure, bronchial asthma, Gastro Oesophageal Reflux Disorder (GERD), atrial fibrillation. At the cardiology ward he was treated with loading dose of digoxin, IV glucocorticoid and nebulization with beta 2 agonist (L-salbutamol) and Non-Invasive Ventilation (NIV). It was found that arrythmia was persistent due to significant hypokalaemia.
_x000D_
Among the following drug which one can cause significant hypokalaemia?
CorrectIncorrectHint
Among the above-mentioned drugs, L-Salbutamol can potentiate hypokalaemia significantly. It is also augmented by the presence of glucocorticoid therapy.
_x000D_
Spironolactone and ACE inhibitors usually cause hyperkalaemia.
_x000D_
Famotidine has no significant role on potassium balance.
_x000D_
Digoxin has no effect on potassium balance in the serum. Although hypokalaemia can cause increased cardiac sensitivity to digoxin and ultimately result in digoxin toxicity.
-
Question 60 of 141
60. Question
A 72-year-old gentleman was brought to emergency department (ED) by the paramedics with history of sudden onset syncopal attack by the side of the road on today morning. As per paramedics, post event he was conscious, alert and able to make normal conversation with them. You have examined the patient clinically and found no apparent abnormality. The victim had history of hypertension for more than ten years. He was on Amlodipine and Bendroflumethiazide. On examination, he is conscious, alert and no focal neuro-deficit was noted. His blood pressure is 138/82 mmHg, pulse is 58/minute and other vital signs were within acceptable range. A 12 lead ECG was done which showed sinus bradycardia with normal axis and prolong QTc interval (560 ms). Routine blood investigations were sent to evaluate the cause of long QTc interval.
_x000D_
What will be the commonest biochemical abnormality expected here?
CorrectIncorrectHint
Long QT syndrome (LQT) mainly of two types – (i) Congenital or inherited and (ii) Acquired. Congenital LQT is mainly due to disorder of ion currents leading to prolongation of ventricular action potential and mainly associated with Timothy syndrome (LQT 8), Andersen-Tawil syndrome (LQT 7), Romano-Ward syndrome, Jervell and Lange-Nielsen syndrome.
_x000D_
Acquired LQT mostly associated with hypomagnesemia, hypocalcaemia, hypokalaemia and some drug-induced state (drug induced QT prolongation) like Amiodarone, Sotalol, Erythromycin, Haloperidol, Ziprasidone, Citalopram, Terfenadine etc. Some pathological state like hypothyroidism and bradycardia also precipitates LQT. Untreated LQT may lead to torsades de pointes and syncope or even sudden cardiac death.
-
Question 61 of 141
61. Question
A 72-year-old gentleman is referred by his general physician to the cardiology care for management of his recently diagnosed congestive heart failure (CHF).
_x000D_
The gentleman has a history of poorly controlled hypertension. Over the last four months he has been aware of deteriorating shortness of breath, fatigue, and orthopnoea. Over the last month he had been commenced on digoxin (62.5 µg daily), furosemide (80 mg daily), and amiloride (10 mg daily).
_x000D_
On examination, he has a pulse rate of 98 beats per minute and regular; a blood pressure of 140/90 mmHg. His jugular venous pressure (JVP) was not raised, he had some scattered bibasal crackles on auscultation with a displaced apex beat in the anterior axillary line, sixth intercostal space. Auscultation of the heart revealed no murmurs and he had peripheral oedema to the mid tibia.
_x000D_
Investigations Showed:
_x000D_
_x000D_ _x000D_
_x000D_ _x000D_ Serum Sodium
_x000D_
_x000D_
_x000D_ 142 mmol/L
_x000D_
_x000D_
_x000D_ (137-144)
_x000D_
_x000D_
_x000D_
_x000D_ _x000D_ Serum Potassium
_x000D_
_x000D_
_x000D_ 3.6 mmol/L
_x000D_
_x000D_
_x000D_ (3.5-4.9)
_x000D_
_x000D_
_x000D_
_x000D_ _x000D_ Serum Urea
_x000D_
_x000D_
_x000D_ 14 mmol/L
_x000D_
_x000D_
_x000D_ (2.5-7.5)
_x000D_
_x000D_
_x000D_
_x000D_ _x000D_ Serum Creatinine
_x000D_
_x000D_
_x000D_ 138 μmol/L
_x000D_
_x000D_
_x000D_ (60-110)
_x000D_
_x000D_
_x000D_
_x000D_ _x000D_ Serum Digoxin
_x000D_
_x000D_
_x000D_ 1.0 ng/mL
_x000D_
_x000D_
_x000D_ (1.0-2.0)
_x000D_
_x000D_
_x000D_
_x000D_
_x000D_
One month previously his serum urea had been 11 mmol/L and serum creatinine was 110 mol/L. An electrocardiogram (ECG) reveals left ventricular hypertrophy and chest x-ray shows cardiomegaly and calcified aorta.
_x000D_
Which one is the most appropriate next step to be incorporated in the management?
CorrectIncorrectHint
According to the National Institute for Health and Care Excellence (NICE) guidelines on chronic heart failure in adults, the most appropriate next step to be incorporated in the management of this gentleman is B. Add an angiotensin-converting enzyme (ACE) inhibitor to the current regimen.
_x000D_
This is because angiotensin-converting enzyme (ACE) inhibitors are recommended for all patients with heart failure with reduced ejection fraction (HFrEF) to reduce morbidity and mortality, unless contraindicated or not tolerated. This gentleman has signs of HFrEF, such as left ventricular hypertrophy, cardiomegaly, and peripheral oedema. Adding an angiotensin-converting enzyme (ACE) inhibitor to his current therapy could improve his symptoms and outcomes.
_x000D_
This gentleman would be classified as probably NYHA grade III – IV heart failure (dyspnoeic on minimal exertion – rest), based on the New York Heart Association (NYHA) Functional Classification. This means that he has marked to severe limitation of physical activity and experiences fatigue, palpitation, shortness of breath or chest pain with less than ordinary or any activity. This indicates a poor prognosis and a high risk of hospitalization and death.
_x000D_
Before adding beta blockers, which could also help this gentleman, he needs to start on an angiotensin-converting enzyme (ACE) inhibitor and then slowly increase the dose of beta blockers such as carvedilol, metoprolol, or bisoprolol from a low starting point. Beta blockers can lower the heart rate, blood pressure, and oxygen demand of the heart, and have been shown to reduce mortality and hospitalization in patients with HFrEF. However, they should be used with caution in patients with renal impairment, and initiated only when the patient is stable and optimally treated with other medicines. Complications can be avoided by starting with extremely low doses and increasing the dose very slowly, with close monitoring of blood pressure, heart rate, and renal function.
_x000D_
The other options are not as appropriate for this gentleman, because:
_x000D_
- _x000D_
- A. Maintaining on current therapy is not optimal, as the gentleman has worsening symptoms and signs of renal impairment.
- C. Increasing digoxin to 0.25 mg daily is not advisable, as the gentleman already has a serum digoxin level of 1.0 ng/mL, which is within the therapeutic range. Higher doses of digoxin could increase the risk of toxicity and adverse effects.
- D. Adding atenolol at a dose of 25 mg daily is not recommended, as beta-blockers should be used with caution in patients with heart failure and renal impairment. Moreover, atenolol is not a preferred beta-blocker for heart failure, as it has not been shown to reduce mortality in this condition.
- E. Increasing furosemide to 80 mg twice daily is not necessary, as the gentleman does not have a raised jugular venous pressure (JVP) or pulmonary congestion. Excessive diuresis could worsen his renal function and electrolyte balance.
_x000D_
_x000D_
_x000D_
_x000D_
-
Question 62 of 141
62. Question
A 71-year-old lady presents to the Ophthalmology Clinic with a three months history of episodic loss of vision in her left eye. On examination, her Carotid Ultrasound Scan reveals a 42% stenosis of the left internal carotid artery, as assessed by the North American Symptomatic Carotid Endarterectomy Trial (NASCET) Criteria though echocardiogram remains within normal limit.
_x000D_
Which of the following is the most appropriate initial therapy for her?
CorrectIncorrectHint
The ideal way to treat suspected TIA (Transient Ischemic Attack) as per NICE guideline is to start Aspirin as early as possible (300 mg daily dose). The risk of stroke is assessed according to ABCD scoring system. The score of 4 or above warrants the initiation of Aspirin therapy. _x000D_
The ABCD scoring system uses:_x000D_
Age ≥60 1 point_x000D_
Blood Pressure ≥140/90 mmHg at initial evaluation 1 point_x000D_
Clinical features Unilateral weakness 2 points_x000D_
Isolated speech disturbance 1 point_x000D_
Other 0 points_x000D_
Duration of symptoms ≥60 minutes 2 points_x000D_
10-59 minutes 1 point_x000D_
70) percent following a TIA. The perioperative risk of disabling stroke or death is around 3 percent. Latest UK guidelines suggest endarterectomy for symptomatic patients with greater than 70 percent stenosis (as per the North American Symptomatic Carotid Endarterectomy Trial which showed clear benefit). The endarterectomy should be performed as soon as the patient is fit for surgery, preferably within two weeks of TIA. The benefit is equivocal for symptomatic patients with (50-69) percent stenosis, but may be greater in male patients. So NICE guidelines suggest for considering endarterectomy for these patients. There is significantly little benefit for asymptomatic patients, even those with greater than 60 percent stenosis. Patients having less than 50 percent stenosis should be refrained from carotid surgery. Chance of recurrence of stenosis is about (1-20) percentage following endarterectomy, and re-operation is needed in (1-3) percentage of cases. Ipsilateral strokes may happen in 9 percent of affected persons following endarterectomy, and 26 percent of those treated with medical management alone (within 2 years). TIA with suspected non-disabling stroke patients may undergo endarterectomy prior carotid imaging (within one week of onset). In advanced disabling stroke there is little benefit of carotid endarterectomy. _x000D_
The use of modified release dipyridamole is not recommended as a first-line therapy. It can be added with aspirin or where aspirin is contraindicated or not tolerated in diagnosed TIA patients._x000D_
As per European Carotid Surgery Trialists’ Collaborative Group (ECST) patient is suitable for endarterectomy if they have symptomatic carotid stenosis of (70-99) percent. As per North American Symptomatic Carotid Endarterectomy Trial (NASCET) the above explanation has been given. -
Question 63 of 141
63. Question
A 70-years-old lady came to the Emergency Department with history of recurrent blackouts for past one week. In the cardiac monitor she was having significant bradycardia with AV dissociation. Her Electrocardiogram showed ventricular escape rhythm with complete heart block. She had one more episode of blackout during the procedure in the Emergency Department. Her Rhythm Strip was suggestive of P Wave Asystole.
_x000D_
What is the most suitable treatment here after securing airway and breathing?
CorrectIncorrectHint
Complete heart block usually needs medical attention urgently. Signs of dizziness, syncope, shock, low cardiac output state can worsen the life if not treated immediately. Ideal mode of therapy is temporary pacemaker to combat the initial crisis. But transvenous temporary pacing takes long time and skilled personnel. So, initially pharmacological therapy (Atropine, beta agonist like dopamine, epinephrine) or transcutaneous pacing is preferred to stabilise the patient. Among the options above transcutaneous pacing is the most suitable method of therapy during persistent P wave asystole and complete heart block. _x000D_
Pharmacological therapy like Atropine, Adrenaline is not effectful for persistent P wave asystole._x000D_
Adenosine can cause asystole so contraindicated here. It is used for treatment of stable supra ventricular tachycardias. -
Question 64 of 141
64. Question
A 70-years-old gentleman was admitted to the hospital for persistent case of Atrial Fibrillation. He was on anticoagulation for more than two months already. Cardiologist had reviewed in and consider for cardioversion. Post cardioversion he was maintaining with sinus rhythm in a nice way.
_x000D_
Which one of the following agents would be the most preferred as a best cardioverter drug?
CorrectIncorrectHint
Among the options Amiodarone is the best agent for pharmacologic cardioversion. It is an class 3 anti-arrhythmic agent with maximum myocyte membrane stabilizing capacity. Though it has multiple side effects but efficacy wise it is the best agent._x000D_
Other anti-arrhythmic agents are not as efficacy as Amiodarone. -
Question 65 of 141
65. Question
A 70-year-old lady with atrial fibrillation due to ischaemic heart disease is well controlled with digoxin and amiodarone. She presents to the cardiology clinic with a three-month history of weight loss and palpitations.
_x000D_
Examination reveals an irregular pulse rate of 115 beats per minute.
_x000D_
Investigations Show:
_x000D_
_x000D_ _x000D_
_x000D_ _x000D_ Serum Thyroid Stimulating Hormone (TSH)
_x000D_
_x000D_
_x000D_ <0.06 mU/L
_x000D_
_x000D_
_x000D_ (0.2-5.5)
_x000D_
_x000D_
_x000D_
_x000D_ _x000D_ Serum Total T4 (Thyroxine)
_x000D_
_x000D_
_x000D_ 144 nmol/L
_x000D_
_x000D_
_x000D_ (58-174)
_x000D_
_x000D_
_x000D_
_x000D_
_x000D_
Which one of the following would be the most useful investigation in establishing the diagnosis of thyrotoxicosis?
CorrectIncorrectHint
The correct answer is B. Serum free T4 (Thyroxine) concentration.
_x000D_
The lady in the question has atrial fibrillation due to ischaemic heart disease and is taking digoxin and amiodarone. She presents with symptoms of thyrotoxicosis, such as weight loss and palpitations. Her serum thyroid-stimulating hormone (TSH) is low, indicating suppressed pituitary function due to excess thyroid hormone.
_x000D_
Amiodarone is an antiarrhythmic drug that contains iodine and can affect the thyroid function in various ways. It can cause both hypothyroidism and hyperthyroidism, depending on the underlying thyroid status and the iodine intake of an individual.
_x000D_
Amiodarone can also interfere with the peripheral conversion of T4 (thyroxine) to T3 (triiodothyronine), which is the main pathway of thyroid hormone metabolism. T4 (thyroxine) is the predominant hormone produced by the thyroid gland, but T3 (triiodothyronine) is the more active form that binds to the nuclear receptors and regulates gene expression. Normally, about 80% of T4 (thyroxine) is converted to T3 (triiodothyronine) in the peripheral tissues, such as the liver and the kidneys, by the enzyme deiodinase.
_x000D_
Amiodarone can inhibit the activity of deiodinase, leading to reduced conversion of T4 to T3 and increased conversion of T4 to reverse T3 (rT3), which is an inactive form of T3 (triiodothyronine). This can result in decreased levels of T3 (triiodothyronine) and increased levels of rT3 in the blood.
_x000D_
Therefore, the most appropriate measure of thyroid hormone status in this lady is serum free T4 (fT4) concentration, as it reflects the amount of hormone that is available to act on the cells. The serum total T4 (tT4) concentration can be misleading, as it includes the bound fraction of T4 that is not biologically active. The serum total T4 (thyroxine) can also be affected by factors that alter the binding of T4 (thyroxine) to proteins, such as pregnancy, liver disease, or medication.
_x000D_
The serum total T3 (tT3) concentration is not a reliable indicator of thyrotoxicosis in this patient, as it can be low or normal due to the reduced conversion of T4 (thyroxine) to T3 (triiodothyronine) by amiodarone. The serum free T3 (fT3) concentration can also be misleading, as it can be normal or high due to the increased conversion of T4 (thyroxine) to T3 (triiodothyronine) by the thyroid gland in response to the low thyroid-stimulating hormone (TSH). The serum reverse T3 (rT3) concentration is not useful for diagnosing thyrotoxicosis, as it does not reflect the thyroid hormone activity in the tissues.
_x000D_
The antithyroglobulin antibody (TgAb) and antithyroid peroxidase antibody (TPOAb) are markers of autoimmune thyroid disease, such as Hashimoto’s thyroiditis or Graves’ disease. They are not helpful for diagnosing thyrotoxicosis, as they do not affect the thyroid hormone production or metabolism. They can also be negative in some cases of autoimmune thyroid disease.
_x000D_
In summary, the best investigation to establish the diagnosis of thyrotoxicosis in this lady is the serum free T4 (fT4) concentration, as it reflects the amount of thyroid hormone that is affecting the cells. The other options are either not reliable or not relevant for diagnosing thyrotoxicosis in this lady.
-
Question 66 of 141
66. Question
A 70-year-old gentleman, with a history of oesophageal carcinoma, is recovering on the surgical ward after oesophagogastrectomy.
_x000D_
The consultant cardiologist is asked to see him because he has developed worsening central chest pain, looks pale and drenched, and has dropped his blood pressure to 100/50 mmHg, with a pulse rate of 98 beats per minute. He has bibasal crackles on auscultation of the chest.
_x000D_
Investigations Show:
_x000D_
_x000D_ _x000D_
_x000D_ _x000D_ Haemoglobin
_x000D_
_x000D_
_x000D_ 110 g/L
_x000D_
_x000D_
_x000D_ (135-180)
_x000D_
_x000D_
_x000D_
_x000D_ _x000D_ Total Leucocyte Count
_x000D_
_x000D_
_x000D_ 9.2 ×109/L
_x000D_
_x000D_
_x000D_ (4-10)
_x000D_
_x000D_
_x000D_
_x000D_ _x000D_ Platelet Count
_x000D_
_x000D_
_x000D_ 190 ×109/L
_x000D_
_x000D_
_x000D_ (150-400)
_x000D_
_x000D_
_x000D_
_x000D_ _x000D_ Serum Sodium
_x000D_
_x000D_
_x000D_ 141 mmol/L
_x000D_
_x000D_
_x000D_ (134-143)
_x000D_
_x000D_
_x000D_
_x000D_ _x000D_ Serum Potassium
_x000D_
_x000D_
_x000D_ 4.2 mmol/L
_x000D_
_x000D_
_x000D_ (3.5-5)
_x000D_
_x000D_
_x000D_
_x000D_ _x000D_ Serum Creatinine
_x000D_
_x000D_
_x000D_ 148 μmol/L
_x000D_
_x000D_
_x000D_ (60-120)
_x000D_
_x000D_
_x000D_
_x000D_ _x000D_ Electrocardiogram (ECG)
_x000D_
_x000D_
_x000D_ Anterior ST elevation consistent with acute myocardial infarction (MI)
_x000D_
_x000D_
_x000D_
_x000D_
_x000D_
Which one of the following is the most appropriate management?
CorrectIncorrectHint
The most appropriate management for this gentleman is B. Angioplasty. This is a procedure that involves inserting a catheter with a balloon and a stent into the blocked coronary artery and inflating the balloon to restore blood flow. A stent is a metal mesh that keeps the artery open after the balloon is deflated and removed. Angioplasty is the preferred treatment for patients with ST-segment elevation myocardial infarction (STEMI), which is a type of heart attack caused by a complete occlusion of a coronary artery. Angioplasty can reduce the mortality, morbidity, and complications of ST-elevated myocardial infarction (STEMI), especially if performed within 12 hours of symptom onset.
_x000D_
The other options are less appropriate because:
_x000D_
- _x000D_
- A. Aspirin, clopidogrel, and low molecular weight heparin (LMWH) are antiplatelet and anticoagulant drugs that prevent the formation and growth of blood clots in the coronary arteries. They are essential adjunctive therapies for patients with ST-elevated myocardial infarction (STEMI), but they are not sufficient to achieve reperfusion of the occluded artery. They should be given as soon as possible to patients with ST-elevated myocardial infarction (STEMI), but they should not replace angioplasty as the primary revascularization strategy.
- C. Aspirin, clopidogrel, low-molecular weight heparin (LMWH), and abciximab are antiplatelet and anticoagulant drugs that prevent the formation and growth of blood clots in the coronary arteries. Abciximab is a glycoprotein IIb/IIIa inhibitor that blocks the final step of platelet aggregation. It can be used as an additional antiplatelet agent for patients with ST-elevated myocardial infarction (STEMI) undergoing angioplasty, but it is not recommended for patients who are not candidates for angioplasty or who are treated with fibrinolytic drugs. It can increase the risk of bleeding complications.
- D. Thrombolysis is a treatment that involves injecting a fibrinolytic drug into the bloodstream to dissolve the blood clot in the coronary artery. It can be an alternative to angioplasty for patients with ST-elevated myocardial infarction (STEMI) who cannot undergo angioplasty within 120 minutes of first medical contact or who have contraindications to angioplasty. However, it is less effective and more risky than angioplasty, as it has lower rates of reperfusion, higher rates of reocclusion, and higher rates of bleeding and stroke.
- E. Coronary artery bypass grafting (CABG) is a surgery that involves using a healthy blood vessel from another part of the body to bypass the blocked coronary artery and restore blood flow to the heart muscle. It can be an option for patients with ST-elevated myocardial infarction (STEMI) who have complex coronary anatomy, severe multivessel disease, or failed angioplasty. However, it is not the first-line treatment for patients with ST-elevated myocardial infarction (STEMI), as it is more invasive, costly, and associated with longer hospital stay and recovery time than angioplasty.
_x000D_
_x000D_
_x000D_
_x000D_
-
Question 67 of 141
67. Question
A 70-year-old gentleman, who has a long-standing stable heart failure treated with furosemide and enalapril, now complains of swelling in his right knee and his general physician treats him with celecoxib, a cyclooxygenase-2 (COX-2) inhibitor. Three weeks later he comes to the clinic with increasing breathlessness and ankle oedema.
_x000D_
Which one of the following effects of celecoxib is the most likely to explain his symptoms?
CorrectIncorrectHint
The most likely effect of celecoxib that explains this gentleman’s symptoms is A. Onset of fluid retention.
_x000D_
Celecoxib is a non-steroidal anti-inflammatory drug (NSAID) that belongs to the class of COX-2 inhibitors. It is used to treat pain and inflammation in various conditions, such as arthritis, acute pain, and menstrual cramps.
_x000D_
However, celecoxib can also cause some serious side effects, especially in people with heart problems or risk factors for heart disease. One of the possible side effects of celecoxib is fluid retention, which means the body holds on to more water than usual. This can lead to swelling of the ankles, feet, or hands, as well as weight gain and shortness of breath. Fluid retention can also worsen heart failure, which is a condition where the heart cannot pump enough blood to meet the body’s needs.
_x000D_
Therefore, celecoxib may not be a suitable choice for this gentleman, who already has a history of heart failure and is taking medications to treat it. Furosemide is a diuretic that helps the body get rid of excess fluid and salt, while enalapril is an angiotensin-converting enzyme (ACE) inhibitor that lowers blood pressure and improves heart function. Celecoxib may interfere with the effects of these drugs and cause more fluid to accumulate in the body, resulting in this gentleman’s symptoms of breathlessness and ankle oedema.
_x000D_
The other effects of celecoxib are less likely to explain the symptoms in this gentleman. Celecoxib may cause anaemia, decreased absorption of furosemide, reduced action of enalapril, or decreased myocardial contractility, but these effects are rare or not well-established. Moreover, they do not directly cause fluid retention, which is the main cause of this gentleman’s worsening condition.
-
Question 68 of 141
68. Question
A 70-year-old gentleman was brought to Emergency Department (ED) by the paramedics with complaints of transient circulatory collapse at road side few hours later. On detailed examination, it was revealed that he had several episodes of dizziness and fainting prior mostly at home and those were recovered spontaneously. He was a chronic smoker and quit smoking recently. Currently he is conscious alert and oriented to time, place and person. His other neurological examinations were unremarkable. ECG showed no significant arrhythmia. Echocardiography showed no significant valvular disorder except concentric LVH. His bilateral carotid doppler study shows stenosis of 84% at right side and more than 92% stenosis on the left side.
_x000D_
What is the best management for him?
CorrectIncorrectHint
Usually dizziness is associated with posterior circulation pathology (ischemia or infarction). The posterior circulation disease (stroke) mostly associated with cranial nerve palsies, nystagmus, cerebellar dysfunction and bilateral or unilateral sensory-motor deficiencies. As he had no significant residual manifestation of dizziness and fainting (above mentioned signs), it is unlikely to have posterior circulation pathology for him. Though he had significant carotid artery stenosis which can cause anterior circulation pathology (dysphasia, altered sensorium, visuo-spatial dysfunction, significant sensory-motor deficit, homonymous hemianopia, etc.) but current scenario doesn’t suggestive so. That is why the best option will be to discharge him and a regular follow-up as an out patient basis.
-
Question 69 of 141
69. Question
A 70-year-old gentleman was admitted to the hospital with severe acute laboured breathing. He denied having any chest pain but said that he had become progressively breathless over the past four months.
_x000D_
On examination, his pulse rate was 115 beats per minute and regular. His blood pressure was 100/60 mmHg and his jugular venous pressure was elevated to the angle of the jaw. His heart sounds (S1, S2) were normal. Auscultation of his chest revealed bilateral fine inspiratory crackles to the mid zones. On fundoscopy, he had haemorrhages in both fundi.
_x000D_
Investigations Revealed:
_x000D_
_x000D_ _x000D_
_x000D_ _x000D_ Haemoglobin
_x000D_
_x000D_
_x000D_ 60 g/L
_x000D_
_x000D_
_x000D_ (115-165)
_x000D_
_x000D_
_x000D_
_x000D_ _x000D_ Haematocrit
_x000D_
_x000D_
_x000D_ 0.21
_x000D_
_x000D_
_x000D_ (0.36-0.47)
_x000D_
_x000D_
_x000D_
_x000D_ _x000D_ Mean Corpuscular Volume (MCV)
_x000D_
_x000D_
_x000D_ 120 fL
_x000D_
_x000D_
_x000D_ (80-96)
_x000D_
_x000D_
_x000D_
_x000D_ _x000D_ Mean Corpuscular Haemoglobin (MCH)
_x000D_
_x000D_
_x000D_ 34.0 pg
_x000D_
_x000D_
_x000D_ (28-32)
_x000D_
_x000D_
_x000D_
_x000D_ _x000D_ White Blood Cell Count
_x000D_
_x000D_
_x000D_ 3.2 ×109/L
_x000D_
_x000D_
_x000D_ (4-11)
_x000D_
_x000D_
_x000D_
_x000D_ _x000D_ Platelet Count
_x000D_
_x000D_
_x000D_ 98 ×109/L
_x000D_
_x000D_
_x000D_ (150-400)
_x000D_
_x000D_
_x000D_
_x000D_ _x000D_ Serum Vitamin B12
_x000D_
_x000D_
_x000D_ Result Pending
_x000D_
_x000D_
_x000D_
_x000D_ _x000D_ Serum Folate
_x000D_
_x000D_
_x000D_ Result Pending
_x000D_
_x000D_
_x000D_
_x000D_
_x000D_
Electrocardiogram (ECG) showed left bundle branch block, which had been documented previously.
_x000D_
He is given intravenous furosemide 80 mg, which results in an excellent diuresis.
_x000D_
Which one of the following is the next most appropriate immediate step to be incorporated in his management?
CorrectIncorrectHint
The most appropriate immediate step to be incorporated in the management of this gentleman is E. Blood transfusion.
_x000D_
This gentleman has severe anaemia with a haemoglobin level of 60 g/L, which is well below the normal range of 115-165 g/L. He also has macrocytic anaemia, as indicated by the elevated mean corpuscular volume (MCV) and mean corpuscular haemoglobin (MCH). Macrocytic anaemia can be caused by vitamin B12 or folate deficiency, among other causes. The serum levels of these vitamins are pending, but even if they are low, they would not explain the severity of the anaemia. Therefore, other causes of macrocytic anaemia, such as bone marrow disorders, liver disease, or alcohol abuse, should be ruled out.
_x000D_
This gentleman also has signs of heart failure, such as acute laboured breathing, elevated jugular venous pressure, bilateral fine inspiratory crackles, and left bundle branch block on electrocardiogram (ECG). These signs indicate that the heart is unable to pump blood effectively to meet the body’s oxygen demand. Heart failure can be a complication of anaemia, as the heart has to work harder to compensate for the reduced oxygen-carrying capacity of the blood. Heart failure can also cause anaemia, as it can impair the kidney function and reduce the production of erythropoietin, a hormone that stimulates red blood cell formation.
_x000D_
The gentleman was given intravenous furosemide, a diuretic that reduces the fluid overload and pulmonary oedema associated with heart failure. This can improve his breathing and blood pressure, but it does not address the underlying cause of the anaemia or the heart failure. Therefore, the next most appropriate step is to transfuse blood to this gentleman, as this can rapidly increase the haemoglobin level and improve the oxygen delivery to the tissues. Blood transfusion can also reduce the cardiac workload and improve the symptoms of heart failure.
_x000D_
The other options are not as appropriate as blood transfusion for the following reasons:
_x000D_
- _x000D_
- A. Start intramuscular vitamin B12 and oral folic acid: This option may be beneficial if this gentleman has vitamin B12 or folate deficiency, but it would not provide immediate relief for the severe anaemia or the heart failure. Vitamin B12 and folate supplements take several days to weeks to correct the macrocytic anaemia. Moreover, vitamin B12 deficiency can cause neurological complications, such as peripheral neuropathy, optic atrophy, and dementia, which may be irreversible if not treated promptly. Therefore, if vitamin B12 deficiency is suspected, it should be treated with intravenous or intramuscular injections, not oral supplements, as oral absorption may be impaired in some patients.
- B. Start oral ferrous sulphate: This option is not indicated, as this gentleman does not have iron deficiency anaemia, which is characterized by microcytic anaemia (low MCV and MCH). Iron supplements would not improve the macrocytic anaemia or the heart failure, and may cause adverse effects, such as gastrointestinal irritation, constipation, or iron overload.
- C. Thrombolyse with tissue plasminogen activator (t-PA): This option is contraindicated, as the gentleman does not have any evidence of acute myocardial infarction or pulmonary embolism, which are the main indications for thrombolytic therapy. Thrombolysis with t-PA can cause serious bleeding complications, especially in patients with anaemia, low platelet count, or retinal haemorrhages, as in this case.
- D. Bone marrow aspiration: This option may be useful to diagnose the cause of the macrocytic anaemia, as it can reveal abnormalities in the bone marrow cells, such as megaloblastic changes, dysplasia, fibrosis, or infiltration by malignancy. However, it is not an immediate step, as it requires a specialized procedure, laboratory analysis, and interpretation. It also does not provide any treatment for the anaemia or the heart failure.
_x000D_
_x000D_
_x000D_
_x000D_
-
Question 70 of 141
70. Question
A 70-year-old gentleman gets admitted to the Emergency Department with severe central squeezing chest pain with profuse sweating for the past several hours. Electrocardiogram (ECG) shows evidence of an inferior myocardial infarction (MI). Coronary angiogram was done followed by a primary stenting to the proximal right coronary artery.
_x000D_
Five hours after initial presentation, he starts feeling dizzy and having difficulty breathing. His pulse rate is 36 beats per minute and regular; blood pressure is 80/55 mmHg. Heart sounds (S1, S2) are soft; Chest is clear to auscultation. Electrocardiogram (ECG) shows 2:1 atrioventricular (AV) block with broad QRS and T wave inversion inferiorly. IV atropine was administered but had no effect towards relieving his condition.
_x000D_
Which one of the following is the next most important treatment?
CorrectIncorrectHint
Based on the information provided in this question, this gentleman is suffering from a bradycardia (slow heart rate) and hypotension (low blood pressure) due to an inferior myocardial infarction (MI) with 2:1 atrioventricular (AV) block. This is a serious condition that can lead to cardiogenic shock, cardiac arrest, or death.
_x000D_
The next most important treatment for this gentleman is B. Emergency temporary transvenous pacing wire. This is a procedure that involves inserting a thin wire through a vein in the groin or neck and advancing it to the heart. The wire delivers electrical impulses to stimulate the heart and restore a normal rhythm and rate. This is a life-saving intervention that can improve the gentleman’s haemodynamics and symptoms until a permanent pacemaker can be implanted.
_x000D_
The other options are not appropriate for this gentleman:
_x000D_
- _x000D_
- A. Monitor his progress: This is not sufficient, as the gentleman is in a critical condition and needs immediate intervention to prevent further deterioration or death.
- C. Emergency insertion of a permanent pacemaker: This is not feasible, as the gentleman is unstable and requires urgent temporary pacing before undergoing a more invasive and complex procedure.
- D. IV isoprenaline: This is a drug that can increase the heart rate and blood pressure, but it can also cause arrhythmias, angina, or myocardial ischaemia. It is not recommended for patients with inferior myocardial infarction (MI) and atrioventricular (AV) block, as it can worsen the outcome.
- E. IV dopamine: This is a drug that can increase the blood pressure and cardiac output, but it can also increase the oxygen demand of the heart and cause tachycardia, arrhythmias, or ischaemia. It is not recommended for patients with inferior myocardial infarction (MI) and atrioventricular (AV) block, as it can worsen the outcome.
_x000D_
_x000D_
_x000D_
_x000D_
-
Question 71 of 141
71. Question
A 70-year-old gentleman came to endocrine clinic with complaints of increase fatigue, poor healing of his left leg ulcer for long duration. Recent investigations showed uncontrolled diabetics with signs of local infections of the ulcer site. His surgeon advised him for good diabetic control. On detailed discussion it was revealed that he was suffering from chronic heart failure. It is currently controlled with Carvedilol, Ramipril, and Torsemide.
_x000D_
Among the following hypoglycaemic agents which one should be avoided for this gentleman’s glycaemic control?
CorrectIncorrectHint
As Pioglitazone can cause retention of fluid, conditions like heart failure, hyperdynamic circulatory state etc., may worsen further with its (Pioglitazone) use. Other oral hypoglycaemic agents like Acarbose, Glipizide, Dapagliflozin, Metformin can be used with pre-existing chronic heart patient. During critical state (shock, acute heart failure, sepsis, multi organ dysfunction state etc.) ideally oral hypoglycaemic agents should be avoided or controlling blood sugar. Regular or short acting insulins are preferred there.
-
Question 72 of 141
72. Question
A 69-years-old gentleman is admitted to the Emergency Department with sudden onset central chest pain from last few hours. The chest pain radiates through to his back and continuous in intensity. He is also complaining of dyspnoea and profuse sweating for the last one hour. On examination in the Emergency Department,
_x000D_
Blood Pressure is 160/80 mmHg in his left arm and
_x000D_
150/70 mmHg in his right arm.
_x000D_
Electrocardiography (ECG) shows changes in Leads II, III, and AVF showing ST Elevation of 2 mm.
_x000D_
Which one of the following is the most likely diagnosis?
CorrectIncorrectHint
The history clinches the diagnosis of Dissecting Thoracic Aortic Aneurysm. As per the ECG, the inferior wall infarction suggestive of dissection of aneurysm at the level of right coronary artery at its ascending aortic ostium. As per the classification (DeBakey and Stanford) ascending aortic dissection should undergo surgical correction mostly. Prior Surgery short acting Beta Blocker (ESMOLOL) to be initiated to control the hypertension._x000D_
Inferior wall STEMI is close differential but thrombolysis will invite immediate death of the patient with aortic dissection. _x000D_
Coarctation Of the Aorta may have similar clinical presentation but it’s a chronic state with lesser chance of circulatory collapse._x000D_
Perforated Duodenal Ulcer and Pancreatitis may have some of the common clinical presentation but ECG finding and difference of blood pressure and upper limbs will not be same here. -
Question 73 of 141
73. Question
A 68-years-old gentleman with past history of chronic heart failure and Dilated Cardiomyopathy (DCM) came to you with complain of persistent on and off breathlessness (on exertion), palpitation and progressive weakness. On examination,
_x000D_
Pulse Rate is 89/Minute and Regular
_x000D_
Blood Pressure: 150/90 mmHg
_x000D_
Heart Sounds (S1, S2): Normal
_x000D_
Chest auscultation normal vesicular breath sound bilaterally.
_x000D_
Currently he is on Lisinopril 30 mg once daily and Furosemide 80 mg once daily.
_x000D_
Choose the best treatment option from the following list.
CorrectIncorrectHint
In chronic heart failure Beat Blockers have mortality and improved quality of life benefits. Once patient is relieved from acute decompensation of heart failure, it should be incorporated as early as possible. Various studies (COPERNICUS, CIBIS, MERIT, etc) have been suggested so. In acute decompensation Beta Blocker should be avoided._x000D_
Digoxin has no significant mortality benefits though it may reduce the recurrent hospitalisation and improves quality of life (DIG study). _x000D_
Amiodarone has insignificant role in chronic heart failure without arrhythmias._x000D_
Spironolactone has significant role in improving symptoms and outcome in severe (class 3-4) chronic heart failure (RALES trials)._x000D_
Adding Valsartan along with Lisinopril has no significant improvement (VALHEFT). -
Question 74 of 141
74. Question
A 68-years-old gentleman came to his general physician with complaints of headache and neck discomfort for last few weeks intermittently. He is known diabetic (type ll DM) for last eight years and on regular follow up to the Endocrine Clinic. On examination,
_x000D_
His blood pressure is recorded as 185/90 mmHg.
_x000D_
Pulse 82/minute, Regular
_x000D_
Other vitals findings are within normal limits. His Electrocardiogram (ECG) shows Left Ventricular Hypertrophy (LVH). His fasting blood sugar is within normal limit, other metabolic panels are within normal limits except the serum creatinine value is elevated at 150 µmol/L and Urine analysis shows presence of microalbumin.
_x000D_
Among of the following drugs which is the best therapy for controlling his Hypertension?
CorrectIncorrectHint
As per NICE and British Hypertension Society (BHS) guidelines, the best way to treat hypertension with diabetic nephropathy and Left Ventricular Hypertrophy (LVH) in elderly population is with Angiotensin Converting Enzyme Inhibitor (ACEI) or Angiotensin ll Receptor Blocker (ARB).
-
Question 75 of 141
75. Question
A 68-year-old male admitted to hospital with complaints of chest pain and profuse sweating for last few hours. He is obese, hypertensive and newly detected diabetic. He was diagnosed as anterior wall AMI which was managed at coronary care unit after thrombolysis successfully. Subsequently he was shifted to general ward from coronary care unit after stabilizing from the disease process. At general ward he developed agitations, confusions and occasional disorientation though he followed the commands in a nice way. On query it was found that he was a regular alcoholic with intake of more than 85 units of alcohol per week. You were on call at the general ward and nurses have asked you to review the patient immediately.
_x000D_
What will be the best management for him to tackle this condition?
CorrectIncorrectHint
Probably he is suffering from alcohol withdrawal symptoms. It is mostly associated with restlessness, anxiety, confusional state or precipitations of seizures. So, the ideal therapy would be incorporations of benzodiazepines. In acute state IV benzodiazepines are preferred and in chronic state chlordiazepoxide (Oral) is preferred. If alcohol withdrawal is not been managed properly, seizures and other complications can increase the mortality in significant numbers.
_x000D_
Thiamine is deficient in critically ill chronic alcoholics and may precipitate Wernicke’s encephalopathy in acute state. So, thiamine supplementations are needed for them. If the affected person is not being replenished with thiamine in time, more irreversible neuronal damage can cause Korsakoff’s psychosis. But in the scenario IV diazepam is more suitable than thiamine as alcohol withdrawal seizures is more life threatening than Wernicke’s encephalopathy.
_x000D_
As the patient is following all the commands and there were no signs of focal neuro deficits, so intracerebral bleeding secondary to thrombolysis is unlikely. So, CT scan brain will not be an immediate choice here.
_x000D_
Psychiatric evaluation is needed for long term evaluation of alcoholism or else in acute state where the alcohol withdrawal symptoms are not being controlled with first line of emergency therapy. So, it is not the better choice as immediate option.
_x000D_
Haloperidol, a typical antipsychotic can cause hypotension, prolong QT interval and potentiate ventricular arrythmias, so in post MI state it is not recommended.
-
Question 76 of 141
76. Question
A 68-year-old lady admitted to coronary care unit with complaints of chest pains, sweating and severe shortness of breath for last three hours. Her initial ECG showed ST segment elevated MI (anterior wall). She was thrombolysed and other standard anti ischemic therapy was initiated. Currently she is on Bisoprolol, Aspirin, Atorvastatin, Ramipril. The thrombolysis was done very successfully as her recent ECG shows no ST segment elevation and signs of reperfusion. Currently she is complaining of bilateral lower limbs pain and discolouration over the both feet. On examination diffuse petechial rashes are noted on both feet. The neurovascular examinations are within normal limits of both lower limbs.
_x000D_
Her blood parameters are as follows:
_x000D_
_x000D_ _x000D_
_x000D_ _x000D_ Haemoglobin
_x000D_
_x000D_
_x000D_ 135 g/L
_x000D_
_x000D_
_x000D_ (120-160)
_x000D_
_x000D_
_x000D_
_x000D_ _x000D_ Platelet Count
_x000D_
_x000D_
_x000D_ 148 ×109/L
_x000D_
_x000D_
_x000D_ (150-400)
_x000D_
_x000D_
_x000D_
_x000D_ _x000D_ Total Leucocyte Count
_x000D_
_x000D_
_x000D_ 11.8 ×109/L
_x000D_
_x000D_
_x000D_ (4-11)
_x000D_
_x000D_
_x000D_
_x000D_ _x000D_ Neutrophils
_x000D_
_x000D_
_x000D_ 6.0 ×109/L
_x000D_
_x000D_
_x000D_ (1.5-7)
_x000D_
_x000D_
_x000D_
_x000D_ _x000D_ Lymphocytes
_x000D_
_x000D_
_x000D_ 3.2 ×109/L
_x000D_
_x000D_
_x000D_ (1.5-4)
_x000D_
_x000D_
_x000D_
_x000D_ _x000D_ Eosinophils
_x000D_
_x000D_
_x000D_ 1.8 ×109/L
_x000D_
_x000D_
_x000D_ (0.04-0.4)
_x000D_
_x000D_
_x000D_
_x000D_ _x000D_ IgE Antibody
_x000D_
_x000D_
_x000D_ 4 kU/L
_x000D_
_x000D_
_x000D_ (<2)
_x000D_
_x000D_
_x000D_
_x000D_
_x000D_
What is the most suitable diagnosis form the above mentioned scenario?
CorrectIncorrectHint
As she is suffering from acute MI and developed petechial rashes (post thrombolysis state) over the feet, most suggestive of cholesterol embolization syndrome. Cholesterol emboli are micro particles and can cause partial obstructions of large or medium sized arteries (present in lower limbs). That’s why her lower limbs have features of partial obstruction and intact neurovascular functions. In cholesterol induced arteriopathy eosinophilia is quite common.
-
Question 77 of 141
77. Question
A 68-year-old gentleman, a known case of longstanding Hypertension, has recurrent episodes of loss of consciousness and has now presented to the Emergency Department for an urgent review. On assessment, he is found to have runs of Non-sustained Ventricular Tachycardia during Telemetry.
_x000D_
Laboratory Reports show a Serum Magnesium of 0.38 mmol/L (0.75-1.05).
_x000D_
This biochemical abnormality can be attributed to which one of the following?
CorrectIncorrectHint
Severe hypomagnesemia, hypokalaemia, hyponatraemia and hypocalcaemia are the side effects of prolong diuretic therapy. _x000D_
Hyperparathyroidism is not associated with hypomagnesemia. _x000D_
Chronic renal failure may cause hypermagnesemia._x000D_
Hypophosphatemia is associated with hypomagnesemia -
Question 78 of 141
78. Question
A 68-year-old gentleman with refractory hypertension was referred by his general physician to the cardiology clinic for further review and treatment.
_x000D_
He had a longstanding hypertension which had been well controlled over many years by multiple oral antihypertensive agents but recently he was found to have a blood pressure of 195/115 mmHg which proved resistant to any additional treatment.
_x000D_
He was generally asymptomatic and complied with medication. Investigations showed normal serum urea and serum electrolytes (U&Es).
_x000D_
Which one of the following is the most likely cause of this resistant hypertension?
CorrectIncorrectHint
Renovascular disease is the most likely cause of resistant hypertension in this gentleman, who has a history of longstanding hypertension that has worsened despite multiple antihypertensive agents and normal electrolytes. Renovascular disease refers to the narrowing or occlusion of the renal arteries, which supply blood to the kidneys. This reduces the renal perfusion and activates the renin-angiotensin-aldosterone system (RAAS), a hormonal pathway that regulates blood pressure and fluid balance. The renin-angiotensin-aldosterone system (RAAS) increases the production of angiotensin II, a potent vasoconstrictor, and aldosterone, a hormone that promotes sodium and water retention. These effects lead to increased blood pressure and volume overload.
_x000D_
Renovascular disease is often asymptomatic in the early stages, but can cause severe and refractory hypertension, renal dysfunction, hypokalemia, and flash pulmonary edema in the later stages. The most common cause of renovascular disease is atherosclerosis, which affects the proximal segments of the renal arteries. Other causes include fibromuscular dysplasia, which affects the distal segments of the renal arteries, and Takayasu’s arteritis, which affects the aorta and its branches. The risk factors for renovascular disease include age, smoking, diabetes, dyslipidaemia, and peripheral vascular disease.
_x000D_
The diagnosis of renovascular disease is based on clinical suspicion, laboratory tests, and imaging studies. Clinical suspicion is raised by the presence of resistant hypertension, abdominal bruit, unexplained renal failure, or recurrent flash pulmonary oedema. Laboratory tests may show elevated serum creatinine, low serum potassium, and high plasma renin activity. Imaging studies, such as duplex ultrasonography, computed tomography angiography (CTA), magnetic resonance angiography (MRA), or invasive angiography, can confirm the presence and severity of renal artery stenosis.
_x000D_
The management of renovascular disease involves optimizing the vascular risk factors, controlling the blood pressure, preserving the renal function, and preventing the cardiovascular complications. The mainstay of medical therapy is the use of angiotensin-converting enzyme (ACE) inhibitors or angiotensin II receptor blockers (ARBs), which block the effects of angiotensin II and reduce the blood pressure and proteinuria. However, these drugs should be used with caution, as they may worsen the renal function or cause hyperkalaemia in some patients. Other antihypertensive drugs, such as calcium channel blockers, beta-blockers, and diuretics, may be added as needed. Vascular intervention, such as percutaneous transluminal renal angioplasty (PTRA) with or without stenting, may be considered in selected cases, such as bilateral or solitary kidney stenosis, failed medical therapy, or intolerable side effects. However, the evidence for the benefit of vascular intervention over medical therapy is limited and controversial, and the decision should be individualized based on the patient’s characteristics and preferences.
_x000D_
The other options given here are less likely because:
_x000D_
- _x000D_
- A. Phaeochromocytoma is a rare tumour of the adrenal medulla, which secretes catecholamines, such as epinephrine and norepinephrine. It can cause paroxysmal or sustained hypertension, headache, sweating, palpitations, and anxiety. However, it is unlikely in this gentleman, who is asymptomatic and has normal serum urea and electrolytes.
- B. Chronic pyelonephritis is a chronic infection of the kidneys, which can cause scarring, inflammation, and fibrosis of the renal parenchyma. It can lead to hypertension, proteinuria, and renal failure. However, it is unlikely in this gentleman, who has normal serum urea and electrolytes and no signs of infection or pyuria.
- D. Conn’s syndrome (primary hyperaldosteronism) is a condition in which the adrenal glands produce excess aldosterone, leading to hypertension, hypokalaemia, and metabolic alkalosis. It can be caused by an adrenal adenoma, adrenal hyperplasia, or exogenous aldosterone. However, it is unlikely in this gentleman, who has normal electrolytes and no signs of adrenal mass or hyperplasia.
- E. Polycystic kidney disease is a genetic disorder that causes multiple cysts to form in the kidneys, which can impair the renal function and increase the blood pressure. However, it is unlikely in this gentleman, who has normal serum urea and electrolytes and no family history of the disease.
_x000D_
_x000D_
_x000D_
_x000D_
-
Question 79 of 141
79. Question
A 71-year-old gentleman came to cardiology clinic with complaints of palpitations and chest discomfort for last few days. On query it was revealed that he had past history of anterior wall MI and ventricular tachycardia (VT) secondary to it. He was on Amiodarone 200 mg twice daily for last three months. On examination he is Tachycardic, Tachypnoeic (respiratory rate 38/min), Blood pressure is 138/88 mmHg with some medications. His ECG showed prolonged QTc interval (580 ms)
_x000D_
What is the best method to treat this condition?
CorrectIncorrectHint
Amiodarone is class lll anti-arrhythmic agent that prolongs the phase 3 of cardiac action potential. It is a blocker of voltage gated potassium and calcium channels. Widely used in VT, VF, wide complex tachycardia, narrow complex tachycardia. It can cause prolongation of QTc interval. So, drug induced prolong QTc should be treated by stopping the offending drug and arrythmia can be controlled by adding other new drugs. Flecainide is contraindicated for QT prolong state.
-
Question 80 of 141
80. Question
A 68-year-old gentleman with a history of myocardial infarction (MI) five years earlier was admitted to the Emergency Department with progressive shortness of breath and decreasing exercise tolerance. He has smoked 50 cigarettes per day for the last 40 years.
_x000D_
His current medications are lansoprazole, aspirin and lisinopril.
_x000D_
Which one of the following laboratory tests would help to identify the reason for his symptoms?
CorrectIncorrectHint
The gentleman shows signs of heart failure that may be due to ischaemic heart disease, which affects the blood supply to the heart. Another possible diagnosis is chronic obstructive pulmonary disease (COPD), which affects the lungs and breathing, especially in heavy smokers like him.
_x000D_
Heart failure is a condition where the heart cannot pump enough blood to meet the body’s needs. It causes symptoms such as breathlessness, tiredness and swelling in the ankles. It can be diagnosed by looking for signs such as crackles in the lungs, fluid in the chest cavity, high pressure in the neck veins and swelling in the legs. It can also be confirmed by doing an echocardiogram, which shows the heart’s structure and function, and measuring the level of natriuretic peptides, which are hormones that help regulate fluid balance and blood pressure.
_x000D_
The heart may be enlarged, have an extra sound or have abnormal sounds in heart failure. The European Society of Cardiology guidelines say that there should be symptoms, signs and objective evidence before making a diagnosis of heart failure. It is also important to find out the cause of heart failure.
_x000D_
Brain natriuretic peptide (BNP) is a hormone that is produced by the heart chambers and reflects the pressure in the left ventricle (LV), the severity of breathlessness, and the state of hormonal regulation. It goes up significantly in left ventricular dysfunction, and it matches the severity of symptoms (and goes down after successful treatment). It is also high in right or left heart failure, whether due to problems with squeezing or filling of the heart. Therefore, it is a useful marker for diagnosing heart failure in patients who have breathlessness from various causes. It may also be used to predict the outcome, guide the treatment or screen for heart failure, but this is still being studied. However, it should be noted that it can also be high in other conditions, such as age, kidney failure, obesity and other diseases, so it should be interpreted in the context of the clinical situation.
_x000D_
In practice, since this gentleman has had a previous heart attack (MI), he should have a specialist assessment and a Doppler echocardiogram, which measures the blood flow in the heart, within two weeks, according to the current National Institute for Health and Care Excellence (NICE) guidelines. But if this is not possible, brain natriuretic peptide (BNP) is the next best option. Brain natriuretic peptide (BNP) is the first choice for patients who have not had a previous heart attack (MI). If the level is more than 400 pg/ml, they should have an urgent specialist assessment and echocardiogram; if the level is between 100 and 400 pg/ml, they should have further tests within six weeks.
_x000D_
Other tests, such as complete blood count (CBC), serum urea and serum electrolytes (U&Es) and creatinine, serum C-reactive protein (CRP) and clotting screen, are not specific and are unlikely to lead to a diagnosis in this case.
-
Question 81 of 141
81. Question
A 68-year-old gentleman presents to the cardiology clinic with reduced exercise tolerance. He has a past medical history of an aortic valve replacement.
_x000D_
On examination, there is an ejection systolic murmur heard loudest in the aortic area, along with a high pitched early diastolic sound. The jugular venous pressure (JVP) rises on inspiration, and is associated with a rapid descent.
_x000D_
Which one of the following is the most likely cause?
CorrectIncorrectHint
The most likely cause of this gentleman’s presentation is C. Constrictive pericarditis. This is a condition where the pericardium, the sac that surrounds the heart, becomes thickened and stiff, limiting the filling of the heart chambers. Constrictive pericarditis can result from various causes, such as infections, surgery, radiation, or autoimmune diseases.
_x000D_
Some of the clinical features that suggest constrictive pericarditis are:
_x000D_
- _x000D_
- Reduced exercise tolerance due to impaired cardiac output.
- Ejection systolic murmur due to increased flow across the aortic valve or coexisting aortic stenosis.
- High-pitched early diastolic sound, also called a pericardial knock, due to the sudden cessation of ventricular filling by the rigid pericardium.
- Jugular venous pressure (JVP) rise on inspiration, also known as Kussmaul’s sign, due to the inability of the right ventricle to accommodate the increased venous return.
- Rapid “y” descent in the jugular venous pressure (JVP) waveform due to rapid emptying of the right atrium into the right ventricle.
_x000D_
_x000D_
_x000D_
_x000D_
_x000D_
_x000D_
The other options are less likely because:
_x000D_
- _x000D_
- Aortic stenosis would cause a harsh, crescendo-decrescendo systolic murmur that radiates to the carotids, and may be associated with a delayed or diminished carotid pulse.
- Superior vena cava obstruction would cause distension of the jugular veins, but not a pulsatile jugular venous pressure (JVP) or a pericardial knock. It would also cause facial swelling, dilated chest veins, and upper limb oedema.
- Bacterial endocarditis would cause a new or changing murmur, fever, and other signs of infection or embolization, such as splinter haemorrhages, Osler’s nodes, Janeway lesions, or Roth spots.
- Aortic regurgitation would cause a decrescendo diastolic murmur along the left sternal border, and may be associated with a wide pulse pressure, a bounding pulse, and a displaced apical impulse.
_x000D_
_x000D_
_x000D_
_x000D_
-
Question 82 of 141
82. Question
A 68-year-old gentleman is brought to the Emergency Department with a left femoral neck fracture following a mechanical fall at his home and is admitted for further management.
_x000D_
X-ray of the pelvis shows several prominent calcified vessels.
_x000D_
Which one of the following is the most appropriate next step in management of this finding?
CorrectIncorrectHint
The calcified vessels in the pelvis are most likely phleboliths, which are tiny masses of calcium that form inside veins. They are usually harmless and do not cause any symptoms. They are common in older adults and may be associated with varicose veins or venous malformations.
_x000D_
The most appropriate next step in the management of this finding is A. Ignore it. Phleboliths do not require any specific treatment and do not pose any risk of complications. They can be easily distinguished from kidney or urinary stones by their location and appearance on imaging. Therefore, there is no need to anticoagulate, start a nitrate infusion, order a pulmonary scan, or request a serum troponin test for this gentleman.
-
Question 83 of 141
83. Question
A 68-year-old gentleman in the Intensive Care Unit has received 2000 ml of colloid in four hours. He is receiving an escalating noradrenaline infusion (currently at 0.76 mcg/kg/min), and has a blood pressure of 90/60 mmHg; pulse rate of 92 beats per minute; central venous pressure of 10 mmHg; and capillary refill time of 2 – 3 seconds. His plasma lactate concentration is 2.9 mmol/L (<1.5).
_x000D_
Which one of the following is an appropriate method of measuring adequate intravascular filling?
CorrectIncorrectHint
The correct answer is D. PiCCO (pulse contour cardiac output).
_x000D_
PiCCO (pulse contour cardiac output) is a minimally invasive method of measuring intravascular filling and cardiac output using a femoral or axillary arterial catheter and a central venous catheter. PiCCO (pulse contour cardiac output) can provide information about the mean systemic filling pressure (Pmsf), which is the pressure in the cardiovascular system when there is no blood flow and is a quantitative index of intravascular volume. PiCCO (pulse contour cardiac output) can also measure the global end-diastolic volume (GEDV), which is the volume of blood in the four cardiac chambers at the end of diastole and is a marker of cardiac preload. PiCCO (pulse contour cardiac output) can help guide fluid therapy and vasoactive medications in critically ill patients with haemodynamic instability.
_x000D_
The other methods of measuring intravascular filling are less appropriate for the gentleman. Pulmonary artery flotation catheter (PAFC) is an invasive method that requires insertion of a catheter into the pulmonary artery and can measure the pulmonary capillary wedge pressure (PCWP) as an estimate of left atrial pressure and left ventricular filling. However, pulmonary artery flotation catheter (PAFC) is associated with risks of complications, such as arrhythmias, pulmonary artery rupture, infection, and thrombosis. Moreover, pulmonary capillary wedge pressure (PCWP) may not reflect the true intravascular volume status in patients with pulmonary hypertension, mitral valve disease, or positive pressure ventilation.
_x000D_
LiDCO (lithium dilution cardiac output) is a minimally invasive method that uses a lithium indicator dilution technique and pulse contour analysis to measure cardiac output and intravascular volume. However, LiDCO (lithium dilution cardiac output) may not be accurate in patients with low cardiac output, peripheral vascular disease, or arterial waveform distortion. LiDCO (lithium dilution cardiac output) also requires frequent calibration and may be affected by changes in vascular tone and compliance.
_x000D_
Transoesophageal echocardiography (TOE) is a non-invasive method that uses ultrasound to visualize the heart and its structures and assess cardiac function and filling. However, transoesophageal echocardiography (TOE) may not be feasible in patients who are intubated, have oesophageal pathology, or are unable to tolerate the procedure. Transoesophageal echocardiography (TOE) also requires expertise and training to perform and interpret.
_x000D_
Oesophageal Doppler monitoring is a minimally invasive method that uses Doppler ultrasound to measure the blood flow velocity in the descending aorta and estimate the stroke volume and cardiac output. However, oesophageal Doppler monitoring may not be reliable in patients with aortic valve disease, aortic aneurysm, or aortic dissection. Oesophageal Doppler monitoring also requires frequent calibration and may be affected by changes in vascular resistance and compliance.
-
Question 84 of 141
84. Question
A 68-year-old Afro-Caribbean gentleman, a known case of hypertension, presents to the cardiology clinic with ankle oedema after taking an oral antihypertensive prescribed by his general physician. On examination, he is found to have a blood pressure of 190/110 mmHg.
_x000D_
Which one of the following would be the preferred drug for this gentleman?
CorrectIncorrectHint
Bendroflumethiazide (option A) is the correct answer because bendroflumethiazide is a thiazide-like diuretic that lowers blood pressure by increasing the excretion of water and salt from the body. According to the National Institute for Health and Care Excellence (NICE) guidelines, diuretics are the first-line antihypertensive drug treatment for people of African or Caribbean family origin, if a calcium channel blocker is not suitable, for example because of oedema. This is because diuretics have been shown to be more effective in lowering blood pressure in this population than other classes of drugs, such as angiotensin converting enzyme inhibitors (ACEIs), angiotensin receptor blockers (ARBs), or calcium channel blockers (CCBs).
_x000D_
The other incorrect answer options can be ruled out as follows:
_x000D_
- _x000D_
- B. Atenolol: This is incorrect because atenolol is a beta blocker that lowers blood pressure by slowing down the heart rate and reducing the force of contraction. However, beta blockers are not recommended as the first-line treatment for hypertension, as they have been shown to have less cardiovascular benefit and more adverse effects, such as fatigue, sexual dysfunction, and depression, than other classes of drugs. Beta blockers are preferred for post-myocardial infarction and ischaemic heart disease, but not for hypertension alone.
- C. Amlodipine: This is incorrect because amlodipine is a calcium channel blocker (CCB) that lowers blood pressure by relaxing the blood vessels and reducing the workload of the heart. However, calcium channel blockers (CCBs) are not suitable for this gentleman, as he has ankle oedema, which is a common side effect of calcium channel blockers (CCBs). Calcium channel blockers (CCBs) are the first-line antihypertensive drug treatment for people of African or Caribbean family origin, but only if they do not have oedema.
- D. Verapamil: This is incorrect because verapamil is a calcium channel blocker (CCB) that lowers blood pressure by relaxing the blood vessels and reducing the workload of the heart. However, verapamil is not suitable for this gentleman, as he has ankle oedema, which is a common side effect of calcium channel blockers (CCBs). Moreover, verapamil is not recommended as the first-line treatment for hypertension, as it has been shown to have less cardiovascular benefit and more adverse effects, such as constipation, bradycardia, and heart block, than other calcium channel blockers (CCBs), such as amlodipine or felodipine.
- E. Ramipril: This is incorrect because ramipril is an angiotensin-converting enzyme (ACE) inhibitor that lowers blood pressure by blocking the production of angiotensin II, a hormone that causes vasoconstriction and sodium retention. However, angiotensin-converting enzyme (ACE) inhibitors are not recommended as the first-line treatment for people of African or Caribbean family origin, as they have been shown to be less effective and more likely to cause adverse effects, such as cough, angioedema, and hyperkalaemia, than calcium channel blockers (CCBs) or diuretics. Angiotensin-converting enzyme (ACE) inhibitors are preferred for those patients with heart failure or diabetic nephropathy, but not for hypertension alone.
_x000D_
_x000D_
_x000D_
_x000D_
-
Question 85 of 141
85. Question
A 68-year-old gentleman came to cardiology clinic with complaints of chest discomfort, fatigue and shortness of breath for last few months. The symptoms are progressively increasing in nature. On discussion, it was revealed that he is on Amlodipine and Lisinopril for his hypertension for last eight years. On examination, his BP is 162/98 mmHg, Pulse is 110/minute and Pulsus alternans were found.
_x000D_
Among the following conditions where will you get pulsus alternans?
CorrectIncorrectHint
Pulsus alternans is found in severe left ventricular systolic dysfunction. It manifests due to regular alternation of myocardial contracting force secondary to severe LV dysfunction.
_x000D_
-
Question 86 of 141
86. Question
A 68-year-old gentleman came to his General Physician with complaints of progressively increasing shortness of breath and high-grade fever along with severe generalised weakness for last 3 days. On detailed discussion he revealed of significant weight loss for last 2 months. The relevant investigations for weight loss showed a mass in the sigmoid colon (through colonoscopy). It was diagnosed as carcinoma colon and he is waiting for planned surgery. On examination, he was febrile, tachycardic, tachypnoeic, blood pressure is 100/60 mm of Hg. On auscultation a systolic murmur was heard. Transthoracic 2D Echocardiography showed vegetations on the mitral valve. So, he was diagnosed as a case of Infective Endocarditis.
_x000D_
Among the following organisms which one is associated with above mentioned disease process?
CorrectIncorrectHint
Streptococcus bovis induced Infective Endocarditis (IE) has higher prevalence among the aged population (more than 60 years of age) and it is strongly associated with neoplastic diseases of colon. There are few studies by which a general concept has been taken that Streptococcus bovis bacteraemia should be evaluated for neoplastic diseases of colon immediately. So, Colonoscopy is the basic minimum investigation to be done in case of Streptococcus bovis Infective Endocarditis (IE).
-
Question 87 of 141
87. Question
A 66-year-old gentleman came to medicine outdoor with complaining of progressive tiredness and lethargy for the last few months. He was also complaining of lack of concentration into his daily work. On detailed discussion it was revealed that, he had history of new onset Atrial Fibrillation (AF) for which he was on tablet Amiodarone 200 mg twice daily for last six months. On examination his vitals were normal with controlled rate irregular pulse, no palpable neck swelling or gland noted. Other systemic and general examinations will be normal limits. No anaemia and signs of heart failure were noted.
_x000D_
His recent investigations are as followed:
_x000D_
_x000D_ _x000D_
_x000D_ _x000D_ Serum Free T4 (Thyroxine)
_x000D_
_x000D_
_x000D_ 21 pmol/L
_x000D_
_x000D_
_x000D_ (9-26)
_x000D_
_x000D_
_x000D_
_x000D_ _x000D_ Serum Total T3 (Triiodothyronine)
_x000D_
_x000D_
_x000D_ 0.5 nmol/L
_x000D_
_x000D_
_x000D_ (0.9-2.8)
_x000D_
_x000D_
_x000D_
_x000D_ _x000D_ Serum Thyroid Stimulating Hormone (TSH)
_x000D_
_x000D_
_x000D_ 9.8 mU/L
_x000D_
_x000D_
_x000D_ (<5)
_x000D_
_x000D_
_x000D_
_x000D_
_x000D_
How will you explain the clinical scenario and the blood parameters here?
CorrectIncorrectHint
Here we can see, Triiodothyronine (T3) is less than normal and Thyroxine (T4) is normal with raised thyroid stimulating hormone (TSH) which ultimately implies hypothyroidism secondary to Amiodarone use. Amiodarone can cause blocking of peripheral conversion of T4 to T3.
-
Question 88 of 141
88. Question
A 66-year-old gentleman arrived at the Emergency Department with a sudden episode of left-sided weakness that lasted for about twenty minutes and fully resolved spontaneously. Examination reveals that he has an ongoing Atrial Fibrillation. In this situation which of the following is the most appropriate management regime?
CorrectIncorrectHint
Ongoing Atrial Fibrillation with features of thromboembolic events suggested the initiation of anticoagulation as early as possible (CHADS2 score 2). Target INR range will be 2-3.
-
Question 89 of 141
89. Question
A 65-years-old lady was brought to the Emergency Department (ED) with history of sudden onset crushing chest pain, profuse sweating, and difficulty in breathing. The chest pain was centrally located and often radiating to back. The Electrocardiogram (ECG) done during transport of the patient showed Acute Myocardial Infarction with ST elevation in leads V2-V4 and ST depression in leads II and III (corresponding changes). At ED, she was planned for thrombolysis.
_x000D_
What is the absolute contraindication for thrombolysis in this lady?
CorrectIncorrectHint
The absolute contraindications for thrombolysis:_x000D_
• Active internal bleeding of various reasons_x000D_
• Pregnancy_x000D_
• Severe and uncontrolled hypertension._x000D_
• Ischaemic stroke within six months_x000D_
• Recent significant head injury_x000D_
• Intracranial neoplasm or space occupying lesion (SOL)_x000D_
• Aortic dissection -
Question 90 of 141
90. Question
A 65-year-old lady with known background of chronic kidney disease came to cardiology clinic with history of new onset palpitations for last two days. On examination, it was found that she is suffering from atrial fibrillation with high ventricular rate (162/minute). Her recent creatinine value is 188 mmol/l (70-110). The cardiologist consider for loading dose of digoxin to control her high ventricular rate.
_x000D_
How loading dose of digoxin will be assessed for this lady?
CorrectIncorrectHint
Digoxin is excreted through kidneys. So, maintenance dose depends on renal functions. The loading dose is calculated according to the volume of distribution of a drug. It is calculated by various factors like age of the recipient, creatinine clearance, body surface area etc. So, from the above-mentioned options the loading dose determinant will be renal clearance not the volume of distribution here as digoxin has very narrow therapeutic index.
-
Question 91 of 141
91. Question
A 65-year-old lady with atrial fibrillation (AF) gets admitted for direct-current (DC) cardioversion and the procedure resulted in successful restoration of sinus rhythm.
_x000D_
Which one of the following drugs would most likely be appropriate to maintain sinus rhythm following the direct-current (DC) cardioversion?
CorrectIncorrectHint
The most likely appropriate drug to maintain sinus rhythm after DC cardioversion is A. Amiodarone.
_x000D_
Amiodarone is a class III antiarrhythmic drug that prolongs the action potential and refractory period of cardiac cells, thus preventing the re-entry of electrical impulses that cause atrial fibrillation (AF). Amiodarone has been shown to be effective in reducing the recurrence of atrial fibrillation (AF) after cardioversion, especially in patients with structural heart disease or heart failure.
_x000D_
The other drugs are less suitable for this purpose because:
_x000D_
- _x000D_
- B. Sotalol is also a class III antiarrhythmic drug, but it has a higher risk of proarrhythmia (especially Torsades de pointes) than amiodarone. Sotalol also requires dose adjustment based on renal function and QT interval monitoring.
- C. Verapamil is a calcium channel blocker that slows down the atrioventricular (AV) node conduction, thus controlling the ventricular rate in atrial fibrillation (AF). However, verapamil does not have a significant effect on maintaining sinus rhythm after cardioversion.
- D. Diltiazem is another calcium channel blocker that has a similar mechanism and effect as verapamil.
- E. Digoxin is a cardiac glycoside that inhibits the sodium-potassium ATPase pump, thus increasing the intracellular calcium and enhancing the contractility of the heart. Digoxin also slows down the atrioventricular (AV) node conduction, but it does not prevent atrial fibrillation (AF) recurrence after cardioversion. Digoxin has a narrow therapeutic window and can cause toxicity if overdosed or interacted with other drugs.
_x000D_
_x000D_
_x000D_
_x000D_
-
Question 92 of 141
92. Question
A 64-year-old lady, a heavy smoker for many years, has had worsening laboured breathing for the past seven years, without a significant cough.
_x000D_
A chest radiograph shows increased lung size along with flattening of the diaphragm, consistent with emphysema. Over the next several years she develops worsening peripheral oedema. Her blood pressure is 110/70 mmHg.
_x000D_
Which one of the following cardiac findings is most likely to be present in this lady?
CorrectIncorrectHint
The lady in the question has emphysema, which is a type of chronic obstructive pulmonary disease (COPD) that causes damage to the air sacs in the lungs and reduces the oxygen exchange. Emphysema can lead to various cardiac complications, such as pulmonary hypertension, cor pulmonale, and right-sided heart failure.
_x000D_
Pulmonary hypertension is a condition in which the blood pressure in the arteries that supply the lungs is abnormally high, due to the increased resistance caused by the lung damage. This puts extra strain on the right ventricle, which pumps blood to the lungs, and can cause it to enlarge and thicken. This is called right ventricular hypertrophy.
_x000D_
Cor pulmonale is a condition in which the right ventricle fails to pump blood effectively, due to the pulmonary hypertension. This can cause fluid to back up in the veins and leak into the tissues, causing swelling or oedema, especially in the legs and ankles. This is called right-sided heart failure.
_x000D_
Therefore, the most likely cardiac finding in this lady is D. Right ventricular hypertrophy, as it is a common consequence of emphysema and pulmonary hypertension. It can be detected by echocardiography, which can measure the size and function of the right ventricle.
_x000D_
The other options are less likely to be present in this lady, as they are not directly related to emphysema or pulmonary hypertension. Mitral valve stenosis is a narrowing of the valve that separates the left atrium and the left ventricle, which can cause left-sided heart failure and pulmonary congestion. Non-bacterial thrombotic endocarditis is a condition in which small clots form on the heart valves, which can cause embolism and infection. Left ventricular aneurysm is a bulging of the wall of the left ventricle, which can impair its function and cause arrhythmias and thrombosis. Constrictive pericarditis is a condition in which the sac that surrounds the heart becomes stiff and inflamed, which can restrict the filling of the heart chambers and cause heart failure.
_x000D_
In summary, the best answer to the question is D. Right ventricular hypertrophy, as it is a common cardiac complication of emphysema and pulmonary hypertension. The other options are either not associated with emphysema or less likely to cause the symptoms of the lady.
-
Question 93 of 141
93. Question
A 65-year-old gentleman with a four month history of worsening dyspnoea visits the pulmonology clinic for a review. He has been healthy, well and fit all throughout his life so far with no major illnesses.
_x000D_
On examination, his blood pressure is 120/90 mmHg, he has a murmur, and has audible crackles at both bases. His serum glucose is normal with 5.6 mmol/L (3.0-6.0); total serum cholesterol is 4.8 mmol/L (<5.2) and serum creatine kinase is within the normal range.
_x000D_
Which one of the following is the most likely explanation for his worsening dyspnoea?
CorrectIncorrectHint
Based on the information provided as above, the most likely explanation for the worsening dyspnoea in this gentleman is A. Calcified bicuspid aortic valve.
_x000D_
A calcified bicuspid aortic valve is a condition where the aortic valve, which normally has three leaflets, has only two leaflets that become stiff and thickened over time. This causes the valve to narrow and restrict blood flow from the heart to the rest of the body. This can lead to symptoms such as dyspnoea, chest pain, fainting, and heart murmur.
_x000D_
The other options are less likely because:
_x000D_
- _x000D_
- Mitral valve annulus calcification is a degenerative process that affects the ring of tissue around the mitral valve, which separates the left atrium and the left ventricle. It can cause mitral regurgitation, where blood leaks back into the left atrium, or mitral stenosis, where the valve opening is narrowed. However, these conditions usually cause dyspnoea on exertion, not at rest, and are more common in women than men.
_x000D_
_x000D_
- _x000D_
- Aortic dissection is a life-threatening condition where the inner layer of the aorta, the large artery that carries blood from the heart, tears and separates from the outer layer. This can cause severe chest or back pain, a difference in blood pressure between the arms, and a weak or absent pulse in one arm. It can also affect the blood supply to the brain, kidneys, and other organs. However, aortic dissection is rare and usually occurs in people with high blood pressure, connective tissue disorders, or a history of trauma or surgery.
_x000D_
_x000D_
- _x000D_
- Alcoholic cardiomyopathy is a type of heart failure caused by long-term heavy alcohol consumption. It weakens the heart muscle and reduces its ability to pump blood effectively. It can cause symptoms such as dyspnoea, fatigue, swelling of the legs and abdomen, and irregular heartbeat. However, alcoholic cardiomyopathy usually affects people who drink more than 14 drinks per week for men or seven drinks per week for women, and the patient has no history of alcohol abuse.
_x000D_
_x000D_
- _x000D_
- Tricuspid valve endocarditis is an infection of the tricuspid valve, which separates the right atrium and the right ventricle. It can cause fever, chills, night sweats, weight loss, and a new or changed heart murmur. It can also cause complications such as pulmonary embolism, heart failure, and abscesses in the heart or other organs. However, tricuspid valve endocarditis is uncommon and usually occurs in people who have a history of intravenous drug use, congenital heart defects, or implanted devices such as pacemakers or catheters.
_x000D_
-
Question 94 of 141
94. Question
A 65-year-old lady is admitted for a surgical procedure and undergoes a routine pre-anaesthetic check-up, during which she is noted to have a soft systolic murmur at the left sternal edge. Further electrocardiogram (ECG) and chest x-ray were done and they come out to be normal but a transthoracic echocardiography revealed a small posterior pericardial effusion with normal valves.
_x000D_
Which one of the following would be the most appropriate next step in this lady’s management?
CorrectIncorrectHint
Considering the details provided in this case, the most suitable course of action for managing this lady would be to proceed with option B, which involves providing reassurance.
_x000D_
This is because a small pericardial effusion with normal valves and no signs of cardiac tamponade or infection is usually benign and does not require invasive procedures or specific treatment. A follow-up echocardiogram may be warranted to monitor the size and haemodynamic impact of the effusion.
_x000D_
The other options are less appropriate because:
_x000D_
- _x000D_
- Right heart catheterisation is an invasive procedure that measures the pressures and flows in the right side of the heart and the pulmonary circulation. It is usually indicated for patients with suspected pulmonary hypertension, right ventricular dysfunction, or valvular heart disease. It is not necessary for a patient with a small pericardial effusion and normal valves.
- A diagnostic pericardial aspiration is a procedure that involves inserting a needle into the pericardial space and draining the fluid for analysis. It is usually indicated for patients with large, symptomatic, or haemodynamically significant pericardial effusions, or when there is a suspicion of infection, malignancy, or autoimmune disease. It is not necessary for a patient with a small pericardial effusion and no signs of these conditions.
- A purified protein derivative test for tuberculosis is a skin test that detects the presence of antibodies to the bacteria that cause tuberculosis. It is usually indicated for patients with a history of exposure to tuberculosis, or who have symptoms or signs of active tuberculosis infection. It is not necessary for a patient with a small pericardial effusion and no signs of tuberculosis.
- Mammography is a screening test that uses low-dose x-rays to detect breast cancer. It is usually indicated for women aged 50 to 74 years who have no symptoms or signs of breast cancer. It is not related to the management of pericardial effusion.
_x000D_
_x000D_
_x000D_
_x000D_
-
Question 95 of 141
95. Question
A 64-year-old gentleman came to the Emergency Department with history of loose motion and vomiting for the last three days. He is known hypertensive and on Amlodipine and hydrochlorothiazide for last one and half years. On examination, he looks dehydrated, sick and his Blood Pressure is 110/66 mmHg and a Pulse Rate of 89/Minute. Other vitals signs are within normal limit. His abdomen is soft but diffusely tender to touch.
_x000D_
His blood parameters are:
_x000D_
_x000D_ _x000D_
_x000D_ _x000D_ Haemoglobin
_x000D_
_x000D_
_x000D_ 15.2 g/L
_x000D_
_x000D_
_x000D_ (115-160)
_x000D_
_x000D_
_x000D_
_x000D_ _x000D_ White Blood Cell Count
_x000D_
_x000D_
_x000D_ 9.0 ×109/L
_x000D_
_x000D_
_x000D_ (4-10)
_x000D_
_x000D_
_x000D_
_x000D_ _x000D_ Platelet Count
_x000D_
_x000D_
_x000D_ 200 ×109/L
_x000D_
_x000D_
_x000D_ (150-400)
_x000D_
_x000D_
_x000D_
_x000D_ _x000D_ Serum Sodium
_x000D_
_x000D_
_x000D_ 144 mmol/L
_x000D_
_x000D_
_x000D_ (134-143)
_x000D_
_x000D_
_x000D_
_x000D_ _x000D_ Serum Potassium
_x000D_
_x000D_
_x000D_ 2.7 mmol/L
_x000D_
_x000D_
_x000D_ (3.5-5)
_x000D_
_x000D_
_x000D_
_x000D_ _x000D_ Serum Creatinine
_x000D_
_x000D_
_x000D_ 143 μmol/L
_x000D_
_x000D_
_x000D_ (60-120)
_x000D_
_x000D_
_x000D_
_x000D_
_x000D_
What will be his ECG finding from the below options?
CorrectIncorrectHint
He is having severe hypokalaemia with dehydration (due to the disease process and diuretic therapy). Hypokalaemia usually have following ECG findings:
_x000D_
- _x000D_
- Progressively flattening of T-waves
- Non-specific T-wave inversion
- Prominent U-waves
- Non-specific ST depression
- QT prolongation
- Occasional premature ventricular ectopic
- Wide QRS complex arrhythmia
_x000D_
_x000D_
_x000D_
_x000D_
_x000D_
_x000D_
_x000D_
_x000D_
So, from the options ST depression will be the answer here. Hypokalaemia if untreated may worsens with wide QRS complex arrhythmia (VT/Tosades de pointes). Hypokalaemia can present with tiredness (fatigue), general weakness, muscle pain (myalgias) or muscle paralysis.
-
Question 96 of 141
96. Question
A 64-year-old gentleman came to cardiology clinic with complaints of generalised myalgia (limbs and trunk) for last few months after having being on new sets of medications. On detailed discussion, he revealed that he is diagnosed with ischemic heart disease and new onset diabetes mellitus for last few months. He was on simvastatin and metformin. His recent creatinine kinase reports found to be normal but his lipid profile seems to be deranged.
_x000D_
_x000D_
_x000D_ _x000D_
_x000D_ _x000D_ Total Cholesterol
_x000D_
_x000D_
_x000D_ 6.1 mmol/l
_x000D_
_x000D_
_x000D_ (<5.2)
_x000D_
_x000D_
_x000D_
_x000D_ _x000D_ Low-Density Lipoprotein (LDL) Cholesterol
_x000D_
_x000D_
_x000D_ 3.9 mmol/l
_x000D_
_x000D_
_x000D_ (<3.36)
_x000D_
_x000D_
_x000D_
_x000D_ _x000D_ High-Density Lipoprotein (HDL) Cholesterol
_x000D_
_x000D_
_x000D_ 1.26 mmol/l
_x000D_
_x000D_
_x000D_ (>1.55)
_x000D_
_x000D_
_x000D_
_x000D_ _x000D_ Serum Triglycerides
_x000D_
_x000D_
_x000D_ 1.5 mmol/l
_x000D_
_x000D_
_x000D_ (0.45-1.69)
_x000D_
_x000D_
_x000D_
_x000D_
_x000D_
Among the following options which drug would you suggest to control his dyslipidaemia?
CorrectIncorrectHint
From the given clinical scenario the most probable diagnosis is statin induced myopathy. As creatinine kinase reports found to be normal, so possibility rhabdomyolysis or severe myositis are unlikely (the approximate occurrence is 0.1 to 0.2%). So, among the following options ezetimibe has least chance of rhabdomyolysis or severe myositis. More-over it can help to reduce LDL also.
_x000D_
Nicotinic acid will be useful for hypertriglyceridemia.
-
Question 97 of 141
97. Question
A 63-year-old lady is admitted in the Emergency Department with sudden onset of chest pain and is diagnosed with an acute myocardial infarction (MI).
_x000D_
Her acute illness is complicated by low blood pressure and poor tissue perfusion for several days. Her serum lactate becomes elevated. Her serum urea and creatinine are noted to be increasing.
_x000D_
_x000D_ _x000D_
_x000D_ _x000D_ _x000D_
_x000D_
_x000D_ Day 1
_x000D_
_x000D_
_x000D_ Day 2
_x000D_
_x000D_
_x000D_ Day 3
_x000D_
_x000D_
_x000D_ Normal Range
_x000D_
_x000D_
_x000D_
_x000D_ _x000D_ Serum Urea (mmol/L)
_x000D_
_x000D_
_x000D_ 10
_x000D_
_x000D_
_x000D_ 24
_x000D_
_x000D_
_x000D_ 36
_x000D_
_x000D_
_x000D_ 2.5-7.5 mmol/L
_x000D_
_x000D_
_x000D_
_x000D_ _x000D_ Serum Creatinine (µmol/L)
_x000D_
_x000D_
_x000D_ 118
_x000D_
_x000D_
_x000D_ 144
_x000D_
_x000D_
_x000D_ 189
_x000D_
_x000D_
_x000D_ 60-110 µmol/L
_x000D_
_x000D_
_x000D_
_x000D_
_x000D_
Granular and hyaline casts are present on urine routine and microscopy examination.
_x000D_
Which one of the following renal lesions is most likely to be present in this situation?
CorrectIncorrectHint
This lady has acute kidney injury (AKI), which is a sudden decline in kidney function that can occur due to various causes. Acute kidney injury (AKI) is a common complication of acute myocardial infarction (AMI), especially in patients with cardiogenic shock, low blood pressure, and poor tissue perfusion. Acute kidney injury (AKI) can lead to increased morbidity and mortality in patients with acute myocardial infarction (AMI).
_x000D_
The most likely renal lesion to be present in this lady is C. Acute tubular necrosis (ATN). Acute tubular necrosis (ATN) is a condition where the tubular cells of the kidney are damaged or destroyed due to ischaemia (lack of blood supply) or nephrotoxicity (exposure to harmful substances). Acute tubular necrosis (ATN) can cause a rise in serum urea and creatinine, as well as the presence of granular and hyaline casts in the urine. Acute tubular necrosis (ATN) is the most common cause of acute kidney injury (AKI) in acute myocardial infarction (AMI) patients, especially those with cardiogenic shock.
_x000D_
The other renal lesions are less likely to be present in this lady. Minimal change disease is a glomerular disorder that causes nephrotic syndrome (proteinuria, hypoalbuminaemia, oedema, and hyperlipidaemia). It is usually idiopathic or associated with allergic or immune-mediated conditions. Renal vein thrombosis is a condition where the renal vein is blocked by a blood clot, causing reduced blood flow and swelling of the kidney. It can cause haematuria, flank pain, and renal impairment. It is usually associated with nephrotic syndrome, malignancy, trauma, or hypercoagulable states. Pyelonephritis is a bacterial infection of the kidney and the urinary tract, causing fever, flank pain, dysuria, and pyuria. It can cause renal impairment if complicated by abscess, obstruction, or sepsis. Nodular glomerulosclerosis is a glomerular disorder that causes thickening and scarring of the glomeruli, leading to proteinuria and renal impairment. It is usually associated with diabetes mellitus or hypertension.
-
Question 98 of 141
98. Question
A 63-year-old gentleman with a nine-year history of type 2 diabetes mellitus (T2DM) is being treated with metformin 1 gm twice daily and gliclazide 160 mg twice daily.
_x000D_
He is obese, has gained weight over the last eight months and his serum HbA1c has deteriorated from 58 to 66 mmol/mol (20-42). He is being considered for treatment with either Insulin or pioglitazone.
_x000D_
He wants to know the side effects of pioglitazone.
_x000D_
Which one of the following is regarded as a typical side effect of pioglitazone therapy?
CorrectIncorrectHint
Pioglitazone is a medicine used to treat type 2 diabetes mellitus by lowering the blood sugar level and preventing long-term complications. However, it may also cause some unwanted side effects in some people.
_x000D_
According to the health research studies, the most common side effects of pioglitazone are:
_x000D_
- _x000D_
- Weight gain
- Blurred vision
- Respiratory tract infection
- Numbness
- Bone fracture
_x000D_
_x000D_
_x000D_
_x000D_
_x000D_
_x000D_
Some of the serious side effects of pioglitazone are:
_x000D_
- _x000D_
- Congestive heart failure
- Chest pain
- Swelling of the face, fingers, feet, or lower legs
- Trouble breathing
- Unusual bleeding or bruising
- Dark urine
- Yellow eyes or skin
_x000D_
_x000D_
_x000D_
_x000D_
_x000D_
_x000D_
_x000D_
_x000D_
Based on this information, the correct answer to this question is C. Fluid retention. This is regarded as a typical side effect of pioglitazone therapy, as it occurs in more than 10% of patients treated with this medicine. Fluid retention can lead to oedema, which is the swelling of body tissues due to excess fluid.
_x000D_
Let us discuss why the other answer options are incorrect:
_x000D_
- _x000D_
- A. Photosensitivity rash: This is not a typical side effect of pioglitazone, but it may occur with some other medicines used to treat diabetes, such as sulfonylureas (like gliclazide) or glinides. A photosensitivity rash is a skin reaction that occurs when exposed to sunlight or artificial light sources.
- B. Lactic acidosis: This is a rare but serious side effect of metformin, not pioglitazone. Lactic acidosis is a condition where the body produces too much lactic acid, which can cause symptoms such as nausea, vomiting, stomach pain, rapid breathing, and low blood pressure.
- D. Myositis: This is not a typical side effect of pioglitazone, but it may occur with some other medicines used to treat diabetes, such as statins or fibrates. Myositis is a condition where the muscles become inflamed and painful, which can affect movement and strength.
- E. Acanthosis nigricans: This is not a side effect of pioglitazone, but it may be associated with diabetes itself or other conditions such as obesity, hormonal disorders, or cancer. Acanthosis nigricans is a skin condition where the skin becomes thickened and darkened, especially in the folds and creases of the body.
_x000D_
_x000D_
_x000D_
_x000D_
-
Question 99 of 141
99. Question
A 63-year-old gentleman is admitted to the Emergency Department with central crushing chest pain, profuse sweating, and vomiting for around an hour.
_x000D_
He is conscious with a pulse rate of 105 beats per minute and a blood pressure of 170/100 mmHg. An electrocardiogram (ECG) shows > 2 mm ST elevation in leads II, III, aVF. Complete blood count (CBC) and serum urea and serum electrolytes (U&Es) are normal. Troponin T is 100 ng/ml.
_x000D_
Apart from the presence of xanthelasma, there are no other positive findings on clinical examination.
_x000D_
He is given oxygen, aspirin, clopidogrel, morphine, and Inj. atenolol 5 mg intravenously.
_x000D_
Which one of the following is the best next step that should be taken?
CorrectIncorrectHint
The best next step that should be taken is A. Immediate referral to a cardiologist for primary angioplasty. This is because this gentleman has a ST-elevation myocardial infarction (STEMI), which is a type of heart attack that requires urgent revascularization of the blocked coronary artery. Primary angioplasty is a procedure that uses a balloon and a stent to open the artery and restore blood flow to the heart muscle. It is the preferred treatment for ST-elevated myocardial infarction (STEMI) if it can be performed within 120 minutes of the first medical contact.
_x000D_
The other options are less appropriate because they do not provide timely revascularization or address the underlying cause of the ST-elevated myocardial infarction (STEMI). Thrombolysis is an alternative to primary angioplasty, but it has lower efficacy and higher risk of bleeding complications. It should only be used if primary angioplasty is not available or delayed beyond 120 minutes. Low molecular weight heparin is an anticoagulant that prevents further clot formation, but it does not dissolve the existing clot or open the artery. It can be used as an adjunctive therapy to primary angioplasty or thrombolysis, but not as a standalone treatment. Transfer to coronary care unit is a necessary step for monitoring and supportive care, but it does not replace the need for revascularization. Simvastatin is a statin drug that lowers cholesterol and reduces the risk of future cardiovascular events, but it does not have an immediate effect on the acute ST-elevated myocardial infarction (STEMI). It can be started after the revascularization, but not before. Therefore, the answer is A. Immediate referral to a cardiologist for primary angioplasty.
-
Question 100 of 141
100. Question
A 62-year-old gentleman was suffering from systemic hypertension for long duration. Initially he was on Amlodipine but even after maximum doses of its BP was not controlled. So, subsequently hydrochlorothiazide and Losartan also being added to control his blood pressure. But even after administering 3 different anti-hypertensive drugs BP remains uncontrolled. The physician added Aliskiren to combat the BP.
_x000D_
Aliskiren acts which of the following mechanism?
CorrectIncorrectHint
From the above scenario probably, he is suffering from hyperactivity of Renin-Angiotensin System (RAS). In spite of having Angiotensin Receptor Blocker (Losartan), Calcium Channel Blocker (Amlodipine) and Diuretics (Hydrochlorothiazide) the BP is not controlling. So, to supress the RAS here needs direct renin-inhibitor, Aliskiren. In the RAS system conversion from Angiotensinogen to Angiotensin I is governed by renin as catalyst. Aliskiren actively blocks renin and prevents formation of Angiotensin I.
-
Question 101 of 141
101. Question
A 62-year-old gentleman is admitted to the Emergency Department with an excruciating chest pain of six hours of duration and electrocardiogram (ECG) done at the time of his admission shows a ST elevation in the inferior leads.
_x000D_
An electrocardiogram (ECG) from a previous clinic visit three months ago shows a sinus rhythm. He has insulin-dependent diabetes mellitus (IDDM) and chronic renal failure (CRF).
_x000D_
Investigations Reveal:
_x000D_
_x000D_ _x000D_
_x000D_ _x000D_ Fasting Plasma Glucose
_x000D_
_x000D_
_x000D_ 7.6 mmol/L
_x000D_
_x000D_
_x000D_ (3.0-6.0)
_x000D_
_x000D_
_x000D_
_x000D_ _x000D_ Serum Sodium
_x000D_
_x000D_
_x000D_ 138 mmol/L
_x000D_
_x000D_
_x000D_ (137-144)
_x000D_
_x000D_
_x000D_
_x000D_ _x000D_ Serum Potassium
_x000D_
_x000D_
_x000D_ 4.2 mmol/L
_x000D_
_x000D_
_x000D_ (3.5-4.9)
_x000D_
_x000D_
_x000D_
_x000D_ _x000D_ Serum Urea
_x000D_
_x000D_
_x000D_ 12 mmol/L
_x000D_
_x000D_
_x000D_ (2.5-7.5)
_x000D_
_x000D_
_x000D_
_x000D_ _x000D_ Serum Creatinine
_x000D_
_x000D_
_x000D_ 210 µmol/L
_x000D_
_x000D_
_x000D_ (60-110)
_x000D_
_x000D_
_x000D_
_x000D_
_x000D_
Which one of the following represents an absolute contraindication to the use of thrombolysis?
CorrectIncorrectHint
Based on the information provided above in the question, this gentleman has a high probability of having an acute inferior myocardial infarction (MI), which is a type of heart attack that affects the lower part of the heart muscle. Thrombolysis is a treatment that involves injecting a clot-dissolving drug into a vein to restore blood flow to the heart and limit the damage to the heart muscle.
_x000D_
However, thrombolysis is not suitable for everyone, as it can increase the risk of bleeding complications. There are some absolute and relative contraindications to thrombolysis, which means that the treatment should not be given or should be given with caution, respectively.
_x000D_
Among the options listed above, the correct answer is A. Previous history of haemorrhagic stroke. This is an absolute contraindication to thrombolysis, as it increases the risk of intracranial bleeding, which can be fatal. Other absolute contraindications include known structural cerebral vascular lesion, known malignant intracranial neoplasm, ischaemic stroke within 3 months (excluding stroke within 3 hours), suspected aortic dissection, active bleeding or bleeding diathesis (excluding menses), and significant closed-head trauma or facial trauma within 3 months.
_x000D_
The other options are not absolute contraindications, but they may be relative contraindications or warnings, depending on the severity and timing of the condition. For example, gastrointestinal bleeding in the last two months (B) may increase the risk of rebleeding, but it may not preclude thrombolysis if the bleeding has stopped and the patient is haemodynamically stable. On warfarin therapy (C) may increase the risk of bleeding, especially if the international normalized ratio (INR) is >1.7, but it may not be an absolute contraindication if the INR is <1.7 and the patient has a high risk of death or disability from myocardial infarction (MI). Ischaemic stroke a year ago (D) may increase the risk of intracranial bleeding, but it may not be an absolute contraindication if the stroke was mild and the patient has recovered well. Allergy to penicillin (E) is not a contraindication to thrombolysis, as penicillin is not a component of any thrombolytic drug.
-
Question 102 of 141
102. Question
A 62-year-old Afro-Caribbean lady presents at an outpatient clinic, which was full of patients that day, for a review.
_x000D_
She had a hysterectomy when she was 44 years of age for symptomatic uterine fibroids following completion of her family, and developed pre-eclampsia in all her three pregnancies. She is a current and lifelong smoker, takes no alcohol and previously worked as a personal assistant of the general manager of an export-import house.
_x000D_
Her current medications consist of estrace 300 mcg once daily, salbutamol when and as required, and seretide twice daily.
_x000D_
On examination, her blood pressure is 150/90 mmHg. Chest auscultation was normal. Examination is otherwise unremarkable, with normal fundoscopy, urine dipstick analysis and electrocardiogram (ECG).
_x000D_
Which one of the following additional strategies would be advisable for further controlling her hypertension?
CorrectIncorrectHint
The correct answer is D. Repeat blood pressure test later because this lady’s blood pressure may have been elevated due to the stressful situation of the outpatient clinic, and a single reading is not sufficient to diagnose hypertension. Therefore, she should be offered ambulatory blood pressure monitoring (ABPM), which measures the blood pressure at regular intervals during the day and night, and provides a more accurate and representative assessment of the blood pressure level and variability. The diagnosis of hypertension should be confirmed by the average value of at least 14 measurements taken during a patient’s usual waking hours. If this lady has hypertension, she should be advised to modify her lifestyle factors, such as quitting smoking, reducing weight, limiting salt and alcohol intake (not applicable in this case), and increasing physical activity, before starting pharmacotherapy. The choice of medication should be based on her age, ethnicity, comorbidities, and preferences.
_x000D_
Let us discuss why the other options are incorrect:
_x000D_
A. Prescribe spironolactone 25 mg: This is an incorrect answer because spironolactone is a potassium-sparing diuretic that can lower blood pressure by reducing fluid retention and blocking the action of aldosterone, a hormone that causes sodium and water retention. However, spironolactone may not be suitable for this lady, as it can interact with estrace, a form of oestrogen hormone replacement therapy, and increase the risk of hyperkalaemia, breast tenderness, and menstrual irregularities. Moreover, spironolactone is not recommended as a first-line treatment for hypertension, and should be used with caution in patients with renal impairment or diabetes.
_x000D_
B. Increase amlodipine to 10 mg: This is an incorrect answer because amlodipine is a calcium channel blocker that can lower blood pressure by relaxing the blood vessels and reducing the workload of the heart. However, increasing the dose of amlodipine may not be necessary or beneficial for this lady, as she may not have hypertension based on a single reading, and a higher dose may cause more side effects, such as fluid retention, oedema, and weight gain. Furthermore, amlodipine may also interact with estrace and increase the risk of these adverse effects. Therefore, she should be monitored with ambulatory blood pressure monitoring (ABPM) and lifestyle modification before increasing the dose of amlodipine.
_x000D_
C. Prescribe ramipril 10 mg: This is an incorrect answer because ramipril is an angiotensin-converting enzyme (ACE) inhibitor that can lower blood pressure by preventing the formation of angiotensin II, a hormone that causes vasoconstriction and sodium and water retention. However, prescribing ramipril may not be appropriate or effective for this lady, as she may not have hypertension based on a single reading, and ramipril may be less potent and more likely to cause adverse effects, such as cough, angioedema, and hyperkalaemia, in patients of African or Caribbean descent with hypertension. Additionally, the dose of ramipril is too high for a starting dose, and should be titrated gradually depending on this lady’s blood pressure, renal function, and tolerance.
_x000D_
E. Screen for phaeochromocytoma: This is an incorrect answer because phaeochromocytoma is a rare tumour of the adrenal glands that causes excessive production of catecholamines, such as adrenaline and noradrenaline, which can lead to severe and paroxysmal hypertension. However, this lady does not have any symptoms or signs suggestive of phaeochromocytoma, such as headache, palpitations, sweating, anxiety, tremor, or weight loss. Moreover, phaeochromocytoma is more common in younger patients and those with a family history of the condition or genetic syndromes, such as multiple endocrine neoplasia (MEN), von Hippel-Lindau disease, or neurofibromatosis. Therefore, screening for phaeochromocytoma is not indicated or warranted for this lady.
-
Question 103 of 141
103. Question
A 60-years-old lady visited the Cardiology Clinic with history of multiple episodes of syncope and syncope like symptoms over the past five months. She is anxious as the episode makes her life very uncomfortable and at times causing severe life-threatening situations. Currently she is anxious about her cardiac status so she had undergone a 72-Hour Electrocardiogram (ECG, HOLTER monitoring) Recording.
_x000D_
Which of the following would be the most significant finding on her HOLTER monitoring?
CorrectIncorrectHint
From the above options second degree heart block with Right Bundle Branch Block (RBBB) type has maximum propensity to develop complete heart block and associated syncope or syncope like episodes. Ideally pacemaker to be inserted to manage this situation prior confirmation of clinical and electrical overlapping of the symptoms._x000D_
Few SVT and Atrial or Ventricular Ectopics can rarely precipitate the above mention medical situation. -
Question 104 of 141
104. Question
A 61-year-old gentleman develops sudden severe chest pain that radiates to his back. After entering to the Emergency Reception, he collapses almost immediately. He was physically active throughout his life. He has a history of Hypertension. On detailed query it was found that he had undergone a recent Treadmill Test and no cardiac disease was diagnosed. He does not smoke nor does he drink alcohol and maintain a healthy food habit throughout his life.
_x000D_
Which one of the following is the most likely diagnosis?
CorrectIncorrectHint
As per the question and options given most probably, he was suffering from aortic dissection (AD). It occurs due to the tear of intimal layer of artery. It is usually associated with acute onset chest pain (radiating to back usually), sweating, vomiting, light-headedness and circulatory collapse. History of pre-existing hypertension is common. CT angiography is a very helpful tool to diagnose the condition provided if the person is hemodynamically stable. _x000D_
All other options given above may cause acute circulatory collapse but with normal TMT (Treadmill Test) and acute onset chest pain are rarely found with them. -
Question 105 of 141
105. Question
A 60-years-old lady came to endocrine clinic with complaints of easy fatigue, increased somnolence and both lower limbs swelling from ankle to toe for last 3 weeks. The symptoms were progressively increasing throughout the duration. On detailed enquiry it was revealed that she had received Radioactive Iodine (RAI) therapy 3 years before. Her latest Thyroid Function assessment was given below. Currently she is not in any significant apart from multi-vitamin capsules.
_x000D_
Here are her latest blood parameters:
_x000D_
_x000D_
_x000D_ _x000D_
_x000D_ _x000D_ Free Thyroxine (FT4)
_x000D_
_x000D_
_x000D_ 14.7 pmol/L
_x000D_
_x000D_
_x000D_ (10-22)
_x000D_
_x000D_
_x000D_
_x000D_ _x000D_ Serum Thyroid Stimulating Hormone (TSH)
_x000D_
_x000D_
_x000D_ 18 mU/L
_x000D_
_x000D_
_x000D_ (0.4-5)
_x000D_
_x000D_
_x000D_
_x000D_ _x000D_ Serum Total Cholesterol
_x000D_
_x000D_
_x000D_ 7.5 mmol/L
_x000D_
_x000D_
_x000D_ (<5.2)
_x000D_
_x000D_
_x000D_
_x000D_ _x000D_ Plasma Triglycerides
_x000D_
_x000D_
_x000D_ 3.6 mmol/L
_x000D_
_x000D_
_x000D_ (0.45-1.69)
_x000D_
_x000D_
_x000D_
_x000D_
_x000D_
Among the following which one is best treatment option for her?
CorrectIncorrectHint
As per the above-mentioned clinical features and biochemical parameters she is suffering from subclinical hypothyroidism (increased TSH & normal thyroxine). As she is developing features of hypothyroidism (non-pitting oedema, increased somnolence, generalise weakness) and significant hypercholesterolaemia and hypertriglyceridemia it is recommended to start levothyroxine therapy as early as possible. Subsequently dyslipidaemia will correct after the initiation of this therapy.
-
Question 106 of 141
106. Question
A 60-year-old male carpenter came to Emergency Department (ED) with complaints of central chest pain and profuse sweating during his work for last four hours. On detailed discussion he admits of having chronic smoking habits (seven cigarettes/day for more than twenty years) and recently detected type ll diabetes mellitus and hypertension. Currently he is on metformin and amlodipine. At ED, ECG showed ST depression along the antero-lateral wall. So, he was given nitro-glycerine spray along with loading dose of anti-platelet and statin. Gradually his chest pain subsides.
_x000D_
Among the following options which one is the most appropriate for this gentleman currently?
CorrectIncorrectHint
Probably he is suffering from Acute Coronary Syndrome (ACS). As per the question, diabetic and hypertensive along with chronic smoker with this age and clinical symptoms (chest pain & sweating) highly suggestive of ACS. ACS encompasses unstable Angina, ST Elevation Myocardial Infarction (STEMI) & Non-ST Elevated Myocardial Infarction (NSTEMI). ACS and stable angina should be differentiated from stable angina as the management of both disorders are different. The classical symptoms of ACS include:
_x000D_
- _x000D_
- Retrosternal Chest Pain (Central Chest Pain) – It may be localised or referred or radiating to arms, jaw, neck and back. It lasts usually 15-30 mins. Often associated with nausea, vomiting, profuse sweating, shortness of breath and unstable vitals. It may associate with circulatory collapse also.
- The pattern of chest pain/chest discomfort in stable angina is more predictable, associated with exercise, stress and cold weather. It is known as Angina Pectoris. Duration of chest pain is short lasting and regular. If untreated may terminate into chest pain of ACS.
_x000D_
_x000D_
_x000D_
Glyceryl Trinitrate (GTN/Nitro-glycerine) may respond quickly in both kind of chest pain. Also responds well in oesophageal spasm disorder. So, it should not be used to diagnose cardiac chest pain from non-cardiac one.
_x000D_
Ideally 12-lead ECG is diagnostic of ACS. But sometimes ECG changes may be evident several hours later. So, normal ECG of symptomatic patient does not rule out ACS. If ACS (STEMI) is being diagnosed through ECG, then immediately reperfusion therapy should be initiated either through thrombolysis or primary Percutaneous Coronary Intervention (PCI) as per set up and protocol. In case of NSTEMI serial troponin value (10-12 hours apart) should be checked as it has diagnostic as well as prognostic value. Once diagnosed with NSTEMI by cardiac marker troponin and corresponding ECG changes (ST depression, T wave inversion, Q wave) initial therapy of ACS should be started (anti-platelet, morphine/nitro-glycerine for chest pain, Beta-Blockers for supressing high sympathetic activity, oxygen inhalation if SpO2 is less than 94 percent and other cardiac remodelling agents with anticoagulants).
_x000D_
For normal ECG with symptomatic patients there are possibilities of cardiac as well as non-cardiac reason. So, serial ECG and troponin estimation (10-12 hours apart) have significant role to rule out cardiac cause of chest pain.
_x000D_
Endoscopy helps to rule out gastrointestinal causes of epigastric chest pain after ruling out cardiac cause first. So, not recommended here.
_x000D_
Coronary Angiography may be helpful for NSTEMI, unstable angina and positive stress test (treadmill test) patients. During PCI it is also done to locate the vessels.
_x000D_
Exercise Electrocardiogram (ECG) is done for those who had strong symptoms of ACS but resting ECG is inconclusive.
_x000D_
Echocardiogram is useful tool for myocardial functional activity only. Not a useful tool to diagnose ACS.
-
Question 107 of 141
107. Question
A 60-year-old lady, who has been receiving haemodialysis for the past seven years, is found dead in the bed by her son in the morning. She had a long history of type 1 diabetes for over 25 years and in her later years she suffered from neuropathy and retinopathy as well as chronic renal failure. Most recently she was under investigation for angina.
_x000D_
Which one of the following is the most likely cause of death in this lady?
CorrectIncorrectHint
The correct answer is E. Myocardial infarction (MI). This is because cardiovascular disease (CVD) is the major cause of morbidity and mortality in patients with end-stage renal disease (ESRD) on haemodialysis for a long term. This lady also had several risk factors for cardiovascular disease (CVD), such as diabetes mellitus, neuropathy, retinopathy, chronic renal failure, and angina. Moreover, cardiovascular disease (CVD) is the leading cause of death among dialysis patients, followed by sepsis/infection. Therefore, myocardial infarction (MI) is the most probable cause of death in this case.
_x000D_
The other options are less likely, because:
_x000D_
- _x000D_
- Patients who undergo long-term haemodialysis suffer from increasing arterial calcification, which is associated with both increased risk of myocardial infarction and stroke, but the greatest absolute increase is in myocardial infarction (MI) rates. This means that cerebrovascular accident (CVA) (option A) is less likely than myocardial infarction (MI) as a cause of death in this patient population.
- Pulmonary embolus (option B) is a possible complication of haemodialysis, but it is not very common. The incidence of pulmonary embolism in dialysis patients is estimated to be around 0.5% per year. Moreover, pulmonary embolism usually presents with acute symptoms such as dyspnoea, chest pain, and haemoptysis, which are not consistent with the scenario of this lady being found dead in the bed by her son in the morning.
- Hyperkalaemia (option C) is a serious electrolyte disorder that can occur in patients with renal failure, especially if they miss or skip dialysis sessions. Hyperkalaemia can cause cardiac arrhythmias and cardiac arrest. However, hyperkalaemia is usually preventable and treatable with adequate dialysis and dietary restriction of potassium. Furthermore, hyperkalaemia usually manifests with signs and symptoms such as muscle weakness, nausea, palpitations, and electrocardiographic changes, which are not compatible with the scenario of this lady being found dead in the bed by her son in the morning.
- Hypoglycaemia (option D) is a potential complication of diabetes mellitus, especially in patients who use insulin or oral hypoglycaemic agents. Hypoglycaemia can cause neurological impairment, seizures, coma, and death. However, hypoglycaemia is also preventable and treatable with appropriate monitoring and adjustment of glucose levels and medication doses. Moreover, hypoglycaemia usually presents with symptoms such as sweating, tremor, hunger, anxiety, confusion, and loss of consciousness, which are not consistent with the scenario of the lady being found dead in the bed by her son in the morning.
_x000D_
_x000D_
_x000D_
_x000D_
-
Question 108 of 141
108. Question
A 60-year-old lady, known alcoholic for long duration presents to her General Physician with complaints of back pain and mild diarrhoea for few days. On detailed enquiry it was found that a Permanent Pacemaker was inserted one and half months back. On examination, she had a fever of 102.2°F and her abdomen was soft and non-tender, with no distension.
_x000D_
Which one of the following is the most likely diagnosis?
CorrectIncorrectHint
Staphylococci are skin commensal. So, any percutaneous procedures like insertion of Pacemaker, central venous access have always high chance of developing Staphylococcal septicaemia. So here Staphylococcal Discitis is the most suitable option._x000D_
Though she is known long term alcoholic, possibility of pancreatitis unlikely here as the abdominal clinical signs and symptoms were absent. _x000D_
Ischemic colitis is usually associated with bloody diarrhoea. _x000D_
Chance of Pseudomembranous Colitis after prophylactic antibiotic therapy during Pacemaker insertion in unlikely after this long gap (after six weeks). -
Question 109 of 141
109. Question
A 60-year-old gentleman with a history of schizophrenia on thioridazine is found to have episodes of Torsades de pointes (TdP) ventricular tachycardia (VT).
_x000D_
His blood pressure is 120/70 mmHg.
_x000D_
Which one of the following is the most appropriate management for this gentleman?
CorrectIncorrectHint
Torsades de pointes (TdP) is a type of very fast and irregular heart rhythm that starts in the lower chambers of the heart (ventricles). It is often caused by a prolonged QT interval, which is the time it takes for the heart to recharge between beats. Some medications, such as thioridazine, can cause QT prolongation and increase the risk of Torsades de pointes (TdP).
_x000D_
The most appropriate management for this gentleman is B. IV magnesium. This is a mineral that can help stabilize the heart rhythm and prevent further episodes of Torsades de pointes (TdP). Magnesium sulfate, given as an infusion, is effective at aborting the arrhythmia and preventing recurrence.
_x000D_
The other options are not recommended for the following reasons:
_x000D_
- _x000D_
- A. Synchronised direct-current (DC) cardioversion: This is a procedure that delivers an electric shock to the heart to restore a normal rhythm. It is not indicated for Torsades de pointes (TdP), unless the patient is unstable or unresponsive to other treatments. It may also be difficult to synchronize the shock with the irregular QRS complexes of Torsades de pointes (TdP).
- C. IV beta-blocker: This is a type of medication that can lower the heart rate and blood pressure. It is not helpful for Torsades de pointes, as it may worsen the QT prolongation and increase the risk of ventricular fibrillation. Beta-blockers should be avoided in patients with Torsades de pointes (TdP).
- D. IV lidocaine: This is another type of medication that can treat fast and irregular heart rhythms. It is not effective for Torsades de pointes (TdP), as it may also prolong the QT interval and cause hypotension and cardiac depression. Lidocaine should be avoided in patients with Torsades de pointes (TdP).
- E. Overdrive pacing: This is a technique that uses a temporary pacemaker to speed up the heart rate and suppress the abnormal rhythm. It is not a first-line treatment for Torsades de pointes (TdP), as it may be difficult to achieve and maintain capture of the ventricles. It may also cause more harm than benefit in some cases. Overdrive pacing should be reserved for refractory or recurrent Torsades de pointes (TdP).
_x000D_
_x000D_
_x000D_
_x000D_
-
Question 110 of 141
110. Question
A 60-year-old gentleman presented to his general physician with complaint of gradual enlargement of his abdomen with mild heaviness over the past seven months.
_x000D_
He has experienced no abdominal or chest pain. On physical examination he has a non-tender abdomen with no masses palpable but a fluid thrill is found.
_x000D_
An ultrasound scan (USS) of abdomen shows a large abdominal fluid collection with a small cirrhotic liver.
_x000D_
A chest radiograph shows a globally enlarged heart.
_x000D_
Which one of the following is most likely to be present as an underlying cause of his abdominal enlargement?
CorrectIncorrectHint
Dilated cardiomyopathy (DCM) is the most likely cause of this gentleman’s abdominal enlargement, which is due to ascites. Ascites is a fluid accumulation in the abdominal cavity, which can occur as a complication of liver cirrhosis. Liver cirrhosis is a chronic liver disease characterized by scarring and impaired function of the liver. This gentleman has alcoholic liver cirrhosis, which is caused by long-term and excessive alcohol consumption.
_x000D_
Alcoholic liver cirrhosis can lead to dilated cardiomyopathy (DCM), which is a type of heart disease that affects the heart muscle. Dilated cardiomyopathy (DCM) causes the heart chambers to dilate and weaken, resulting in reduced pumping ability and heart failure. Heart failure is a condition in which the heart cannot meet the body’s demand for blood and oxygen. This can cause fluid retention in various parts of the body, including the abdomen, legs, and lungs.
_x000D_
The other options are less likely because:
_x000D_
- _x000D_
- B. Lymphocytic myocarditis is a rare form of inflammation of the heart muscle, usually caused by a viral infection. It can cause chest pain, arrhythmias, and heart failure, but it is not associated with fluid retention in the abdomen or a small cirrhotic liver. A chest radiograph may show a normal or slightly enlarged heart. An ultrasound scan may show a normal liver.
- C. Nonbacterial thrombotic endocarditis is a condition in which sterile clots form on the heart valves, usually due to a hypercoagulable state or malignancy. It can cause embolic complications, such as stroke or limb ischaemia, but it does not cause heart failure or fluid retention in the abdomen. A chest radiograph may show a normal heart. An ultrasound scan may show a normal liver.
- D. Severe occlusive coronary atherosclerosis is a condition in which the coronary arteries become narrowed or blocked by plaque, reducing blood flow to the heart muscle. It can cause angina, heart attack, or ischaemic cardiomyopathy, but it does not cause fluid retention in the abdomen or a small cirrhotic liver. A chest radiograph may show a normal or slightly enlarged heart. An ultrasound scan may show a normal or fatty liver.
- E. Myocardial amyloid deposition is a condition in which abnormal proteins called amyloids accumulate in the heart muscle, causing stiffness and reduced function. This can lead to heart failure, which is characterized by fluid retention in the abdomen and other parts of the body. A chest radiograph may show an enlarged heart due to increased wall thickness and chamber dilation. An ultrasound scan may show a small cirrhotic liver due to impaired blood flow from the heart. However, this condition is rare and usually associated with other systemic manifestations of amyloidosis, such as peripheral neuropathy, renal failure, and skin lesions.
_x000D_
_x000D_
_x000D_
_x000D_
-
Question 111 of 141
111. Question
A 60-year-old gentleman is admitted with a high blood pressure of 200/110 mmHg and episodic runs of ventricular tachycardia (VT).
_x000D_
Investigations confirm the presence of a right adrenal phaeochromocytoma.
_x000D_
Which one of the following would be the most appropriate initial therapy?
CorrectIncorrectHint
The gentleman in question is experiencing severe hypertension induced by catecholamines, along with paroxysmal ventricular tachycardia. The recommended approach involves initiating treatment with an alpha-blocker, specifically phenoxybenzamine, followed by the introduction of beta blockers.
_x000D_
_x000D_
This combination of medications is expected to effectively manage the episodes of ventricular tachycardia (VT). However, in cases where sustained ventricular tachycardia (VT) persists, the acute administration of lidocaine can be considered as an additional intervention.
_x000D_
Based on the above discussion, the most appropriate initial therapy for this gentleman with a phaeochromocytoma is D. Phenoxybenzamine. This is an alpha-blocker that lowers blood pressure by relaxing the blood vessels and preventing the effects of high-adrenaline hormones. Phenoxybenzamine is given before surgery to reduce the risk of hypertensive crisis during tumour removal.
_x000D_
The other options are not recommended for the following reasons:
_x000D_
- _x000D_
- A. Amiodarone: This is an antiarrhythmic drug that can treat ventricular tachycardia, but it may worsen blood pressure and heart failure in patients with phaeochromocytoma.
- B. Lidocaine: This is another antiarrhythmic drug that can treat ventricular tachycardia, but it may also cause hypotension and cardiac depression in patients with phaeochromocytoma.
- C. Atenolol: This is a beta-blocker that can lower heart rate and blood pressure, but it should not be used alone in patients with phaeochromocytoma, as it may cause unopposed alpha-adrenergic stimulation and paradoxical hypertension. Beta-blockers can only be added after adequate alpha-blockade is achieved.
- E. Propafenone: This is a third antiarrhythmic drug that can treat ventricular tachycardia, but it has similar adverse effects as lidocaine and amiodarone in patients with phaeochromocytoma.
_x000D_
_x000D_
_x000D_
_x000D_
-
Question 112 of 141
112. Question
A 59-year-old gentleman was admitted to coronary care unit with history of inferior wall acute Myocardial Infarction (MI). He was on standard anti ischemic therapy for last four days. Now he is complaining of new onset chest pain since yesterday night on and off.
_x000D_
Among the following bio-markers, which one will be the most suitable to detect reinfarction?
CorrectIncorrectHint
Creatine Kinase-MB (CK-MB) rises within 3-8 hours of AMI and reaches its peak within 9-30 hours. As it has also short half-life, so comes normal within 2-3 days. _x000D_
Post AMI earliest marker detected in blood is Myoglobin. It has short half-life (even lesser than CK-MB). So, will not be useful here in this scenario._x000D_
Serum Lactate Dehydrogenase (LDH) elevates lately, 10 hours after the onset AMI, peaks at 24-48 hours and remains elevated for 6-8 days._x000D_
Troponin I is highly sensitive and specific cardiac biomarker for AMI. It rises within hours of onset of AMI but can persist elevated state as long as 10-14 days. -
Question 113 of 141
113. Question
A 58-year-old gentleman came to cardiology clinic to control his long-standing hypertension which was not controlled with multiple anti-hypertensive agents for last eleven months. He was initially on tablet Ramipril 1.25 mg and Bendroflumethiazide 2.5 mg. But as his BP was not controlled with above regimen, his GP started tablet Amlodipine 10 mg and Bisoprolol 2.5 mg daily. It was continued for six months yet his Blood Pressure was pretty high (160/88 mmHg). Due to his persistent hypertension, lifestyle and physical activity have been hampered largely. On examination his BP is 162/88 mmHg, Pulse is 76/min and other vitals are acceptable. His Urine Dipstick is normal and Renal parameters are within acceptable range. ECG and Echocardiography showed features of Left Ventricular Hypertrophy (LVH) otherwise no abnormality detected.
_x000D_
What will be the appropriate therapy in this scenario?
CorrectIncorrectHint
From the above scenario, it seems that the person is suffering from chronic resistant hypertension. From his regimen Angiotensin Converting Enzyme Inhibitor (ACEI) i.e., tablet Ramipril can be increased from 1.25 mg to 2.5 mg. As his renal parameters are okay, so increasing dose of ACEI will most likely cause no harm immediately. He is already on maximum dose of Amlodipine and Bendroflumethiazide, so increasing their dose will have no beneficial effect on controlling his Blood Pressure.
_x000D_
Nitrates are recommended for relieving angina and in heart failure conditions. Isolated use of nitrate as an anti-hypertensive agent is not recommended as it may develop tolerance.
_x000D_
Spironolactone is used in heart failure (post myocardial infarction state), hyperaldosteronism, ascites secondary to portal hypertension and in nephrotic syndrome. As per the result of the investigations, the person is not suffering from any of the conditions mentioned above.
_x000D_
He is not a candidate for haemodialysis.
-
Question 114 of 141
114. Question
A 58-year-old gentleman rushes to the Emergency Department with sudden onset central pulverizing chest pain. He was thoroughly examined and initial examination is normal.
_x000D_
A 12-lead electrocardiogram (ECG) shows ST segment elevation in leads II, III, aVF and ST depression in V1, V2 and V3.
_x000D_
Which coronary artery is occluded?
CorrectIncorrectHint
The correct answer is C. Right coronary artery. This is because the electrocardiogram (ECG) findings of ST segment elevation in leads II, III, aVF, and ST depression in V1, V2, and V3 indicate an inferior-posterior myocardial infarction (MI), which is usually caused by occlusion of the right coronary artery. The right coronary artery supplies blood to the inferior and posterior walls of the left ventricle, as well as the right ventricle and the sinoatrial (SA) and atrioventricular (AV) nodes. Therefore, right coronary artery occlusion can result in inferior-posterior myocardial infarction (MI), as well as right ventricular myocardial infarction (MI) and heart block.
_x000D_
Let us explain why the other options are less likely:
_x000D_
- _x000D_
- Left main stem (option A) occlusion is a rare and catastrophic event that causes extensive ischaemia and necrosis of the left ventricle, which accounts for about 70% of the cardiac output. The electrocardiogram (ECG) findings of left main stem occlusion are ST segment elevation in leads V1 to V6, I, and aVL, and ST segment depression in leads II, III, and aVF.
- Left anterior descending (option B) occlusion is the most common cause of anterior myocardial infarction (MI), which affects the anterior wall and the septum of the left ventricle. The electrocardiogram (ECG) findings of left anterior descending occlusion are ST segment elevation in leads V1 to V4, and sometimes I and aVL.
- Obtuse marginal (option D) occlusion is a branch of the left circumflex artery that supplies the lateral wall of the left ventricle. The electrocardiogram (ECG) findings of obtuse marginal occlusion are ST segment elevation in leads I, aVL, V5, and V6.
- Circumflex (option E) occlusion is a branch of the left main stem artery that supplies the lateral and posterior walls of the left ventricle. The electrocardiogram (ECG) findings of circumflex occlusion are ST segment elevation in leads I, aVL, V5, and V6, and sometimes II, III, and aVF.
_x000D_
_x000D_
_x000D_
_x000D_
_x000D_
NOTE:
_x000D_
Understanding the territories supplied by the different coronary arteries is important in predicting the potential problems that may arise following a myocardial infarction (MI). Here is a breakdown of the territories supplied by each artery:
_x000D_
– Circumflex artery: This artery supplies the lateral region of the heart.
_x000D_
– Left anterior descending artery: This artery provides blood flow to the anterior (front) and septal (middle) regions of the heart.
_x000D_
– Left main stem: This artery branches into the left anterior descending artery and circumflex artery, supplying most of the left ventricle. Complete occlusion of the left main stem is typically fatal.
_x000D_
– Obtuse marginal artery: This is one of the branches of the circumflex artery and supplies the “high lateral” region of the left ventricle, as seen in electrocardiogram (ECG) leads I and aVL.
_x000D_
Having a basic understanding of coronary anatomy is crucial as it can help predict potential complications following a myocardial infarction. For instance, the right coronary artery supplies the atrioventricular (AV) node, so it is common to observe heart block following an inferior myocardial infarction. On the other hand, heart block following an anterior myocardial infarction is considered a grave prognostic marker, as it indicates a large infarction in the anterior wall of the heart.
_x000D_
By considering the specific territories supplied by different coronary arteries, healthcare professionals can better anticipate and manage complications that may arise following a myocardial infarction, ultimately improving patient outcomes
-
Question 115 of 141
115. Question
A 58-year-old lady with known history of heart failure came to cardiology clinic with complaints of left sided ankle pain and swelling for last few days. She was not having any history of trauma or new onset shortness of breath. On detailed discussion she revealed that she was on Digoxin therapy for management of her heart failure for last one year.
_x000D_
Among the following drugs which one will cause Digoxin Toxicity?
CorrectIncorrectHint
Digoxin toxicity may present with increased therapeutic concentration or even normal therapeutic concentration in plasma. The half-life of digoxin is around 36-48 hours. Digoxin is primarily excreted to urine. So, usually increased therapeutic concentration toxicity may happen in renal impairment. Some drug interactions may cause decreased clearance of it. Verapamil, macrolides antibiotics, antifungals like amphotericin, propylthiouracil, Beta Blockers, amiodarone, diuretics, benzodiazepines etc are chief offending drugs may cause decease clearance of digoxin and ultimate effects of toxicity._x000D_
Among the above options Bumetanide is loop diuretic, so it can cause digoxin toxicity. -
Question 116 of 141
116. Question
A 58-year-old male came to cardiology clinic with complaints of progressively increasing grade lll NYHA exertional dyspnoea for last six months. His Echocardiography revealed left ventricular ejection fraction 38%. He had history of ischemic heart disease and hypertension for several years. How is Left Ventricular Ejection Fraction calculated?
CorrectIncorrectHint
The ejection fraction is the percentage of stroke volume of blood pumped from the ventricles at the end of systole. It is calculated as {End Diastolic Volume (EDV) – End Systolic Volume (ESV)} ÷ by End Diastolic Volume (EDV) × 100. Stroke volume is {End Diastolic Volume (EDV) – End Systolic Volume (ESV)}.
-
Question 117 of 141
117. Question
A 56-year-old gentleman, a known case of hypertension controlled on medications, presents to the clinic with complaints of two episodes of fainting and one episode of seizure in the past three months with progressively worsening hearing loss over the past five months. Electrocardiogram (ECG) revealed a prolonged corrected QT Interval.
_x000D_
Which one of the following drugs is the most likely cause?
CorrectIncorrectHint
The most likely cause of the prolonged corrected QT interval (QTc) in this gentleman is D. Sotalol. Sotalol is a beta-blocker and an antiarrhythmic drug that can prolong the QT interval by blocking the potassium channels in the cardiac cells. Sotalol is associated with a high risk of Torsade de pointes (TdP), a life-threatening ventricular arrhythmia that can cause fainting and seizures. Sotalol can also cause hearing loss by affecting the cochlear hair cells.
_x000D_
The other drugs are less likely to cause QT prolongation and this gentleman’s symptoms. Moxonidine and telmisartan are antihypertensive drugs that have minimal or no effect on the QT interval. Digoxin is a cardiac glycoside that can shorten the QT interval by increasing the intracellular calcium. Cefaclor is an antibiotic that is not known to prolong the QT interval or cause hearing loss.
_x000D_
Therefore, the gentleman should be advised to stop taking sotalol and seek an alternative treatment for his hypertension and arrhythmia. He should also have his electrolytes checked and avoid other drugs that can prolong the QT interval.
_x000D_
There are many drugs that can prolong the QT interval, which is a measure of the time between the start of the Q wave and the end of the T wave in the heart’s electrical cycle. A prolonged QT interval can increase the risk of a dangerous arrhythmia called Torsade de pointes (TdP), which can cause fainting, seizures, or sudden death.
_x000D_
Some of the main groups of drugs that can prolong the QT interval are:
_x000D_
- _x000D_
- Antiarrhythmic drugs, such as amiodarone, sotalol, flecainide, and quinidine
- Antipsychotic drugs, such as chlorpromazine, haloperidol, risperidone, and ziprasidone
- Antidepressant drugs, such as citalopram, escitalopram, venlafaxine, and tricyclic antidepressants
- Antibiotic drugs, such as erythromycin, clarithromycin, moxifloxacin, and pentamidine
- Antifungal drugs, such as fluconazole, voriconazole, and itraconazole
- Antihistamine drugs, such as terfenadine, astemizole, and loratadine
- Antimalarial drugs, such as quinine, chloroquine, and mefloquine
- Anticancer drugs, such as arsenic trioxide, dasatinib, and vandetanib
- Other drugs, such as domperidone, methadone, ondansetron, and sildenafil
_x000D_
_x000D_
_x000D_
_x000D_
_x000D_
_x000D_
_x000D_
_x000D_
_x000D_
-
Question 118 of 141
118. Question
A 56-year-old gentleman with marked obesity presents to the Emergency Department with central crushing chest pain for the past three hours. He has a history of angina and hypertension.
_x000D_
High flow oxygen, sublingual GTN, and aspirin are administered and venous access is obtained. Whilst being attached to an electrocardiogram (ECG) monitor he collapses, and the initial rhythm is pulseless ventricular tachycardia (VT), as noted by a doctor present beside his bed at that time. The external defibrillator is located on another ward at a distance of three minutes from the ward where the gentleman was admitted.
_x000D_
Which one of the following is the most appropriate immediate treatment for this gentleman?
CorrectIncorrectHint
According to the guidelines set by the Resuscitation Council (UK), in the event of a monitored and witnessed ventricular fibrillation (VF)/ventricular tachycardia (VT) arrest occurring within a hospital, a specific approach should be followed. This approach involves administering three quick successive (stacked) shocks to the patient. Following the third shock, immediate initiation of chest compressions is crucial. The recommended compression to ventilation ratio is 30:2, and this should be maintained for a duration of 2 minutes. It is worth noting that a precordial thump, which involves delivering a forceful blow to the chest, can be effective if performed within seconds of the onset of a shockable rhythm. However, it is important to prioritize calling for help and accessing a defibrillator. In this scenario, while awaiting the defibrillator, a precordial thump may be considered. If the thump is unsuccessful, chest compressions should be promptly initiated. Furthermore, once chest compressions have commenced, it is recommended to administer intravenous adrenaline every 3-5 minutes. This serves to support the resuscitation efforts and maintain the circulation of the patient. Adherence to these guidelines ensures a systematic and organized approach to managing VF/VT arrests in a hospital setting.
_x000D_
The other options are incorrect because:
_x000D_
- _x000D_
- Option A. Continuous chest compressions should be started: This option is partially correct, as chest compressions are an essential component of cardiopulmonary resuscitation (CPR) for pulseless ventricular tachycardia (pVT). However, chest compressions alone are unlikely to restore a normal rhythm, and a precordial thump should be attempted first if a defibrillator is not available.
- Option B. Intravenous adrenaline should be given: This option is incorrect, as intravenous adrenaline is not indicated for the initial treatment of pulseless ventricular tachycardia (pVT). Adrenaline is a vasopressor that can increase the blood pressure and perfusion of vital organs during CPR, but it has no direct antiarrhythmic effect on pulseless ventricular tachycardia (pVT). Adrenaline may be given after the first or second shock if the pulseless ventricular tachycardia (pVT) persists, but only as an adjunct to defibrillation and cardiopulmonary resuscitation (CPR).
- Option C. A ventilation to compression ratio of 30:2 should be commenced: This option is incorrect, as a ventilation to compression ratio of 30:2 is not recommended for the initial treatment of pulseless ventricular tachycardia (pVT). Ventilation is the process of providing artificial breaths to the patient, either by mouth-to-mouth or by a bag-valve-mask device. Ventilation can improve the oxygenation of the blood, but it can also interrupt chest compressions and reduce the coronary perfusion pressure. Ventilation should be delayed until an advanced airway is in place, and the compression rate should be maintained at 100 to 120 per minute without pauses.
- Option D. Await arrival of defibrillator, then deliver shock: This option is incorrect, as awaiting the arrival of defibrillator without any intervention is a waste of precious time and can worsen the patient’s outcome. Defibrillation is the most effective treatment for pulseless ventricular tachycardia (pVT), as it delivers an electrical shock to the heart that can terminate the abnormal rhythm and restore a normal one. Defibrillation should be performed as soon as possible, but not at the expense of delaying cardiopulmonary resuscitation (CPR) or a precordial thump.
_x000D_
_x000D_
_x000D_
_x000D_
-
Question 119 of 141
119. Question
A 56-year-old gentleman with a well-controlled type 2 diabetes mellitus (T2DM) and no previous history of coronary heart disease presents at annual review as usual.
_x000D_
Currently he takes metformin 500 mg twice daily, aspirin 75 mg once daily, perindopril 4 mg once daily, and simvastatin 20 mg once daily.
_x000D_
On examination, his blood pressure is 140/70 mmHg, he has background diabetic retinopathy, and has a peripheral sensory neuropathy to light touch in the feet.
_x000D_
Investigations Reveal:
_x000D_
_x000D_ _x000D_
_x000D_ _x000D_ Serum HbA1c
_x000D_
_x000D_
_x000D_ 7.2%
_x000D_
_x000D_
_x000D_ (3.8-6.4)
_x000D_
_x000D_
_x000D_
_x000D_ _x000D_ 55 mmol/mol
_x000D_
_x000D_
_x000D_ (18-46)
_x000D_
_x000D_
_x000D_
_x000D_ _x000D_ Serum Total Cholesterol
_x000D_
_x000D_
_x000D_ 4.0 mmol/L
_x000D_
_x000D_
_x000D_ (<5.2)
_x000D_
_x000D_
_x000D_
_x000D_ _x000D_ Serum Triglycerides
_x000D_
_x000D_
_x000D_ 2.7 mmol/L
_x000D_
_x000D_
_x000D_ (0.45-1.69)
_x000D_
_x000D_
_x000D_
_x000D_ _x000D_ Serum High-Density Lipoprotein (HDL) Cholesterol
_x000D_
_x000D_
_x000D_ 0.6 mmol/L
_x000D_
_x000D_
_x000D_ (>1.55)
_x000D_
_x000D_
_x000D_
_x000D_ _x000D_ Low-Density Lipoprotein (LDL) Cholesterol
_x000D_
_x000D_
_x000D_ 2.2 mmol/L
_x000D_
_x000D_
_x000D_ (<3.36)
_x000D_
_x000D_
_x000D_
_x000D_
_x000D_
Which one of the following treatment options will further improve this gentleman’s dyslipidaemia?
CorrectIncorrectHint
The correct answer is B. Fenofibrate.
_x000D_
This gentleman has dyslipidaemia, which is a common complication of type 2 diabetes mellitus (T2DM). Dyslipidaemia is characterized by low high-density lipoprotein (HDL) cholesterol, high triglycerides, and increased levels of small, dense low-density lipoprotein (LDL) cholesterol particles. Dyslipidaemia increases the risk of cardiovascular disease (CVD), which is the leading cause of death in patients with type 2 diabetes mellitus (T2DM).
_x000D_
The primary goal of treatment for dyslipidaemia in type 2 diabetes mellitus (T2DM) is to lower low-density lipoprotein (LDL) cholesterol, as this has been shown to reduce events of cardiovascular disease (CVD) and mortality. Statins are the first-line therapy for low-density lipoprotein (LDL) cholesterol lowering, and this gentleman is already taking simvastatin 20 mg once daily. However, his low-density lipoprotein (LDL) cholesterol level is at the recommended target of <2.6 mmol/L (<100 mg/dL) for patients with type 2 diabetes mellitus (T2DM) and no previous history of cardiovascular disease (CVD). Therefore, he may not benefit from a more intensive statin therapy or the addition of another lipid-lowering agent for low-density lipoprotein (LDL) cholesterol lowering.
_x000D_
However, this gentleman also has low high-density lipoprotein (HDL) cholesterol and high triglycerides, which are associated with increased risk of cardiovascular disease (CVD), especially in patients with type 2 diabetes mellitus (T2DM). Some authorities advocate desirable high-density lipoprotein (HDL) cholesterol levels > 1 mmol/L and plasma triglycerides < 1.7 mmol/L in subjects at risk of cardiovascular disease (CVD). Fenofibrate is a fibrate that can increase high-density lipoprotein (HDL) cholesterol by 10-15% and reduce triglycerides by 15-20%. Fenofibrate has also been shown to reduce events of cardiovascular disease (CVD) in patients with type 2 diabetes mellitus (T2DM) and dyslipidaemia in the FIELD study, although the primary endpoint of cardiovascular disease (CVD) mortality was not achieved. Therefore, fenofibrate may be a suitable option for this gentleman to improve his dyslipidaemia and reduce his risk of cardiovascular disease (CVD).
_x000D_
The other options are less likely to provide significant benefits for this gentleman. Cholestyramine is a bile acid sequestrant that can lower low-density lipoprotein (LDL) cholesterol by 15-30%, but it has no effect on high-density lipoprotein (HDL) cholesterol or triglycerides. It may also cause gastrointestinal side effects, such as constipation, bloating, and nausea. Ezetimibe is a cholesterol absorption inhibitor that can lower low-density lipoprotein (LDL) cholesterol by 15-20%, but it has no effect on high-density lipoprotein (HDL) cholesterol or triglycerides. It is usually used as an add-on therapy to statins, not as a monotherapy. Rosuvastatin is a potent statin that can lower low-density lipoprotein (LDL) cholesterol by up to 55%. It has also been shown to reduce cardiovascular disease (CVD) events and mortality in patients with type 2 diabetes mellitus (T2DM) and high risk of cardiovascular disease (CVD). However, this gentleman has no previous history of cardiovascular disease (CVD) and his low-density lipoprotein (LDL) cholesterol is already at the target level. Therefore, rosuvastatin may not be necessary or beneficial for him. No other treatment required is not a valid option, as this gentleman has suboptimal lipid levels and high risk of cardiovascular disease (CVD), and he needs further intervention to prevent future complications.
-
Question 120 of 141
120. Question
A 55-year-old lady who is by profession a receptionist, came to annual check up at cardiology clinic. She was suffering from essential hypertension for last one year and was on extensive life-style modification, active physical exercise and tablet ramipril 5 mg daily. On examination it was found that her blood pressure came down to 124/74 mmHg from 164/92 mmHg. She had lost seven kg weight in last one year. Currently she is physically better, no abnormality was noted in general and systemic examinations. Her recent blood parameters are suggestive of:
_x000D_
_x000D_
_x000D_ _x000D_
_x000D_ _x000D_ Serum Sodium
_x000D_
_x000D_
_x000D_ 144 mmol/L
_x000D_
_x000D_
_x000D_ (137-144)
_x000D_
_x000D_
_x000D_
_x000D_ _x000D_ Serum Potassium
_x000D_
_x000D_
_x000D_ 4.0 mmol/L
_x000D_
_x000D_
_x000D_ (3.5-4.9)
_x000D_
_x000D_
_x000D_
_x000D_ _x000D_ Serum Urea
_x000D_
_x000D_
_x000D_ 5.2 mmol/L
_x000D_
_x000D_
_x000D_ (2.5-7.5)
_x000D_
_x000D_
_x000D_
_x000D_ _x000D_ Serum Creatinine
_x000D_
_x000D_
_x000D_ 83 μmol/L
_x000D_
_x000D_
_x000D_ (60-110)
_x000D_
_x000D_
_x000D_
_x000D_ _x000D_ Serum Bilirubin
_x000D_
_x000D_
_x000D_ 5.7 µmol/L
_x000D_
_x000D_
_x000D_ (1-22)
_x000D_
_x000D_
_x000D_
_x000D_ _x000D_ Serum Alanine Aminotransferase (ALT)
_x000D_
_x000D_
_x000D_ 27 U/L
_x000D_
_x000D_
_x000D_ (5-35)
_x000D_
_x000D_
_x000D_
_x000D_ _x000D_ Alkaline Phosphatase (ALP)
_x000D_
_x000D_
_x000D_ 88 U/L
_x000D_
_x000D_
_x000D_ (45-105)
_x000D_
_x000D_
_x000D_
_x000D_ _x000D_ Serum Albumin
_x000D_
_x000D_
_x000D_ 30 g/L
_x000D_
_x000D_
_x000D_ (37-49)
_x000D_
_x000D_
_x000D_
_x000D_ _x000D_ Serum Total Protein
_x000D_
_x000D_
_x000D_ 88 g/L
_x000D_
_x000D_
_x000D_ (61-76)
_x000D_
_x000D_
_x000D_
_x000D_ _x000D_ Serum C-Reactive Protein (CRP)
_x000D_
_x000D_
_x000D_ 7 mg/L
_x000D_
_x000D_
_x000D_ (<10)
_x000D_
_x000D_
_x000D_
_x000D_ _x000D_ Electrocardiogram (ECG)
_x000D_
_x000D_
_x000D_ Normal Sinus Rhythm
_x000D_
_x000D_
_x000D_
_x000D_
_x000D_
What is the next appropriate step for controlling her blood pressure?
CorrectIncorrectHint
Incorporation of life-style modifications and anti-hypertensive drug have excellent effect on managing essential hypertension in a nice way here. As per the question she is at low cardiovascular risk zone. So, first priority should be titrating the dose of anti-hypertensive (ACEI-Ramipril). But life-style modification should be continued like previous way. The aim is to reduce and gradually stop the anti-hypertensive drug (if not needed). _x000D_
As per the blood parameters there is decrease albumin and raised total protein may create a suspicion of monoclonal gammopathy. So, estimation of serum calcium, serum protein electrophoresis, serum immunoglobulins, urinary Bence Jones protein, electrophoresis and myeloma screening might help to conclude the diagnosis further._x000D_
As she is having low cardiovascular risk so incorporating Aspirin as primary prevention of ischemic heart disease is not recommended here._x000D_
To access proteinuria and nephropathy a urine dipstick is more preferred investigation over 24 hours Urinary Protein detection as first screening tool. -
Question 121 of 141
121. Question
A 55-year-old lady previously came to cardiology clinic with complaints of progressively increasing breathlessness and palpitations seven months ago. She was obese, postmenopausal and recently detected diabetic. Her ECG showed atrial fibrillation for which she was being prescribed with digoxin, aspirin and low dose furosemide. She again came back with history of transient loss of consciousness along with right sided weakness of upper and lower limbs for last one day. Her non-contrast CT scan brain showed no significant abnormality and the echocardiography reports came to be normal.
_x000D_
What is the best next step for her therapy?
CorrectIncorrectHint
As she is suffering from several episodes of transient ischemic attacks (TIA), right sided hemiparesis, so possibility of thrombo-embolic events (ischemic cerebral attacks) secondary to atrial fibrillation is quite high (even if the echocardiogram and CT scan brain reveals normal reports). So, administration of anticoagulant is first priority here from the above options mentioned.
-
Question 122 of 141
122. Question
A 55-year-old gentleman with hypertension is managed on furosemide, ramipril and digoxin and has a satisfactory haemodynamic state. On routine review, electrocardiogram (ECG) showed a poor left ventricular function.
_x000D_
Which one of the following antihypertensives should be added to his current regimen?
CorrectIncorrectHint
The most appropriate antihypertensive to add to the current regimen of this gentleman with hypertension and poor left ventricular function is A. Carvedilol. This is because carvedilol is a non-cardioselective beta blocker that can reduce blood pressure, heart rate, and cardiac output, as well as improve left ventricular ejection fraction and reverse left ventricular hypertrophy. Carvedilol has also been shown to reduce mortality and morbidity in patients with heart failure.
_x000D_
The other options are less suitable for this gentleman. Diltiazem (B) and nifedipine (E) are calcium channel blockers that can lower blood pressure, but they may also have negative inotropic effects, meaning they can reduce the force of the heart’s contractions and worsen left ventricular function. Hydralazine (C) is a vasodilator that can decrease peripheral resistance and blood pressure, but it may also increase heart rate and cardiac output, which can be detrimental for patients with heart failure. Doxazosin (D) is an alpha blocker that can lower blood pressure, but it may also cause orthostatic hypotension, fluid retention, and reflex tachycardia, which can aggravate heart failure.
-
Question 123 of 141
123. Question
A 55-year-old lady house cleaner is undergoing investigation for breathlessness.
_x000D_
Which one of the following does not support a diagnosis of constrictive pericarditis?
CorrectIncorrectHint
The answer is E. Low jugular venous pressure (JVP) with no “x” and “y” descent. This does not support a diagnosis of constrictive pericarditis, because it is the opposite of what is expected in this condition.
_x000D_
Constrictive pericarditis is a condition where the pericardium, the thin membrane that surrounds the heart, becomes stiff and thick, limiting the heart’s ability to fill with blood. This causes increased pressure in the heart chambers and the veins that return blood to the heart.
_x000D_
One of the signs of constrictive pericarditis is a high jugular venous pressure (JVP) with prominent “x” and “y” descents. The jugular venous pressure (JVP) is the height of the blood column in the jugular vein, which reflects the pressure in the right atrium. The “x” and “y” descents are the downward movements of the jugular venous pressure (JVP) waveform that correspond to the atrial relaxation and ventricular contraction, respectively.
_x000D_
In constrictive pericarditis, the jugular venous pressure (JVP) is elevated because of the high right atrial pressure and the impaired filling of the right ventricle. The “x” and “y” descents are exaggerated because of the rapid emptying of the right atrium and the sudden cessation of the right ventricular filling due to the rigid pericardium.
_x000D_
Therefore, a low jugular venous pressure (JVP) with no “x” and “y” descent would not support a diagnosis of constrictive pericarditis, as it would indicate a normal or reduced right atrial pressure and a normal or increased right ventricular compliance.
_x000D_
The other options are all consistent with constrictive pericarditis, as they are manifestations of the impaired cardiac output and the systemic venous congestion caused by this condition:
_x000D_
- _x000D_
- A. Previous cardiac surgery: This is a possible cause of constrictive pericarditis, as the pericardium may become scarred or calcified after a surgical procedure.
- B. Peripheral oedema: This is a common symptom of constrictive pericarditis, as the increased pressure in the systemic veins leads to fluid accumulation in the tissues, especially in the lower extremities.
- C. Orthopnoea: This is a symptom of constrictive pericarditis, as the increased pressure in the pulmonary veins causes fluid to leak into the lungs, resulting in shortness of breath that worsens when lying down.
- D. Ascites: This is a symptom of constrictive pericarditis, as the increased pressure in the hepatic veins and the portal system leads to fluid accumulation in the abdomen.
_x000D_
_x000D_
_x000D_
_x000D_
-
Question 124 of 141
124. Question
A 55-year-old gentleman is brought to the Emergency Department with sudden onset acute dyspnoea following a convulsion.
_x000D_
On examination, his blood pressure was 220/120 mmHg and fundal examination reveals papilloedema with haemorrhages and cotton wool spots. His serum urea, serum electrolytes, and serum creatinine are normal but chest x-ray reveals pulmonary oedema and cardiomegaly.
_x000D_
Which one of the following is the most appropriate immediate treatment for this gentleman?
CorrectIncorrectHint
The correct answer to the above question is option B. Intravenous sodium nitroprusside. This is because this gentleman has a hypertensive emergency with malignant hypertension and acute pulmonary oedema, which requires immediate and controlled reduction of blood pressure to prevent further end-organ damage. Intravenous sodium nitroprusside is a potent vasodilator that can lower blood pressure rapidly and smoothly by dilating both arterial and venous vessels. It can also improve the cardiac output and reduce the pulmonary capillary wedge pressure in patients with pulmonary oedema. However, it has some drawbacks, such as cyanide toxicity, rebound hypertension, and increased intracranial pressure, which require careful monitoring and titration. In the United Kingdom, intravenous sodium nitroprusside is the preferred agent for hypertensive emergencies, while in the United States, other agents such as labetalol or nicardipine are more commonly used.
_x000D_
The other options are incorrect for the following reasons:
_x000D_
- _x000D_
- Option A. Nifedipine 5 mg sublingually: Nifedipine is a calcium channel blocker that can cause unpredictable and excessive blood pressure reduction, which may lead to cerebral or myocardial ischaemia. It can also worsen pulmonary oedema by increasing pulmonary venous pressure. Sublingual nifedipine is not recommended for hypertensive emergencies and may even be harmful.
- Option C. Intravenous labetalol: Labetalol is a beta-blocker with alpha-blocking activity that can lower blood pressure rapidly and safely without causing reflex tachycardia or coronary vasospasm. It can also reduce the afterload and improve the cardiac function in patients with pulmonary oedema. However, it is not favoured as the first-line treatment option in the United Kingdom because it can result in rapid reduction of blood pressure which can lead to cerebral infarction in watershed areas. It may also cause bradycardia, heart block, or bronchospasm in some patients.
- Option D. Atenolol 50 mg orally: Atenolol is a beta-blocker that can lower blood pressure, but it is not suitable for acute use because it has a slow onset of action and may cause bradycardia or heart block. It can also worsen pulmonary oedema by reducing cardiac output. Oral formulations are not recommended in the initial stages of management as they usually do not have as rapid an onset of action, and cannot be titrated as quickly as is required.
- Option E. Nifedipine LA 30 mg orally: Nifedipine LA is a long-acting formulation of nifedipine, which has the same drawbacks as the sublingual form, but with a delayed and prolonged effect. It can cause unpredictable and excessive blood pressure reduction, which may lead to cerebral or myocardial ischaemia. It can also worsen pulmonary oedema by increasing pulmonary venous pressure. Oral formulations are not recommended in the initial stages of management as they usually do not have as rapid an onset of action, and cannot be titrated as quickly as is required.
_x000D_
_x000D_
_x000D_
_x000D_
-
Question 125 of 141
125. Question
A 55-year-old gentleman is admitted with an inferior myocardial infarction (MI). He receives thrombolysis, makes an uneventful recovery and gets discharged on atenolol, aspirin and atorvastatin.
_x000D_
He is keen to know how long after his myocardial infarction (MI) he must wait before he can drive.
CorrectIncorrectHint
The answer to the above question is D. One month. This is based on the guidelines from the Driver and Vehicle Licensing Agency (DVLA) of the United Kingdom, which state that drivers who have had a myocardial infarction (MI) must refrain from driving for at least four weeks after the event. This is to ensure that the driver has recovered sufficiently and does not pose a risk to themselves or others on the road. The same rule applies to drivers who have had surgical revascularisation, such as coronary artery bypass grafting (CABG) or percutaneous coronary intervention (PCI). However, drivers who have had percutaneous transluminal coronary angioplasty (PTCA) for stable angina can resume driving after one week, as long as they have no complications or symptoms
-
Question 126 of 141
126. Question
A 54-years-old gentleman, who is apparently asymptomatic, attends a Cardiology Clinic for a review. On detailed enquiry he informed of a significant family history of Ischaemic Heart Disease. Currently he smokes of 15 cigarettes per day and drinks about 18 units of alcohol per week. On clinical examination, he is significantly obese (BMI of 34 Kg/m2) and his vitals are
_x000D_
BP 150/90 mmHg.
_x000D_
Pulse 88 per minute, Regular
_x000D_
Respiratory Rate 16 per minute
_x000D_
Temperature 98.3 ̊ F
_x000D_
SpO2 96% in room air
_x000D_
Chest bilateral vesicular breath sound. Normal heart sound.
_x000D_
His basic blood parameters showed
_x000D_
_x000D_
_x000D_ _x000D_
_x000D_ _x000D_ Investigation
_x000D_
_x000D_
_x000D_ Test Value
_x000D_
_x000D_
_x000D_ Range
_x000D_
_x000D_
_x000D_
_x000D_ _x000D_ Fasting Plasma Glucose
_x000D_
_x000D_
_x000D_ 12.3 mmol/L
_x000D_
_x000D_
_x000D_ 3.0-6.0
_x000D_
_x000D_
_x000D_
_x000D_ _x000D_ Serum HbA1c
_x000D_
_x000D_
_x000D_ 68 mmol/mol
_x000D_
_x000D_
_x000D_ 20-46
_x000D_
_x000D_
_x000D_
_x000D_ _x000D_ Serum Cholesterol
_x000D_
_x000D_
_x000D_ 5.3 mmol/L
_x000D_
_x000D_
_x000D_ <5.2
_x000D_
_x000D_
_x000D_
_x000D_
_x000D_
_x000D_
From the given options which one is the most effective way to reduce his cardiovascular (CV) risk?
CorrectIncorrectHint
To reduce cardiovascular (CV) risk (primary prevention) the main pillar of therapy is lifestyle modification. It includes quitting smoking, reducing obesity, avoid unhealthy diet, incorporation of physical activity, correction of dyslipidaemia & hypertension and glycaemic status (in diabetic populations). So, among the options stopping cigarette smoking is the leading avoidable CV risk factor here. The Nurses Health Study in UK showed significant difference of CV risk among current smokers and past smokers in case of women. In males less significant were elicited. _x000D_
Improving Hypertension and Glycaemic Control along with dyslipidaemia have significant mortality benefit for reducing long term CV risk but first and best priority will be cessation of smoking. -
Question 127 of 141
127. Question
A 54-year-old gentleman with left ventricular systolic dysfunction was dyspnoeic on climbing stairs but not at rest. He was commenced on ramipril and furosemide as per his cardiologist’s advice.
_x000D_
Which one of the following drugs would improve the gentleman’s prognosis?
CorrectIncorrectHint
The best answer to this question is D. Bisoprolol. Bisoprolol is a type of beta blocker that can improve the survival and reduce the hospitalization of patients with systolic heart failure. It works by reducing the heart rate and the blood pressure, which lowers the workload of the heart.
_x000D_
Nitrate therapy, amlodipine and amiodarone are not recommended for systolic heart failure, as they may have adverse effects or no proven benefit. Digoxin may improve the symptoms of systolic heart failure, but it does not improve the prognosis.
-
Question 128 of 141
128. Question
A 52-year-old lady came to cardiology clinic with complaints of progressively increasing exertional dyspnoea (NYHA Grade lll) and history of paroxysmal dyspnoea for more than one year. She was on diuretic and angiotensin converting enzyme inhibitor for last three years. But the dyspnoea still persisted. On detailed discussion it was revealed, she had history of severe mitral regurgitation secondary to mitral prolapse. She was waiting for valvular surgery for last three months but due to some technical issues it was yet to be done.
_x000D_
What is the most common cause of Mitral Regurgitation (MR) in this kind of patients?
CorrectIncorrectHint
As per the latest data from NICE guidelines, most common cause of mitral regurgitation in UK is Myxomatous Degeneration of mitral valve. The management involves valvular surgery (replacement/repair).
-
Question 129 of 141
129. Question
A 52-year-old gentleman with a family history of ischaemic heart disease (IHD) and feeling tired of late attends for an insurance medical review.
_x000D_
Investigations Reveal:
_x000D_
_x000D_ _x000D_
_x000D_ _x000D_ Serum Total Cholesterol
_x000D_
_x000D_
_x000D_ 7.1 mmol/L
_x000D_
_x000D_
_x000D_ (<5.2)
_x000D_
_x000D_
_x000D_
_x000D_ _x000D_ Serum High-Density Lipoprotein (HDL) Cholesterol
_x000D_
_x000D_
_x000D_ 1.1 mmol/L
_x000D_
_x000D_
_x000D_ (>1.55)
_x000D_
_x000D_
_x000D_
_x000D_ _x000D_ Serum Triglycerides
_x000D_
_x000D_
_x000D_ 2.1 mmol/L
_x000D_
_x000D_
_x000D_ (0.45-1.69)
_x000D_
_x000D_
_x000D_
_x000D_ _x000D_ Serum Free Thyroxine (FT4)
_x000D_
_x000D_
_x000D_ 10.1 pmol/L
_x000D_
_x000D_
_x000D_ (10-22)
_x000D_
_x000D_
_x000D_
_x000D_ _x000D_ Serum Thyroid Stimulating Hormone (TSH)
_x000D_
_x000D_
_x000D_ 22.8 mU/L
_x000D_
_x000D_
_x000D_ (0.4-5)
_x000D_
_x000D_
_x000D_
_x000D_
_x000D_
Which one of the following is the most appropriate treatment for this gentleman’s dyslipidaemia?
CorrectIncorrectHint
Dyslipidaemia is a condition where the levels of lipids (fats) in the blood are abnormal. It can increase the risk of cardiovascular diseases, such as atherosclerosis, coronary artery disease, and stroke. The most common types of dyslipidaemia are high levels of low-density lipoprotein (LDL) cholesterol, low levels of high-density lipoprotein (HDL) cholesterol, and high levels of triglycerides.
_x000D_
The gentleman in the question has high levels of total cholesterol, low-density lipoprotein (LDL) cholesterol, and triglycerides, and low levels of high-density lipoprotein (HDL) cholesterol. He also has high levels of thyroid stimulating hormone (TSH) and normal levels of free thyroxine (FT4), which indicate subclinical hypothyroidism. Subclinical hypothyroidism is a condition where the thyroid gland does not produce enough thyroid hormones, which regulate the metabolism and affect various body functions. It can cause symptoms such as fatigue, weight gain, cold intolerance, and dry skin. It can also contribute to dyslipidaemia by reducing the clearance of low-density lipoprotein (LDL) cholesterol and triglycerides from the blood.
_x000D_
The most appropriate treatment for this gentleman’s dyslipidaemia is C. Thyroxine. Thyroxine is a synthetic form of the thyroid hormone T4, which is used to treat hypothyroidism. By restoring the normal thyroid function, thyroxine can improve the symptoms of subclinical hypothyroidism and lower the levels of low-density lipoprotein (LDL) cholesterol and triglycerides in the blood. Thyroxine can also increase the levels of high-density lipoprotein (HDL) cholesterol, which can help protect against cardiovascular diseases.
_x000D_
The other options are not as effective or suitable for this gentleman’s dyslipidaemia. Omega-3 fish oils, gemfibrozil, and ezetimibe are lipid-lowering drugs that can reduce the levels of triglycerides and low-density lipoprotein (LDL) cholesterol, but they do not address the underlying cause of subclinical hypothyroidism. They may also have side effects or interactions with other medications. Atorvastatin is a statin drug that can lower the levels of low-density lipoprotein (LDL) cholesterol and triglycerides, and increase the levels of high-density lipoprotein (HDL) cholesterol, but it is not recommended for patients with subclinical hypothyroidism until their thyroid function is normalized. Statins may also have side effects or interactions with other medications.
-
Question 130 of 141
130. Question
A 51-year-old lady comes to the clinic complaining of rapidly worsening lethargy, nausea and occasional vomiting. Over the past few days she has become increasingly unwell and has now become weak enough even to get out of her house.
_x000D_
Other symptoms of note include progressive shortness of breath and frequent cough productive of blood stained sputum. She consulted her physician only over the past eight months about the shape of her nose; she has suffered some collapse of her nasal bridge and is considering plastic surgery.
_x000D_
On examination, collapse of the bridge of her nose is noticed and nasal congestion when she speaks. Her blood pressure is elevated at 160/90 mmHg. On auscultation of the chest, crepitations are heard.
_x000D_
Investigations Show:
_x000D_
_x000D_ _x000D_
_x000D_ _x000D_ Haemoglobin
_x000D_
_x000D_
_x000D_ 125 g/L
_x000D_
_x000D_
_x000D_ (115-165)
_x000D_
_x000D_
_x000D_
_x000D_ _x000D_ Total Leucocyte Count
_x000D_
_x000D_
_x000D_ 11.8 ×109/L
_x000D_
_x000D_
_x000D_ (4-11)
_x000D_
_x000D_
_x000D_
_x000D_ _x000D_ Platelet Count
_x00